You are on page 1of 317

Siyavula: Life Sciences Grade 10

Collection Editors:
Siyavula
Megan Beckett
Siyavula: Life Sciences Grade 10

Collection Editors:
Siyavula
Megan Beckett
Authors:

Jesuit Virtual Learning Academy Colleen Henning


Siyavula Dr Zdzislaw (Gustav) Meglicki, Jr
Megan Beckett Zamekile Sondzaba
Tom Caswell Daniel Williamson
Denver Greene

Online:
< http://cnx.org/content/col11410/1.3/ >

CONNEXIONS

Rice University, Houston, Texas


This selection and arrangement of content as a collection is copyrighted by Siyavula, Megan Beckett. It is licensed
under the Creative Commons Attribution 3.0 license (http://creativecommons.org/licenses/by/3.0/).
Collection structure revised: April 11, 2012
PDF generated: October 29, 2012
For copyright and attribution information for the modules contained in this collection, see p. 304.
Table of Contents
1 Life at the molecular, cellular and tissue level
1.1 The chemistry of life . . . . . . . . . . . . . . . . . . . . . . . . . . . . . . . . . . . . . . . . . . . . . . . . . . . . . . . . . . . . . . . . . . . . . . . . 1
1.2 Cells - the basic units of life . . . . . . . . . . . . . . . . . . . . . . . . . . . . . . . . . . . . . . . . . . . . . . . . . . . . . . . . . . . . . . . . . 1
1.3 Cell division - mitosis . . . . . . . . . . . . . . . . . . . . . . . . . . . . . . . . . . . . . . . . . . . . . . . . . . . . . . . . . . . . . . . . . . . . . . . 1
1.4 Plant and animal tissues . . . . . . . . . . . . . . . . . . . . . . . . . . . . . . . . . . . . . . . . . . . . . . . . . . . . . . .. . . . . . . . . . . . . 25
2 Life processes in plants and animals
2.1 Support and transport systems in plants . . . . . . . . . . . . . . . . . . . . . . . . . . . . . . . . . . . . . . .. . . . . . . . . . . . . 53
2.2 Support systems in animals . . . . . . . . . . . . . . . . . . . . . . . . . . . . . . . . . . . . . . . . . . . . . . . . . . . .. . . . . . . . . . . . . 75
2.3 Transport systems in mammals (humans) . . . . . . . . . . . . . . . . . . . . . . . . . . . . . . . . . . . . . .. . . . . . . . . . . . 114
3 Environmental studies
3.1 Biospheres to ecosystems . . . . . . . . . . . . . . . . . . . . . . . . . . . . . . . . . . . . . . . . . . . . . . . . . . . . . . . . . . . . . . . . . . 155
4 Diversity, change and continuity
Biodiversity and classication . . . . . . . . . . . . . . . . . . . . . . . . . . . . . . . . . . . . . . . . . . . . . . . . . . . . . . . . . . . . . 223
4.1
History of life on Earth . . . . . . . . . . . . . . . . . . . . . . . . . . . . . . . . . . . . . . . . . . . . . . . . . . . . . . . .. . . . . . . . . . . . 231
4.2
Solutions . . . . . . . . . . . . . . . . . . . . . . . . . . . . . . . . . . . . . . . . . . . . . . . . . . . . . . . . . . . . . . . . . . . . . . . . . . . . . . . . . . . . . . . 281
5 Glossary . . . . . . . . . . . . . . . . . . . . . . . . . . . . . . . . . . . . . . . . . . . . . . . . . . . . . . . . . . . . . . . . . . . . . . . . .. . . . . . . . . . . . . . . . . 285
Glossary . . . . . . . . . . . . . . . . . . . . . . . . . . . . . . . . . . . . . . . . . . . . . . . . . . . . . . . . . . . . . . . . . . . . . . . . . . . . . . . . . . . . . . . . . . . . 302
Index . . . . . . . . . . . . . . . . . . . . . . . . . . . . . . . . . . . . . . . . . . . . . . . . . . . . . . . . . . . . . . . . . . . . . . . . . . . . . . . . . . . . . . . . . . . . . . . 303
Attributions . . . . . . . . . . . . . . . . . . . . . . . . . . . . . . . . . . . . . . . . . . . . . . . . . . . . . . . . . . . . . . . . . . . . . . . . . . . . . . . . . . . . . . . .304
iv

Available for free at Connexions <http://cnx.org/content/col11410/1.3>


Chapter 1

Life at the molecular, cellular and tissue

level

1.1 The chemistry of life


1.2 Cells - the basic units of life
1.3 Cell division - mitosis
1.3.1 1.3.1 The Cell Cycle and Mitosis1
Unit 1.3 Cell Division - Mitosis
The Cell Cycle and Mitosis
Animation on mitosis: http://www.sumanasinc.com/webcontent/animations/content/mitosis.html 2

Khan: Phases of mitosishttp://www.khanacademy.org/video/phases- ofmitosis?playlist=Biology


Neok12: Mitosis games and videos http://www.neok12.com/Cell-Division.htm 3

Cell Cycle:http://www.daviddarling.info/encyclopedia/C/cell_cycle.html
Khan: Chromosomes, Chromatids, Chromatin, etc: http://www.khanacademy.org/video/chromosomes
chromatidschromatin tc?playlist=Biology
1.3.1.1 Introduction

The cell cycle is the series of events that takes place in a cell leading to its division and duplication
4

(replication). In cells without a nucleus ( prokaryotic cells e.g. bacteria), the cell cycle occurs through a
5

process termed binary ssion . In cells with a nucleus ( eukaryotes ), the cell cycle can be divided in
6 7

two brief periods: interphase during which the cell grows, accumulating nutrients needed for mitosis
8

and duplicating its DNA and the mitosis (M) phase, just after which the cell splits itself into two
9 10

distinct cells, often called "daughter cells". The cell-division cycle is a vital process by which a single-celled
1 This content is available online at <http://cnx.org/content/m43135/1.1/>.
2 http://www.sumanasinc.com/webcontent/animations/content/mitosis.html
3 http://www.neok12.com/Cell-Division.htm
4 http://en.wikipedia.org/wiki/Cell_(biology)
5 http://en.wikipedia.org/wiki/Prokaryotic
6 http://en.wikipedia.org/wiki/Binary_ssion
7 http://en.wikipedia.org/wiki/Eukaryotes
8 http://en.wikipedia.org/wiki/Interphase
9 http://en.wikipedia.org/wiki/DNA_replication
10 http://en.wikipedia.org/wiki/Mitosis

Available for free at Connexions <http://cnx.org/content/col11410/1.3>

1
2 CHAPTER 1. LIFE AT THE MOLECULAR, CELLULAR AND TISSUE
LEVEL
fertilized egg develops into a mature organism, as well as the process by which hair
11 12
, skin 13
, blood
cells , some internal organs are renewed and wounds are healed
14

Figure 1.1

Diagram - Cell division.


1.3.1.2 Phases

The cell cycle consists of four distinct phases: G 1 15


phase , S phase (synthesis), G
16 17 18
2 19

20
phase (collectively known as interphase ) and M phase (mitosis). M phase is itself composed
21 22 23

of two tightly coupled processes: mitosis, in which the cell's chromosomes are divided between the two
24

daughter cells, and cytokinesis , in which the cell's cytoplasm divides in half forming two distinct
25 26

cells.
11 http://en.wikipedia.org/wiki/Fertilized_egg
12 http://en.wikipedia.org/wiki/Hair
13 http://en.wikipedia.org/wiki/Skin
14 http://en.wikipedia.org/wiki/Blood_cell
15 http://en.wikipedia.org/wiki/G1_phase
16 http://en.wikipedia.org/wiki/G1_phase
17 http://en.wikipedia.org/wiki/G1_phase
18 http://en.wikipedia.org/wiki/S_phase
19 http://en.wikipedia.org/wiki/G2_phase
20 http://en.wikipedia.org/wiki/G2_phase
21 http://en.wikipedia.org/wiki/G2_phase
22 http://en.wikipedia.org/wiki/Interphase
23 http://en.wikipedia.org/wiki/Mitosis
24 http://en.wikipedia.org/wiki/Chromosomes
25 http://en.wikipedia.org/wiki/Cytokinesis
26 http://en.wikipedia.org/wiki/Cytoplasm

Available for free at Connexions <http://cnx.org/content/col11410/1.3>


3

Figure 1.2

Diagram - Schematic of the cell cycle. outer ring: I = Interphase , M = Mitosis ; inner ring: M
27 28

= Mitosis , G = Gap 1 , G = Gap 2 , S = Synthesis ; not in ring: G = Gap


29
1
30
2
31 32
0
0/Resting . [1]
33

Phase
Abbreviation
Description

quiescent/senescent Gap 0 34
G 0 A resting phase where the cell has left the cycle and has stopped
dividing.

27 http://en.wikipedia.org/wiki/Interphase
28 http://en.wikipedia.org/wiki/Mitosis
29 http://en.wikipedia.org/wiki/Mitosis
30 http://en.wikipedia.org/wiki/G1_phase
31 http://en.wikipedia.org/wiki/G2_phase
32 http://en.wikipedia.org/wiki/S_phase
33 http://en.wikipedia.org/wiki/G0_phase

Available for free at Connexions <http://cnx.org/content/col11410/1.3>


4 CHAPTER 1. LIFE AT THE MOLECULAR, CELLULAR AND TISSUE
LEVEL

Interphase Gap 1
35
G 1 Cells increase in size in Gap 1. The
36
G 1 checkpoint control
mechanism ensures that everything is ready for DNA synthesis. 37

Synthesis S38

DNA replication 39
occurs during this phase.

Gap 2 40
G 2 During the gap between DNA synthesis and mitosis, the cell will continue to grow.
The G 2 checkpoint control mechanism ensures that everything is ready to enter the M (mitosis) phase
and divide.

Cell division Mitosis


41
M 42

Cell growth stops at this stage and cellular energy is focused on the orderly division into two daughter
cells. A checkpoint in the middle of mitosis ( Metaphase Checkpoint ) ensures that the cell is ready to
complete cell division.

Phase
Abbreviation
Description

quiescent/senescent Gap 0 34
G 0 A resting phase where the cell has left the cycle and has stopped
dividing.

Interphase Gap 1
35
G 1 Cells increase in size in Gap 1. The
36
G 1 checkpoint control
mechanism ensures that everything is ready for DNA synthesis. 37

Synthesis S38

DNA replication 39
occurs during this phase.

Gap 2 40
G 2 During the gap between DNA synthesis and mitosis, the cell will continue to grow.
The G 2 checkpoint control mechanism ensures that everything is ready to enter the M (mitosis) phase
and divide.

Available for free at Connexions <http://cnx.org/content/col11410/1.3>


5
Cell division Mitosis
41
M 42

Cell growth stops at this stage and cellular energy is focused on the orderly division into two daughter
cells. A checkpoint in the middle of mitosis ( Metaphase Checkpoint ) ensures that the cell is ready to
complete cell division.

Phase
Abbreviation
Description

quiescent/senescent Gap 0 43
G 0 A resting phase where the cell has left the cycle and has stopped
dividing.

Interphase Gap 1
44
G 1 Cells increase in size in Gap 1. The
45
G 1 checkpoint control
mechanism ensures that everything is ready for DNA synthesis. 46

Synthesis S47

DNA replication 48
occurs during this phase.

Gap 2 49
G 2 During the gap between DNA synthesis and mitosis, the cell will continue to grow.
The G 2 checkpoint control mechanism ensures that everything is ready to enter the M (mitosis) phase
and divide.

Cell division Mitosis


50
M 51

Cell growth stops at this stage and cellular energy is focused on the orderly division into two daughter
cells. A checkpoint in the middle of mitosis ( Metaphase Checkpoint ) ensures that the cell is ready to
complete cell division.

Available for free at Connexions <http://cnx.org/content/col11410/1.3>


6 CHAPTER 1. LIFE AT THE MOLECULAR, CELLULAR AND TISSUE
LEVEL

State Phase Abbreviation Description

quiescent/senescent Gap 0 34
G 0 A resting phase where the cell has left the cycle and has stopped
Interphase 35
Gap 1 36
G 1 Cells increase in size in Gap 1. The G 1 checkpoint control
Synthesis 38
S DNA replication occurs during this phase.
39

Gap 2 40
G 2 During the gap between DNA synthesis and mitosis, the cell wil
Cell division 41
Mitosis 42
M Cell growth stops at this stage and cellular energy is focused on
Table 1.1

Table  Phases of the cell cycle (the details of the G and S phases are not required but are included to
give an overview).
34 http://en.wikipedia.org/wiki/G0_phase
35 http://en.wikipedia.org/wiki/Interphase
36 http://en.wikipedia.org/wiki/G1_phase
37 http://en.wikipedia.org/wiki/DNA
38 http://en.wikipedia.org/wiki/S_phase
39 http://en.wikipedia.org/wiki/DNA_replication
40 http://en.wikipedia.org/wiki/G2_phase
41 http://en.wikipedia.org/wiki/Cell_division
42 http://en.wikipedia.org/wiki/Mitosis

Available for free at Connexions <http://cnx.org/content/col11410/1.3>


7
1.3.1.3 Stages of Mitosis

Figure 1.3

Diagram  Allium (Onion) cells in the dierent cycle of mitosis.


1.3.1.3.1 Interphase

The cell spends most of its life in the interphase. During this phase the cell grows to its maximum size and
performs its normal functions. Many scientists do not count interphase as part of mitosis.
1.3.1.3.2 Prophase

The chromatin (a special protein (actually a nucleoprotein) that chromosomes are made of) condenses into
chromosomes (human cells have 46 chromosomes  23 from your father and 23 from your mother). Each
chromosome eventually can be seen to consist of two strands or chromatids joined at a central centromere
in an X shape. The nuclear membrane disappears. The centriole splits and starts to move to opposite poles.
Spindle threads form between the poles.

Available for free at Connexions <http://cnx.org/content/col11410/1.3>


8 CHAPTER 1. LIFE AT THE MOLECULAR, CELLULAR AND TISSUE
LEVEL
1.3.1.3.3 Metaphase

Chromosomes lie on the equator of the cell. Each chromosome is attached to the spindle microbers by its
centromere. The chromosomes appear in a straight line across the middle of the cell.
In the other form of cell division, meiosis, homologous chromosomes line up in pairs, side by side.
1.3.1.3.4 Anaphase

The centromere splits. Each chromosome divides into two sister chromatids. Each chromatid is moved to
opposite poles of the cell by the shortening of the spindle bres.
Chromatids (now called daughter chromosomes ) gather at opposite poles of the cell.
1.3.1.3.5 Telophase

A nuclear membrane forms around each of the daughter chromosomes that have gathered at the poles. The
daughter chromosomes uncoil to form diuse chromatin. The cytoplasm then divides during a process called
cytokinesis . Note cytokinesis is not a stage of mitosis but the process of the cytoplasm splitting into
two. There are now two genetically identical daughter cells. They are identical to the parent cell and to each
other. In an animal cell an invagination or infolding will divide the cytoplasm. In a plant cell a cross
wall divides the cytoplasm.
Animation  Cell cycle and stages of mitosis 
http://highered.mcgraw-hill.com/sites/0072495855/student_view0/chapter2/animatio
n__how_the_cell_cycle_works.html 43

1.3.1.3.6 Summary of mitosis

Two identical daughter cells are formed from the mother cell.
Each daughter cell has the same number of chromosomes as the mother cell.
Each daughter cell will grow to its maximum size.
1.3.1.4 Biological importance of mitosis

Growth  Living tissue grows by mitosis e.g. bone and skin.


Repair - Damaged and worn-out tissues are replaced with new cells by mitosis.
Asexual reproduction - Single-celled (unicellular) organisms and bacteria often reproduce asexually by
mitosis. Organisms like amoeba are able to split from a single individual into two and therefore can reproduce
without a mate and sexual reproduction.
1.3.1.5 Chromosomes

In mitosis at the end of prophase the chromosomes appear as X-shaped threads. Each thread is in fact a
chromatid and they are joined in the centre at a point called the centromere. There are two of each chromo-
some and the full set of chromosomes is often shown with the complimentary or homologous chromosomes
paired up.
43 http://highered.mcgraw-hill.com/sites/0072495855/student_view0/chapter2/animation__how_the_cell_cycle_works.html

Available for free at Connexions <http://cnx.org/content/col11410/1.3>


9

Figure 1.4

1.3.1.6 Activity 1 - Investigating mitosis in allium root tip squash

Introduction
Talking about what chromosomes do during mitosis could be very interesting, but seeing them for yourself
adds an extra dimension.
Video - Preparing Microscope Slides
http://www.youtube.com/watch?v=GHnndVuaync 44

Lesson Organisation
The allium/onoin roots need to be prepared 1-10 days in advance of the lesson. Some practitioners
report that cutting the root tips around noon makes a dierence to the mitotic index, so you may want
your technician to cut and `x' the tips in ethanoic alcohol rather than ask your students to carry out this
step. If you have access to a video microscope it is worth capturing some images as this procedure can be
frustrating.
Method
1. Pour approximately 5 ml. methanol-acetic acid xative into a small beaker. Place 2-3 mm length
onion root tip into the xative. Incubate at 60 C for 15 mins. 2. At the end of the xative incubation period,
pour o the xative into a waste beaker. Be careful not to pour o your xed onion root tip. Now add
approximately 5 ml. 1 M HCl to your xed onion root tip to partially hydrolize the cells. Incubate at 60
C for exactly 10 mins. 3. At the end of the hydrolysis incubation period, pour o the 1 M HCl into a
waste beaker. Be careful not to pour o your xed/hydrolyzed onion root tip. Now add approximately 1 ml.
44 http://www.youtube.com/watch?v=GHnndVuaync

Available for free at Connexions <http://cnx.org/content/col11410/1.3>


10 CHAPTER 1. LIFE AT THE MOLECULAR, CELLULAR AND TISSUE
LEVEL
Feulgen stain to your xed/hydrolyzed onion root tip in order to stain the chromosomes. Bath your onion
root tip in the Feulgen stain for 20-30 mins. to allow the Feulgen stain to penetrate the chromosomes.4. To
make a slide of your stained onion root tip, transfer your onion root tip from the beaker to a microscope
slide and add a small drop of 45% acetic acid. Do not allow the onion root tip to dry out during
the subsequent steps. Add 45% acetic acid if you notice your specimen is drying out. 5.
Pulverize your onion root tip into a ne pulp on the microscope slide by tapping it with a glass rod. Try
to produce as ne a pulp as possible to prevent large cell clumps which will not be useful for microscopic
examination.6. Now place a microscope cover slip on top of your pulverized onion root tip. Put two layers
of paper towel on top of the microscope cover slip and press down hard enough to squash the root but not
enough to break the cover slip. This should result in the onion root tip cells from forming a monolayer which
is ideal for microscopic examination.7. Using the scanning objective focus on the onion root tip cells and
identify a cell undergoing mitosis by looking for pink-staining bodies (chromosomes) within the cell.8. Switch
to the low-magnication10X objective and ne-focus.9. Switch to the high-magnication 40X objective and
ne-focus. At this magnication you should be able to identify cells in several stages of mitosis. Identify a
specic stage of mitosis and go on to step 10.10. Add a drop of oil to your prepared slide and switch to the
oil-immersion 100X objective and ne-focus. Repeat steps 9 and 10 for all four stages of mitosis.
As part of your Lab Exit Quiz you will be asked to: 1) show your prepared onion root tip slide
to a lab instructor under the microscope.2) identify several stages of mitosis on your prepared
onion root tip slide.
Mitotic Index

Figure 1.5

The duration of each stage of mitosis has been recorded and the data (see table below) could be used to
compare the observed frequencies of the dierent stages as recorded by students.
1.3.1.7 Activity 2  Stages of Mitosis

Task  Look at Cells 1 -5. Decide which stage of mitosis each cell is in. For each cell describe the features
that make you think it is in this stage.

Available for free at Connexions <http://cnx.org/content/col11410/1.3>


11

Available for free at Connexions <http://cnx.org/content/col11410/1.3>


Figure 1.6
12 CHAPTER 1. LIFE AT THE MOLECULAR, CELLULAR AND TISSUE
LEVEL
1.3.1.8 Activity 3 - Quiz on Onion Tip Mitosis (with Answers)

1. 1. Why do we study the root tip to nd mitosis instead of any other part of the onion plant? We study
the root tip because it is growing therefore cells are dividing rapidly. This makes it the best part of the plant
to observe various stages of mitosis.
2. Based on you data what can you infer about the relative length of time an onion root-tip cell spends
in each stage of the cell cycle? Most cells are in interphase because most time is spent in this phase.
3. Based on your understanding of the structure of the chromosome, why might it take longer to complete
prophase than the other phases of nuclear division? Prophase is the longest phase of mitosis because the
chromosomes have to coil up into organized bodies. It takes a long time for the chromatin to coil or condense
into chromosomes.
4. How do you account for the dierences between the slides made by dierent groups? Possible answers:
Not all lab groups had the same slide so there can be dierences among the growth rates of the plants that
were used to prepare the slide. The groups may have been looking at dierent areas of the root. Some
groups may not have followed the instructions as carefully as others.
5. If you examined cells in the Zone of Dierentiation (Zone of Maturation) would you expect to get
similar results? No Why or why not? These cells are starting to specialize into mature tissues. They are
no longer meristematic cells.
6. Why did we use the pie chart to graph the data? The pie chart was used because the data represented
the parts of a whole and it is relatively easy to show proportions of the whole event.
1.3.1.9 Assignment 1 - Animation of Mitosis and Multiple Choice Questions on Mitosis

http://bealbio.wikispaces.com/Genetics 45

Description: Gives a detailed description of the steps involved in mitosis with animated videos and
narrative voice-over. Short multiple choice quiz provided containing the following multiple choice questions:

A)

Figure 1.7

continued on next page

45 http://bealbio.wikispaces.com/Genetics

Available for free at Connexions <http://cnx.org/content/col11410/1.3>


13
B)

Figure 1.9

C)

Figure 1.11

D)

Figure 1.13

E)

Figure 1.15

continued on next page

Available for free at Connexions <http://cnx.org/content/col11410/1.3>


14 CHAPTER 1. LIFE AT THE MOLECULAR, CELLULAR AND TISSUE
LEVEL
A)

Figure 1.17

B)

Figure 1.19

C)

Figure 1.21

D)

Figure 1.23

E)

Figure 1.25

continued on next page

Available for free at Connexions <http://cnx.org/content/col11410/1.3>


15

A)

Figure 1.27

B)

Figure 1.29

C)

Figure 1.31

D)

Figure 1.33

continued on next page

Available for free at Connexions <http://cnx.org/content/col11410/1.3>


16 CHAPTER 1. LIFE AT THE MOLECULAR, CELLULAR AND TISSUE
LEVEL
E)

Figure 1.35

A)

Figure 1.37

B)

Figure 1.39

A)

Figure 1.41

continued on next page

Available for free at Connexions <http://cnx.org/content/col11410/1.3>


17
B)

Figure 1.43

Table 1.2

1.3.1.10 Assignment 2 Observing Mitosis in an Onion Root Tip Activity

Mitosis Slides
Identify the stage of mitosis for each of the onion root-tip slides below (most stages are
represented more than once).

Identify st

__i)_____

Figure 1.45

continued on next page

Available for free at Connexions <http://cnx.org/content/col11410/1.3>


18 CHAPTER 1. LIFE AT THE MOLECULAR, CELLULAR AND TISSUE
LEVEL
______iii

Figure 1.47

_______

Figure 1.49

continued on next page

Available for free at Connexions <http://cnx.org/content/col11410/1.3>


19

Table 1.3

Practice locating each of the stages of mitosis in the following slides of the onion root tip.
Each picture contains at least one cell at each stage of mitosis (and some stages are represented
by multiple cells).

Figure 1.51

(vii)
Answers

i. Prophase; ii) Anaphase iii) Metaphase iv) Interphase v) Anaphase vi) Metaphase

Answers to (vii)
The numbered arrows indicate cells at various stages of mitosis (most of the rest of the
cells are in interphase). The key to the stages is to the right of each gure. You will not
be asked to distinguish early from late; this is provided to help clarify the more ambiguous
stages.

Available for free at Connexions <http://cnx.org/content/col11410/1.3>


20 CHAPTER 1. LIFE AT THE MOLECULAR, CELLULAR AND TISSUE
LEVEL

Key:

1. late P
2. Metap
3. Teloph
4. early T
5. early T
6. Metap
7. Proph
8. Anaph
9. late A

Figure 1.52

Table 1.4

1.3.2 1.3.2 Cancer46


Khan video: Cancer
http://www.khanacademy.org/video/cancer?playlist=Biology
Inside Cancer (very good summary)
http://www.insidecancer.org/
46 This content is available online at <http://cnx.org/content/m43059/1.1/>.

Available for free at Connexions <http://cnx.org/content/col11410/1.3>


21

Figure 1.53

Picture - An invasive ductal carcinoma of the breast (pale area at the center) surrounded by spikes of
whitish scar tissue in the surrounding yellow fatty tissue.
1.3.2.1 Introduction

Cancer is essentially a disease of mitosis - the normal 'checkpoints' regulating mitosis are ignored or over-
ridden by the cancer cell. Cancer begins when a single cell is transformed, or converted from a normal cell
to a cancer cell.
Cancer can be caused by agents called carcinogens, such as cigarette smoke, x-rays, nuclear radiation,
UV light and some chemicals.
Carcinogens can cause a DNA mutation that occurs in one of several genes that normally function to
control growth. E.g. the BRCA 1 gene, the "Breast Cancer Gene" normally functions to supress tumor
formation; but if a gene contains mutations such that BRCA1 does not work properly, tumor formation can
begin (Note: mutations in this gene do not mean that a person will develop breast cancer, just that they
have an increased risk for breast cancer).
Once these crucial Cell Cycle genes start behaving abnormally, cancer cells start to develop wildly by
repeated, uncontrolled mitosis.
Video - Very simply explained visual depiction of cancer. http://www.youtube.com/watch?v=LEpTTolebqo
47

47 http://www.youtube.com/watch?v=LEpTTolebqo

Available for free at Connexions <http://cnx.org/content/col11410/1.3>


22 CHAPTER 1. LIFE AT THE MOLECULAR, CELLULAR AND TISSUE
LEVEL
1.3.2.2 Tumours

The cancer cells grow and divide to form mass of cancer cells called a tumor. As the tumor grows larger, it
begins to release proteins from the cell to attract new blood vessel growth.
1.3.2.2.1 Benign Tumours

Tumour cells remain at original site. They do not spread and usually cause little trouble. They can be
removed surgically or killed by radiation, usually eliminating any further cancer development at that site.
1.3.2.2.2 Malignant Tumours

Malignant tumours are dangerous. The malignant cells grow squashing normal cells and preventing them
working normally. Some tumour cells send out signals that tell the body to produce a new blood vessel at
the tumour site. These cells not only have their own food and oxygen supply, they also have an avenue for
escape to a new part of the body - through the new blood vessel and into bloodstream. Cells that break
away from the tumour begin to spread to surrounding tissues (via the bloodstream or lymph) and start
new tumors = metastasis. Usually surgery is performed to remove the tumour, followed by radiation and
chemotherapy.
1.3.2.3 Cancer Treatment

1.3.2.3.1 Radiation therapy

Radiation therapy (also called radiotherapy, X-ray therapy, or irradiation) is the use of ionizing radiation to
kill cancer cells and shrink tumors. The eects of radiation therapy are localised and conned to the region
being treated. Radiation therapy injures or destroys cells in the area being treated (the "target tissue")
by damaging their genetic material, making it impossible for these cells to continue to grow and divide.
Although radiation damages both cancer cells and normal cells, most normal cells can recover from the
eects of radiation and function properly. The goal of radiation therapy is to damage as many cancer cells as
possible, while limiting harm to nearby healthy tissue. Hence, it is given in many fractions, allowing healthy
tissue to recover between fractions.
1.3.2.3.2 Chemotherapy

Chemotherapy is the treatment of cancer with drugs ("anticancer drugs") that can destroy cancer cells. In
current usage, the term "chemotherapy" usually refers to cytotoxic drugs which aect rapidly dividing cells
in general, in contrast with targeted therapy (see below). Chemotherapy drugs interfere with cell division
in various possible ways, e.g. with the duplication of DNA or the separation of newly formed chromosomes.
Chemotherapy has the potential to harm healthy tissue, especially those tissues that have a high replacement
rate (e.g. intestinal lining). These cells usually repair themselves after chemotherapy.
Because some drugs work better together than alone, two or more drugs are often given at the same time.
This is called "combination chemotherapy"; most chemotherapy regimens are given in a combination.

Available for free at Connexions <http://cnx.org/content/col11410/1.3>


23

Figure 1.54

Patient undergoing chemotherapy.


http://www.ickr.com/photos/franziska/314275598/sizes/o/in/photostream/
Animation - Overview- text and diagram, last part is video:
http://insidecancer.org/ Select `Hallmarks of Cancer'- `overview'
Animation  Cancer: An introduction to what cancer is and how it is the by-product of broken DNA
replication.
http://www.khanacademy.org/video/cancer?playlist=Biology
1.3.2.3.3 Activity 4 - Cancer in South Africa

Look at the following table


Percentage of cancer deaths by cause in South Africa 2000

Available for free at Connexions <http://cnx.org/content/col11410/1.3>


24 CHAPTER 1. LIFE AT THE MOLECULAR, CELLULAR AND TISSUE
LEVEL

Rank In All People Cause of Death % In All People % In Men % In Women


1 Tracheal/Bronchal/Lung cancer 16.5 21.9 10.9
2 Oesophageal Cancer 13.4 16.7 9.9
3 Cervical Cancer 8.4 17.2
4 Breast Cancer 7.7 0.2 15.6
5 Liver Cancer 6.4 5.4 6.9
6 Colo-Rectal cancer 6.2 5.4 6.9
7 Prostate Cancer 6.1 11.8
8 Stomach Cancer 5.6 6.5 4.7
9 Pancreatic Cancer 3.7 3.7 3.7
10 Leukaemia 3.5 3.8 3.2
Table 1.5

1.Draw a bar graph to show the percentage deaths for each type of cancer for men and women (12 marks)
2.Which type of Cancer is the most common in a) men and b) women. (2 marks)
3.Why are there no percentages given for a) men with cervical cancer and b) women with prostate cancer?
(2 marks)

Available for free at Connexions <http://cnx.org/content/col11410/1.3>


25
1.3.3 1.3.3 Summary48
1.3.3.1 Summary

1.3.3.1.1 Mitosis and the Cell Cycle

The cell cycle is made of stages including G1, S, G2 and Mitosis.


Cell division by mitosis has ve stages: Interphase (which in theory is the phase between divisions and
not part of mitosis), Prophase, Metaphase, Anaphase and Telophase.
Mitosis is important in growth, repair and asexual reproduction.
1.3.3.1.2 Reproduction in simple organisms

Simple organisms such as bacteria and protozoa reproduce asexually using mitosis,
1.3.3.1.3 Cancer

Cancer is due to cells dividing out of control by mitosis.


There are many types of cancer.
Cancer can be treated by chemotherapy and radiotherapy.
Video - Overview: Binary ssion- Mitosis and cell division- Cancer
http://www.neok12.com/php/watch.php?v=zX45724701445b595e766477&t=Cell- Division 49

1.4 Plant and animal tissues


1.4.1 1.4.1 What is a tissue?50
1.4.1.1

Tissue is a level of organization level between cells and a complete organism. A tissue is a group of
51

cells, not necessarily identical, but from the same origin, that together carry out a specic function. These
are called tissues because of they are functioning together. Organs are then formed by the functional
52

grouping together of multiple tissues.


Tissues- http://www.s-cool.co.uk/a-level/biology/cells-and-organelles/revise-it/tissues 53

1.4.2 1.4.2 Plant Tissues54


1.4.2.1

Video  Intro to plant tissues: http://www.youtube.com/watch?v=4V0VcNCRKTo&feature=related 55

Types of plant tissues: http://users.rcn.com/jkimball.ma.ultranet/BiologyPages/P/PlantTissues.html


56

Plant tissues picture: http://www.britannica.com/EBchecked/media/388/Cell-types-and-tissues 57

48 This content is available online at <http://cnx.org/content/m43130/1.1/>.


49 http://www.neok12.com/php/watch.php?v=zX45724701445b595e766477&t=Cell-Division
50 This content is available online at <http://cnx.org/content/m43172/1.1/>.
51 http://en.wikipedia.org/wiki/Cell_(biology)
52 http://en.wikipedia.org/wiki/Organs
53 http://www.s-cool.co.uk/a-level/biology/cells-and-organelles/revise-it/tissues
54 This content is available online at <http://cnx.org/content/m43140/1.1/>.
55 http://www.youtube.com/watch?v=4V0VcNCRKTo&feature=related
56 http://users.rcn.com/jkimball.ma.ultranet/BiologyPages/P/PlantTissues.html
57 http://www.britannica.com/EBchecked/media/388/Cell-types-and-tissues

Available for free at Connexions <http://cnx.org/content/col11410/1.3>


26 CHAPTER 1. LIFE AT THE MOLECULAR, CELLULAR AND TISSUE
LEVEL

Figure 1.55: Dierent types of plant tissue

Note - Protective tissue (shown here) is better described as epidermal tissue


1.4.2.1.1 Meristematic tissue

Meristematic tissue consists of cells dividing by mitosis. This leads to an increase in length and thickness
of the plant. The primary growth of a plant occurs only in certain regions, such as in the tips of stems or
roots. It is in these regions that meristematic tissue is present. Cells in these tissues are roughly spherical
or polyhedral, to rectangular in shape, and have thin cell walls and no vacuoles or plastids. New cells
produced by meristem are initially those of meristem itself, but as the new cells grow and mature, their
58

characteristics slowly change and they become specialised.


1.4.2.1.2 Parenchyma

Parenchyma forms the majority of stems and roots as well as soft fruits like tomatoes and grapes. It consists
of relatively unspecialised cells with thin cell walls. They are living cells. They are usually loosely packed,
so that large spaces between cells (intercellular spaces) are found in this tissue. This tissue provides support
to plants through turgor pressure and also stores food in its cells in the form of sugar and starch. In some
situations, it contains chlorophyll and performs photosynthesis, and then it is called chlorenchyma
1.4.2.1.3 Collenchyma

Collenchyma cells are thin-walled but possess thickening of cellulose and pectin substances at the
59 60

corners where number of cells join together. This tissue gives a tensile strength (because of the thickened
corners) and exibility due to the thinner areas). The cells are compactly arranged and do not have inter-
58 http://en.wikipedia.org/wiki/Meristem
59 http://en.wikipedia.org/wiki/Cellulose
60 http://en.wikipedia.org/wiki/Pectin

Available for free at Connexions <http://cnx.org/content/col11410/1.3>


27
cellular spaces. It occurs chiey in hypodermis of stems and leaves. It is absent in monocots and
61 62

in roots. Collenchyma tissue acts as a supporting tissue in stems of young plants. It provides mechanical
support, elasticity, and tensile strength to the plant body.

Figure 1.56: Cross section of collenchyma cells http://en.wikipedia.org/wiki/File:Plant_cell_type_collenchyma.png


63

1.4.2.1.4 Sclerenchyma

This tissue consists of thick-walled, dead cells. These cells have hard and extremely thick secondary walls
due to uniform distribution of lignin . Lignin deposition is so thick that the cell walls become strong,
64

rigid and impermeable to water. Sclerenchyma cells are closely packed without inter- cellular spaces between
them. Thus, they appear as hexagonal net in transverse section . The cells are cemented together with
65

the help of lamella . The middle lamella is a wall that lies between adjacent cells.
66 67

The main function of sclerenchyma tissues is to give support and protection to the plant. For example
sclerenchyma keeps stems upright and protects nuts in their hard shells.
61 http://en.wikipedia.org/wiki/Hypodermis
62 http://en.wikipedia.org/wiki/Monocots
63 http://en.wikipedia.org/wiki/File:Plant_cell_type_collenchyma.png
64 http://en.wikipedia.org/wiki/Lignin
65 http://en.wikipedia.org/wiki/Transverse_section#Planes
66 http://en.wiktionary.org/wiki/lamella
67 http://en.wikipedia.org/wiki/Middle_lamella

Available for free at Connexions <http://cnx.org/content/col11410/1.3>


28 CHAPTER 1. LIFE AT THE MOLECULAR, CELLULAR AND TISSUE
LEVEL

Figure 1.57: Cross section of sclerenchyma bers. http://en.wikipedia.org/wiki/File:Plant_cell_type_sclerenchyma_bers.png


68

Note that some of the cells still have their cytoplasm contents and may not have died yet.
1.4.2.1.5 Epidermis

A single-layer of cells that covers plants' leaves , owers , roots and stems . It forms a boundary
69 70 71

between the plant and the external environment. The epidermis serves several functions, it protects
72

against water loss, regulates gas exchange, secretes metabolic compounds, and (especially in roots) absorbs
water and mineral nutrients.
68 http://en.wikipedia.org/wiki/File:Plant_cell_type_sclerenchyma_bers.png
69 http://en.wikipedia.org/wiki/Leaf
70 http://en.wikipedia.org/wiki/Flowers
71 http://en.wikipedia.org/wiki/Plant_stem
72 http://en.wikipedia.org/wiki/Plant

Available for free at Connexions <http://cnx.org/content/col11410/1.3>


29

Figure 1.58: Scanning electron microscope image of Nicotiana alata upper leaf surface, showing tricomes
73
and a few stomates. http://en.wikipedia.org/wiki/File:Leaf_epidermis_w_scale.jpg

1.4.2.1.6 Xylem

Xylem tissue has the dual function of supporting the plant and transporting water and dissolved mineral
salts. It is made up of vessel elements, tracheids, bres and parenchyma cells. These cells are joined end
to end to form long tubes. Vessels and tracheids are dead at maturity. Tracheids have thick secondary cell
walls and are tapered at the ends. The thick walls provide support. They do not have end openings like the
vessels do. The tracheids ends overlap with each other, with pairs of pits present. The pit pairs allow water
to pass horizontally from cell to cell.
1.4.2.1.7 Phloem

Phloem tissue is responsible for translocation which is the transport of soluble organic substances - for
example, sugar. The substances travel along sieve elements but other types of cells are also present, the
companion cells, parenchyma cells and bres. Phloem is living tissue. The end walls, unlike vessel members
in xylem, do not have large openings. The end walls, however, are full of small pores where cytoplasm
extends from cell to cell. These porous connections are called sieve plates. In spite of the fact that their
cytoplasm is actively involved in the conduction of food materials, sieve-tube members do not have nuclei
at maturity. The activity of the sieve tubes is controlled by companion cells through plasmadesmata.
73 http://en.wikipedia.org/wiki/File:Leaf_epidermis_w_scale.jpg

Available for free at Connexions <http://cnx.org/content/col11410/1.3>


30 CHAPTER 1. LIFE AT THE MOLECULAR, CELLULAR AND TISSUE
LEVEL

74
Figure 1.59: Phloem and xylem http://mrb-science.wikispaces.com/Roots+and+Stems

1.4.2.1.8 Investigation 1 - Observing Xylem Tissue

Materials:
•2 soft young stems of a plant such as Impatiens, Aster or Lily  with leaves and a ower
•2 containers
•Water
•Blue / red food colouring
•A scalpel
•A hand lens
•A glass slide
•A cover slip
•A microscope
Method:
1.Fill the containers with water.
2.Put some blue food colouring into one container and red colouring into the other container.
3.Cut the bottom end of the owers' stems diagonally and put one stem into each container.
4.Leave overnight (can leave longer)
5.Cut the stem again near the cut end and use a hand lens to observe the cut surface.
a.Do you notice any of the colouring/stain inside the plant?
b.If so, where? Draw a diagram to show what you see.
74 http://mrb-science.wikispaces.com/Roots+and+Stems

Available for free at Connexions <http://cnx.org/content/col11410/1.3>


31
6.Use the scalpel to cut a few THIN cross sections of the stem.
7.Make a fresh wet mount slide of this section.
8.Observe the section under the microscope and draw a labelled diagram of what YOU SEE!

Figure 1.60: Dierences between monocotyledons and dicotyledons

1.4.3 1.4.3 Animal Tissues75


1.4.3.1 Animal Tissues

There are several main types of animal tissue including : epithelial, muscle, nervous tissue and connective
tissues such as bone, cartilage, blood and loose connective tissue (sometimes called areolar tissue).
Video - Animal tissues introduction: http://www.youtube.com/watch?v=I2bSWCyKOz0&feature=related
76

Tissues of human body: http://www.mhhe.com/biosci/ap/histology_mh/tismodov.html 77

75 This content is available online at <http://cnx.org/content/m43137/1.1/>.


76 http://www.youtube.com/watch?v=I2bSWCyKOz0&feature=related
77 http://www.mhhe.com/biosci/ap/histology_mh/tismodov.html

Available for free at Connexions <http://cnx.org/content/col11410/1.3>


32 CHAPTER 1. LIFE AT THE MOLECULAR, CELLULAR AND TISSUE
LEVEL

Figure 1.61: Mammalian tissues

Available for free at Connexions <http://cnx.org/content/col11410/1.3>


33
1.4.3.1.1 Epithelial tissue

Figure 1.62: Four types of epithelial tissue (Ciliated epithelium is another specialized type of epithelial
78
tissue.) http://www.hartnell.edu/tutorials/biology/tissues.html

Epithelial tissues are formed by cells that cover surfaces such as the surface of the skin, and line tubes and
cavities, such as the digestive organs, blood vessels, kidney tubules and airways. The cells comprising an
epithelial layer are linked via semi-permeable junctions; hence, this tissue provides a barrier between the
external environment and the organ it covers. In addition to this protective function, epithelial tissue may
also be specialized to function in secretion and absorption . Epithelial tissue helps to protect organisms
79 80

from microorganisms, injury, and uid loss. All epithelial tissues are free surfaces attached to the underlying
layers by a basement membrane.
78 http://www.hartnell.edu/tutorials/biology/tissues.html
79 http://en.wikipedia.org/wiki/Secretion
80 http://en.wikipedia.org/wiki/Digestion

Available for free at Connexions <http://cnx.org/content/col11410/1.3>


34 CHAPTER 1. LIFE AT THE MOLECULAR, CELLULAR AND TISSUE
LEVEL
1.4.3.1.2 Muscle tissue

81
Figure 1.63: 3 types of muscle http://www.hartnell.edu/tutorials/biology/tissues.html

Muscle cells form the active contracting tissue of the body known as muscle tissue. Muscle tissue functions
to produce force and cause motion within internal organs. Muscle tissue is separated into three distinct
categories: smooth muscle , which is found in the inner linings of organs; skeletal muscle , which is
82 83

found attached to bone providing movement; and cardiac muscle which is found in the heart , allowing
84 85

it to contract and pump blood throughout an organism.


1.4.3.1.2.1 Smooth muscle

Smooth muscle is unstriated (not striped) in appearance. The contraction of smooth muscle can be relatively
slow and often happens automatically without our conscious control. Some call it involuntary muscle. It
rarely becomes fatigued. Smooth muscle is found in the gut where it squeezes food along the intestines by
peristalsis. It is found in the walls of our blood vessels where it can make the vessel wider or narrower
allowing more or less blood to ow.
1.4.3.1.2.2 Skeletal muscle

Skeletal muscle or voluntary muscle appears striped or striated when seen through a microscope. This is
due to banding from the pattern of actin and myosin protein laments in the muscle. Skeletal muscle is
attached to the bones of the skeleton. Skeletal muscle fatigues or tires quickly. It also contracts quickly and
is controlled by the conscious part of our brain. The biceps is made of skeletal muscle and when the muscle
contracts it shortens bringing the lower arm upwards. Skeletal muscles are often arranged in antagonistic
pairs where if one muscle moves a limb one way the other muscle will move it back.
1.4.3.1.2.3 Cardiac Muscle

Cardiac muscle appears striated like skeletal muscle except in cardiac muscle there are cross bridges or cross
connections linking muscle bres together. Cardiac muscle is myogenic, in other words it generates its own
impulse to contract from within itself. It is found only in the heart and it contracts rhythmically at a speed
dictated by the brain without ever suering fatigue.
81 http://www.hartnell.edu/tutorials/biology/tissues.html
82 http://en.wikipedia.org/wiki/Smooth_muscle
83 http://en.wikipedia.org/wiki/Skeletal_muscle
84 http://en.wikipedia.org/wiki/Cardiac_muscle
85 http://en.wikipedia.org/wiki/Heart

Available for free at Connexions <http://cnx.org/content/col11410/1.3>


35
1.4.3.1.3 Nerve tissue

Figure 1.64

Cells making up the central nervous system and peripheral nervous system are classied as neural
86 87

tissue. In the central nervous system , neural tissue forms the brain and spinal cord and, in the
88 89 90

peripheral nervous system forms the cranial nerves and spinal nerves , including the sensory and
91 92 93

motor neurons . The function of nerve tissue is to transmit electrical messages around the body.
94

86 http://en.wikipedia.org/wiki/Central_nervous_system
87 http://en.wikipedia.org/wiki/Peripheral_nervous_system
88 http://en.wikipedia.org/wiki/Central_nervous_system
89 http://en.wikipedia.org/wiki/Brain
90 http://en.wikipedia.org/wiki/Spinal_cord
91 http://en.wikipedia.org/wiki/Peripheral_nervous_system
92 http://en.wikipedia.org/wiki/Cranial_nerves
93 http://en.wikipedia.org/wiki/Spinal_nerves
94 http://en.wikipedia.org/wiki/Motor_neurons

Available for free at Connexions <http://cnx.org/content/col11410/1.3>


36 CHAPTER 1. LIFE AT THE MOLECULAR, CELLULAR AND TISSUE
LEVEL
1.4.3.1.4 Three Types of Nerve Cells

1.4.3.1.4.1 Sensory neurones

Figure 1.65

Sensory nerve cells (or sensory neurones) carry impulses (electrical signals) from a receptor to the CNS
(central nervous system). The cell body is located o to one side of the axon as it enters a vertebra (one of
the boney discs of the spine).
1.4.3.1.4.2 Motor neurones

1.4.3.1.4.3

Motor neurones carry impulses out from the CNS to eectors, instructing them to do something. In most
cases the eector is a muscle being told to contract but other eectors could be glands or colour cells
(chromatophores). The cell body is at the head of a motor neurone in the vertebra and its tail or dendrites
are attached to the muscle or other eector.
1.4.3.1.4.4 Relay or connector neurones

Relay neurones are very short especially compared to the other two types. They connect a sensory neurone
with a motor neurone across the grey matter region in the CNS inside the spine. The impulse travels from
the cell body at the head end along the short axon to the dendrites.

Available for free at Connexions <http://cnx.org/content/col11410/1.3>


37
1.4.3.1.5 Connective tissue

Figure 1.66: Dierent types of connective tissue http://www.hartnell.edu/tutorials/biology/tissues.html


95

Connective tissues are made up of separate cells which are oating in a matrix. Connective tissues are brous
tissues. They are made up of cells separated by non-living material, which is called matrix . Connective
96

tissues give shape to organs and holds them in place. Both bone and blood are examples of connective tissue.
As the name implies, connective tissue serves a "connecting" function. It supports and binds other tissues.
95 http://www.hartnell.edu/tutorials/biology/tissues.html
96 http://en.wikipedia.org/wiki/Extracellular_matrix

Available for free at Connexions <http://cnx.org/content/col11410/1.3>


38 CHAPTER 1. LIFE AT THE MOLECULAR, CELLULAR AND TISSUE
LEVEL
1.4.3.1.5.1 Cartilage

Cartilage is a exible connective tissue found in many areas of the bodies of humans and other animals,
97

including the joints between bones , the rib cage , the ear , the nose , the elbow, the knee,
98 99 100 101

the ankle, the bronchial tubes and the intervertebral discs


102
. It is not as hard and rigid as bone
103

104
but is stier and less exible than muscle . 105

Figure 1.67

97 http://en.wikipedia.org/wiki/Connective_tissue
98 http://en.wikipedia.org/wiki/Bone
99 http://en.wikipedia.org/wiki/Rib_cage
100 http://en.wikipedia.org/wiki/Ear
101 http://en.wikipedia.org/wiki/Nose
102 http://en.wikipedia.org/wiki/Bronchial_tubes
103 http://en.wikipedia.org/wiki/Intervertebral_disc
104 http://en.wikipedia.org/wiki/Bone
105 http://en.wikipedia.org/wiki/Muscle

Available for free at Connexions <http://cnx.org/content/col11410/1.3>


39
Cartilage is composed of specialized cells called chondroblasts that produce a large amount of
106

extracellular matrix composed of collagen bres, abundant ground substance and elastin
107 108 109 110

bers. Chondroblasts that get caught in the matrix are called chondrocytes . They lie in spaces, called
111

lacunae, with up to eight chondrocytes per lacuna. Cartilage is classied in three types, elastic cartilage
112
, hyaline cartilage and brocartilage , which dier in the relative amounts of these three main
113 114

components.
1.4.3.1.6 Blood

Blood is a tissue because it is made up of several types of cell functioning together. In fact blood is composed
of red blood cells, white blood cells and platelets oating in a yellow liquid called plasma.

Figure 1.68

106 http://en.wikipedia.org/wiki/Chondroblast
107 http://en.wikipedia.org/wiki/Extracellular_matrix
108 http://en.wikipedia.org/wiki/Collagen
109 http://en.wikipedia.org/wiki/Ground_substance
110 http://en.wikipedia.org/wiki/Elastin
111 http://en.wikipedia.org/wiki/Chondrocyte
112 http://en.wikipedia.org/wiki/Elastic_cartilage
113 http://en.wikipedia.org/wiki/Hyaline_cartilage
114 http://en.wikipedia.org/wiki/Fibrocartilage

Available for free at Connexions <http://cnx.org/content/col11410/1.3>


40 CHAPTER 1. LIFE AT THE MOLECULAR, CELLULAR AND TISSUE
LEVEL
1.4.3.1.6.1 Red blood cells

Red blood cells are shaped like biconcave discs (round discs with a dimple in the top and another in the
bottom). Their job is to carry oxygen round the body form the lungs to the tissues. They contain a red
pigment (coloured chemical) called haemoglobin which is brilliant at carrying oxgen.
1.4.3.1.6.2 White blood cells

White blood cells are slightly larger than red ones. They are more irregular in shape and their job is to ght
pathogens (disease-causing organisms like some bacteria and viruses). Some engulf 9swallow up pathogens),
some produce chemicals called antibodies to kill the pathogens while a third type release antitoxins, chemicals
which neutralize the poisons made by the pathogens. When looked at under a microscope white blood cells
may appear purple because a dye is used to stain them.
1.4.3.1.6.3 Platelets

Platelets are fragments of cells oating in the plasma. They are important in clotting and stick together
where a blood vessel is damaged to close the wound.
1.4.3.1.6.4 Plasma

Plasma is a yellow liquid in which the solid cells are suspended or oating. It carries many important
chemicals around the body including the waste carbon dioxide from respiration, hormones, urea, and glucose
and also transports heat.
1.4.3.2 Structure and function

Table to show the relationship between structure and function in 4 animal tissues.

Tissue Example Structure Function Relationship


Epithelial tissue Lining of the air Thin, sometimes Diusion of gases Gases diuse best
sacs in the lungs one cell thick, across lining across thin, moist
often moist surafces
Connective tissue Bone Composed of min- Strength and exi- Strength is pro-
erals and organic bility vided by minerals,
matter exibility by or-
ganic matter
Muscle tissue Heart muscle Contains bres Pumping by con- Fibres ratchet past
traction each other to con-
tract the muscle
Nerve tissue Optic nerve Long thin, insu- Carrying electrical Long and insu-
lated bres messages from eye lated so electrical
to brain message is carried
quickly
Table 1.6

Available for free at Connexions <http://cnx.org/content/col11410/1.3>


41
1.4.3.3 Assignment 1  Mammal tissues

MAMMALIAN TISSUES
Exercise 1.4.3.1
Question 1
1.1 Answer the following questions based on the drawings below.

Figure 1.69

1.1 Provide labels for 1, 2, 3 & 4.(4)


1.2 Which tissue (A or B) is found in the rib cage?(1)
1.3 Which tissue (A or B) is found in the lining of the blood vessels?(1)
Question 2
The following diagram shows a blood smear.
W

Available for free at Connexions <http://cnx.org/content/col11410/1.3>


42 CHAPTER 1. LIFE AT THE MOLECULAR, CELLULAR AND TISSUE
LEVEL

Figure 1.70

rite down the number of the part that . . .


2.1 contains haemoglobin
2.2 is needed for the clotting of blood
2.3 are phagocytic (engulf germs)
2.4 transport nutrients
2.5 transport oxygen
2.6 distributes heat in the body[6]
Question 3
A scientist knows that as you move to higher altitudes (height above sea level), the air has less
oxygen. She therefore wanted to investigate whether altitude inuences the number of red blood
corpuscles in people's blood.
3.1 Write a hypothesis for the scientist's investigation. (2)
3.2 Identify the:
dependent variable (the variable to be tested)
independent variable. (2)
3.3 The scientist drew the following graph after she had nished her investigation. Study it and
answer the questions based on it.

Available for free at Connexions <http://cnx.org/content/col11410/1.3>


43

Figure 1.71

3.4 What is the relationship between the number of red blood corpuscles and altitude? (2)
3.5 Explain the reason for this relationship. (1)
3.6 The Sharks Rugby Team (a rugby team from Durban) often get tired in the last twenty
minutes when the play the Blue Bulls (a team from Pretoria) in Gauteng.
Explain why, in terms of the information from the graph. (3)
3.7 Suggest how the Sharks could overcome this problem as described in the above question.
(2)
3.8 The scientist lost the table from which she drew the graph above, Draw the table of gures
and values to represent the graph above. (6)
3.6 Do you think that the scientist took just one person's blood sample at altitude? Explain
your answer. (2)
[20]
Assignment 2 - Practical and theory Test: Plant Tissues and Photosynthesis
Time: 90 Minutes (+10 minutes reading time)Marks: 70
Instructions:
Section A: practical work and answers should be given on the question paper
Section B: theory and answers should be given on your own lined paper.
Exercise 1.4.3.2
Section A
Aim: Testing for the presence of starch
Question 1
Apparatus
You are provided with the following equipment:
a spotting tile
marking pen
solutions A (starch solution) and B (sugar solution)
Available for free at Connexions <http://cnx.org/content/col11410/1.3>
44 CHAPTER 1. LIFE AT THE MOLECULAR, CELLULAR AND TISSUE
LEVEL
leaf discs marked C and D
Iodine reagent
Droppers
Forceps
The leaf discs have been prepared for testing for the presence of starch .
Describe the procedure ( only ) followed in this preparation. Outline 5 steps.(5)
Methods
Clearly mark your spotting tile A-D using the marker provided.
Using the droppers (in the beakers) place a few drops of the solution A in well A and a few
drops of solution B in well B and the leaf discs (using the forceps) in C and in D in each of
the marked areas.
Call your teacher to observe you apply iodine solution to each area A-D. Wait a
few moments and then make observations. (5)
Draw up a table in the space below to ll in your
observations and conclusions .(10)
Clean up your area by returning all apparatus to the trolley and wiping o the
spotting tile with the cloth provided.
Question 2
Water plants are exposed to light for one minute at a time at dierent temperatures. The
amount of oxygen given o per minute at dierent temperatures is shown graphically in Figure 1.
Figure 1: Graph showing the oxygen given o by a plant placed in light at dierent temperatures

Figure 1.72

2.1 Which one of the following statements is the best summary of the results? Highlight your
choice.(2)
i. Oxygen is formed in plants exposed to light.
ii. A rise in temperature increases the production of oxygen.
iii. The oxygen production is highest between 20ºC and 30ºC.
iv. Temperature aects the rate of oxygen production.
v. A rise in temperature increases the oxygen production to a certain maximum.
2.2 In the experiment what is the:
Independent variable? (1)
Dependent variable? (1)
2.3 Design an experiment to show how you obtain the results shown in the graph. You need
only outline the method.(10)
Section B
Question 3
Choose TWO of any of the following questions, number your choices very clearly:
3.1 The strengthening substances, cellulose and lignin are found in various plant tissues. Name
any four of the plant tissue types and explain the specic role in each tissue mentioned.(8)

Available for free at Connexions <http://cnx.org/content/col11410/1.3>


45
3.2 Compare the structure of a guard cell and an ordinary epidermal cell.(8)
3.3 Parenchyma is termed packing tissue/storage/gaseous exchange tissue. Discuss four fea-
tures of this tissue that enable it to fulll this role.(8)
3.4 Soluble substances need to be transported around the plant body. Highlight 4 features of
phloem tissue that allow for ecient transport.(8)
[16]
Question 4: Mini-Essay
Photosynthesis and food production
It is very important to understand the factors that aect the rate of photosynthesis when growing
crops that provide us with food. The greater the rate of photosynthesis of a plant, the greater its
rate of productivity. The productivity of a plant refers to how much extra dry mass it gains over a
period of time. This extra dry mass is in the form of carbohydrates, proteins or fats that the plant
makes from the products of photosynthesis. A plant crop that is very productive will gain more
dry mass over a period of time, and will therefore provide us with more food to eat. Farmers will
also get paid more if they produce a greater quantity of food. Plants, such as tomatoes, grown in
greenhouses (or growing tunnels) may grow better than those grown outside.
You need to write a motivation to a farmer where you convince him that the cost of a greenhouse
(plant growing tunnel) will be recouped /earned with the greater prot that he will earn from his
crops.
In your answer you need to consider:
At least 4 structural features of the greenhouse that will enable the plant to grow better,
while at the same time considering the factors necessary for a plant to photosynthesis most
eectively.
[20]
Marking Rubric:

Assessment Crite-
rion
Content 0-4Little reference 5-9Some green- 10-12Allgreenhouse features mentioned,
to green house fea- house features and at least 4 factors necessary for pho-
tures, minimal fac- mentioned, a few tosynthesis mentioned
tors mentioned factors mentioned
Skills 0-2Very little accu- 3-4Some confu- 5Coherent, linked concepts, logical path-
racy, much irrele- sion, some linkage, way correct
vant data some irrelevance
Presentation 1 no paragraphs, 2Paragraphed but 3Good intro and conclusion, para-
intro or conclu- poor intro and graphed; good biological expression
sion; poor expres- conclusion
sion
Table 1.7

1.4.4 1.4.4 Applications of indigenous knowledge and biotechnology115


1.4.4.1 Traditional and modern biotechnology

Traditional Biotechnology based on indigenous methods of fermentations is becoming a source of increasing


interest in the scientic community. Below is an example of the kind of interest being shown in traditional
methods of biotechnology.
115 This content is available online at <http://cnx.org/content/m43060/1.1/>.

Available for free at Connexions <http://cnx.org/content/col11410/1.3>


46 CHAPTER 1. LIFE AT THE MOLECULAR, CELLULAR AND TISSUE
LEVEL

Figure 1.73

The above is the abstract from a scientic paper. The abstract provides a summary of the contents of
the paper at the start of the published paper.
1.4.4.2 Medical biotechnology

In medicine modern biotechnology nds promising applications in such areas as: drug production, phar-
macogenomics (how a person's genes aects their response to drugs), gene therapy and genetic testing (or
genetic screening): techniques in molecular biology detect genetic diseases. To test the developing fetus for
Down syndrome, Amniocentesis and chorionic villus sampling can be used.[2]
1.4.4.3 Cloning

Cloning is the process of articially producing genetically identical individuals that occurs in nature when
organisms such as bacteria, insects or plants reproduce asexually. Cloning in biotechnology refers to processes
used to create copies of DNA fragments (molecular cloning), cells (cell cloning), or organisms.

Available for free at Connexions <http://cnx.org/content/col11410/1.3>


47
1.4.4.3.1 Dolly the sheep

Figure 1.74

Dolly was born 5 July 1996 to three mothers (one provided the egg, another the DNA and a third carried
the cloned embryo to term). She was created using the technique of somatic cell nuclear transfer, where the
cell nucleus from an adult cell is transferred into an unfertilised oocyte (developing egg cell) that has had its
nucleus removed. The hybrid cell is then stimulated to divide by an electric shock, and when it develops into
a blastocyst it is implanted in a surrogate mother. Dolly was the rst clone produced from a cell taken from
an adult mammal. The production of Dolly showed that genes in the nucleus of such a mature dierentiated
somatic cell are still capable of reverting back to an embryonic totipotent state, creating a cell that can then
go on to develop into any part of an animal. Dolly's existence was announced to the public on 22 February
1997.
1.4.4.3.1.1 Activities and Links on Biotechnology

•http://www.pbs.org/wgbh/harvest/engineer/transgen.html
Takes students on a step-by-step process showing them how to genetically engineer a plant and bacterium
•http://www.biotechnologyonline.gov.au/biotechnologyonline/topitems/resour ces.html
Australian government resource for educators and learners on various aspects of biotechnology.
Available for free at Connexions <http://cnx.org/content/col11410/1.3>
48 CHAPTER 1. LIFE AT THE MOLECULAR, CELLULAR AND TISSUE
LEVEL
•http://www.scienticamerican.com/biotechnology
All the latest articles on developments in biotechnology written in accessible language.
•http://seedmagazine.com/content/tag/biotechnology/
Discusses latest issues in biotechnology and their political, economic and cultural implications.
1.4.5 1.4.5 Organs116
1.4.5.1 Organs

1.4.5.1.1 Organs and tissues

An organ is a collection of tissues joined in structural unit to serve a common function. Functionally
117

related organs often cooperate to form whole organ systems . Organs exist in all higher biological organisms,
they are not restricted to animals, but can also be identied in plants, e.g. the leaf is an organ in a plant as
is the root, stem, owers and fruits.

Figure 1.75

116 This content is available online at <http://cnx.org/content/m43162/1.1/>.


117 http://en.wikipedia.org/wiki/Tissue_(biology)

Available for free at Connexions <http://cnx.org/content/col11410/1.3>


49
Cabbage by Christian Guthier
118
http://www.fotopedia.com/items/ickr-2671495796

1.4.5.1.2 Leaf Structure

The leaf is the main photosynthesising organ of a plant. The Leaf is an organ because it contains a number
of tissues including epidermis, xylem, phloem and parenchyma working together to make food for the plant
in photosynthesis.

Figure 1.76: Diagram of a section through a leaf

1. Upper epidermis
2. Palisade
3. Air space
4. Mesophyll
5. Xylem
6. Phloem
7. Stoma

1.4.5.1.2.1 Leaf Adaptations

The upper epidermis is transparent to allow light to pass through to the chlorophyll-containing cells deeper
in the leaf.
The palisade cells are tightly packed just underneath the upper epidermis. They are full of chloroplasts
for photosynthesis.
Intercellular spaces allow for diusion and gaseous exchange.
Spongy mesophyll contains chloroplasts for photosynthesis and air spaces to aid diusion of gases.
Xylem transports water to the cells, as water is a requirement of photosynthesis.
Phloem transports the products of photosynthesis (starch etc.) away from the leaves.
Stomata (controlled by guard cells) allow carbon dioxide to enter and oxygen to leave by diusion for
photosynthesis.
Video  (free, to be embedded) Plant Structure: http://www.neok12.com/php/watch.php?v=zX64544265547901594f5567&
119

118 http://www.fotopedia.com/items/ickr-2671495796
119 http://www.neok12.com/php/watch.php?v=zX64544265547901594f5567&t=Plants

Available for free at Connexions <http://cnx.org/content/col11410/1.3>


50 CHAPTER 1. LIFE AT THE MOLECULAR, CELLULAR AND TISSUE
LEVEL
Video  (free, to be embedded) Photosynthesis http://www.neok12.com/php/watch.php?v=zX4b7a504d047d5b6e660173&
120

1.4.5.1.3 Assignment 3 - Structure of a dicot leaf

A Label parts 1-7(7)


B Write down the function and one adaptation of each part. (14)

Figure 1.77

120 http://www.neok12.com/php/watch.php?v=zX4b7a504d047d5b6e660173&t=Plants

Available for free at Connexions <http://cnx.org/content/col11410/1.3>


51
1.4.6 1.4.6 Summary121
1.4.6.1 Summary

A tissue is a group of cells grouped together and working together.


An organ is group of tissues grouped together and working together.
Plant tissues include xylem, phloem, collenchyma, sclerenchyma, epidermis and meristematic tissue.
Animal tissues include epithelial tissue, connective tissue, muscle tissue and nerve tissue.
Plant and animal tissues are used in traditional technology, medical technology and cloning processes.
The leaf is an example of a plant organ and its structure is adapted to its role in photosynthesis.

121 This content is available online at <http://cnx.org/content/m43064/1.1/>.

Available for free at Connexions <http://cnx.org/content/col11410/1.3>


52 CHAPTER 1. LIFE AT THE MOLECULAR, CELLULAR AND TISSUE
LEVEL

Available for free at Connexions <http://cnx.org/content/col11410/1.3>


Chapter 2

Life processes in plants and animals

2.1 Support and transport systems in plants


2.1.1 2.1.1 - Anatomy of dicotyledenous plants1
2.1.1.1 Anatomy of Dicotyledonous Plants

Plant structure:
http://www.emc.maricopa.edu/faculty/farabee/biobk/biobookplantanat.html
Plants are made up of roots, stems, leaves and owers. The function of the root is to hold the plant
rmly in the ground as well as to absorb water from the soil. The function of the stem is to transport the
food made by the leaf to the rest of the plant as well as to hold the plant upright. The main function of the
leaves is to photosynthesize (make food).
1 This content is available online at <http://cnx.org/content/m43142/1.1/>.

Available for free at Connexions <http://cnx.org/content/col11410/1.3>

53
54 CHAPTER 2. LIFE PROCESSES IN PLANTS AND ANIMALS

Figure 2.1

2.1.1.1.1 Dierences between monocotyledonous and dicotyledonous plants

Traditionally the owering plants (angiosperms) are divided into two groups, monocotyledons (monocots)
and dicotyledons (dicots). Monocots are the grass and grass-like owering plants (e.g. maize), while dicots
include the rest of the owering plants (e.g. bean). The embryos of monocots have only a single (mono-)
cotyledon (the rst leaf) while the embryos of dicots have paired (di-) cotyledons. Other dierences between
monocots and dicots are shown in the table below. Monocots have long narrow leaves with parallel veins
while dicots have broad leaves with net-like veins. In monocots the ower parts are in multiples of three while
in dicots they are in multiples of four or ve. In monocots the vascular bundles of the stem are scattered
while in dicots there is a ring of vascular bundles. Monocots grains have one furrow or pore while dicot
grains have three furrows or pores. Monocots have adventitious roots while in dicots the roots develop from
a radicle.

Available for free at Connexions <http://cnx.org/content/col11410/1.3>


55

Figure 2.2

2.1.1.1.2 Plant structure

Most plants are stationary which means that they cannot move from place to place. Some plants grow
really tall in order to obtain sunlight. They need to stand tall and erect and therefore need to support
themselves. They have tissues present in almost all parts of their body e. g. roots, stems, branches, leaves.
These supporting tissues keep the stem rm and other parts such as leaves in a favourable position for
photosynthesis to occur as eciently as possible.
Refer to Unit 1.4 for functions of the dierent tissues found in roots, stems and leaves.
2.1.1.1.3 Dicotyledonous Root

External structure of the dicot root

Available for free at Connexions <http://cnx.org/content/col11410/1.3>


56 CHAPTER 2. LIFE PROCESSES IN PLANTS AND ANIMALS

Figure 2.3

This diagram shows the external structure of a dicot root.


• Root cap protects the tip of the root and it is slimy to facilitate movement through the soil as the root
grows.
• Above the root cap is the meristematic region where cells divide continuously by mitosis to produce
new cells.
• Cells enlarge in size in the region of elongation. This results in the root growing in length.
• Thousands of tiny root hairs are found in the root hair region. The function of this region is to absorb
water and dissolved mineral salts from the soil.
• The root grows thicker and may produce lateral roots in the mature region.

Available for free at Connexions <http://cnx.org/content/col11410/1.3>


57
2.1.1.1.3.1 Internal structure of the dicot root

Figure 2.4

• No waterproof cuticle in the root as this would hinder the absorption of water.
• The epidermis is a single layer of cells on the outside that protects the inner tissues. Some epidermal
cells are specialized to form root hair cells. These absorb water and dissolved mineral salts.
• The cortex consists of parenchyma cells. These cells are large to store water and food. They also
facilitate the movement of water from the root hair cells on the outside to the xylem on the inside.
• The endodermis is lined with Casparian strips, distinctive bands made of a water-impermeable, waxy
substance called suberin, that prevents water and minerals from passively seeping between the cells
and thus forces water to enter through the cell membranes of the endodermal cells in order to enter
the stele (vascular cylinder).
• The stele consists of the:
· Pericycle (responsible for forming lateral roots)
· Xylem (responsible for transporting water and mineral salts to the stem)
· Phloem (responsible for transporting food from the leaves to the roots)

Available for free at Connexions <http://cnx.org/content/col11410/1.3>


58 CHAPTER 2. LIFE PROCESSES IN PLANTS AND ANIMALS

2.1.1.1.3.2 Movement of water through the dicotyledonous root

Figure 2.5

This diagram shows the movement of water through the root


• Water is found in the spaces between the soil particles. Water enters through the cell wall and cell
membrane of the root hair cell by osmosis. Water lls the vacuole of the root hair cell.
• Water can now move across the parenchyma cells of the cortex in two ways:
• Most of the water passes along the cell walls of the parenchyma cells by diusion. Movement of
water and solutes between the intercellular spaces without crossing the plasma membrane is known as
apoplastic movement.
• Some of the water passes from the vacuole of one parenchyma cell to the vacuole of the next cell by
osmosis. Movement of water and solutes through the cells is known as symplastic movement.
• The water must pass through the endodermis to enter the xylem.
• Once water is in the xylem of the root, it will pass up the xylem of the stem.

Transpiration and movement of water: http://www.phschool.com/science/biology_place/l abbench


/lab9/xylem.html
This website shows a diagram of how water moves up through the plant.
http://www.neok12.com/Plants.htm
This video shows plant transport and provides some interactive quiz games.
2.1.1.1.3.3 Investigation: Water uptake by roots

Aim : To measure the uptake of water by roots


Apparatus

• Plastic 2 litre Coke bottle


• water
• soil
• scissors
• measuring scale
• tree or plant cuttings
Available for free at Connexions <http://cnx.org/content/col11410/1.3>
59
• ruler
Method

1. Remove the label from the Coke bottle. Cut the top of the bottle o 20cm from the bottom of the
bottle. Poke holes in the bottom of the bottle for drainage. Hold the plant cutting in the container
while you ll the container with soil. Leave one or two leaf buds about 5cm above the soil.
2. Weigh the container to get the total weight of the bottle, soil and plant.
3. Water the plant with enough water so that it starts to run out of the bottom of the bottle through the
holes.
4. Weigh the container again after the water has stopped running out and subtract the total weight in
step 2 from the weight in step 4 to get the weight of the water. 1 litre of water is equal to 1 kilogram
of water, therefore you can work out exactly how much water is in the container.
5. Set the containers by the window where they will receive enough sunlight. Wait for the leaves to start
growing (1-3 weeks).
6. After the leaves are growing, weigh the containers every 1-3 days for 3 weeks. Subtract the new weight
from the weight calculated in step 4. The new number is the amount of water that the plant is using.
Water the plant as necessary (when the soil becomes dry), but remember to reweigh the container
when you add more water so that you can still tell how much water the plant is taking up.
7. Draw a graph of the amount of water the plant is using. The X-axis should show the number of days
since the beginning of the experiment and the Y-axis should show the amount of water that the plant
has used.
Question on investigation  Surely as the plant grows it forms new leaves etc which have mass? How do we
subtract the mass of the new leaves to prevent
this making the mass of the entire container inaccurate in terms of how much mass is
just water usage? - I also wondered this but obtained the investigation elsewhere so don' t know. I
thought perhaps the plant does not put on that much mass compared to the mass of the water so it becomes
insignicant
2.1.1.1.4 Dicotyledonous stem

• Leaves develop from the nodes.


• The sections of stem between the nodes are called internodes.
• An axillary bud is often found at the node. These forms lateral branches.
• A terminal bud is found at the tip of the stem and allows the stem to increase in length.

Available for free at Connexions <http://cnx.org/content/col11410/1.3>


60 CHAPTER 2. LIFE PROCESSES IN PLANTS AND ANIMALS

Figure 2.6

Available for free at Connexions <http://cnx.org/content/col11410/1.3>


61
2.1.1.1.4.1 Internal structure of the dicotyledonous stem

Figure 2.7

This diagram of a cross section shows the internal structure of a young dicot stem
• A waterproof cuticle is found on the outside of the epidermis to prevent water loss.
• The epidermis consists of a single layer of cells to protect the underlying tissue.
• The cortex is made up of parenchyma cells that stores water and food.
• The vascular bundles are arranged in a ring in the medulla and are surrounded by non-living scle-
renchyma cells for strengthening and support.
• Each vascular bundle contains the following:
· Cambium (contains meristematic cells that divide to widen the stem)
· Phloem (transports food from leaves to the roots)
· Xylem (transports water from the roots to the stem)

http://bcs.whfreeman.com/thelifewire/content/ch p36/36020.html
This is a link to an online tutorial about phloem, xylem and pressure ow.
2.1.1.1.4.2 Movement of water up the stem

• Water moves up the xylem from the roots to the leaves.


• Adaptations of xylem for transporting water:
· Long, elongated tubes joined end-to-end without any cross-walls, forming good conducting tubes.
· The cell walls are thickened with lignin for support (annual or spiral thickening) so that they do
not collapse due to the upward pull of water
· Pitted vessels and tracheids allow for lateral movement of water into neighbouring xylem vessels.
· Cells are dead, so there is no obstruction to water transport

Available for free at Connexions <http://cnx.org/content/col11410/1.3>


62 CHAPTER 2. LIFE PROCESSES IN PLANTS AND ANIMALS

Figure 2.8

Diagram of xylem
Three forces are responsible for the movement of water up the xylem  capillarity, root pressure and
transpiration suction force.
• Capillarity involves forces of cohesion (forces of attraction between water molecules) and adhesion
(forces of attraction between water molecules and the sides of the xylem vessels). Because the xylem's
lumen (opening) is so tiny, water will move up by capillary. However, this force is weak and its role in
moving water up the stem is small.
• Root pressure is a force that pushes water up the xylem. As water enters the root by osmosis, it pushes
the water that is already in the xylem of the stem upwards.
• Transpiration suction force is a very important force that pulls water up the xylem of the stem. As
water evaporates from the stomata of the leaves during transpiration, it creates a sucking force that
will pull the water up the xylem.

2.1.1.1.4.3 Investigation: plant tissue anatomy (root and stem)

Aim: To examine the structure of the root and stem


Available for free at Connexions <http://cnx.org/content/col11410/1.3>
63
Apparatus

• Scalpel or knife
• Celery stalk (stem)
• Carrot (root)
• Glass slide
• Iodine solution (Stain) or water
• Cover slip
• Dissecting needle or tweezers
• Paper and pencil
Method
1. Cut a very thin slice (cross section) from the middle of the celery stem or the carrot root.
2. Place this section on a glass slide.
3. Cover the specimen with iodine solution in order to stain it. This makes it more visible under the
microscope. The specimen can also be placed on a drop of water if iodine is not available.
4. Cover the specimen by carefully lowering the cover slip onto it with a dissecting needle or tweezers.
Take care not to trap any air bubbles.
This link gives information about making a wet mount microscope slide and shows an instructional video.
http://www.microbehunter.com/2010/08/13/making-a-wet-mount-microscope-slide/
5. Call your teacher.
6. Switch on the microscope making sure the lowest objective is in position (the 4x objective).
7. Place your slide on the stage.
8. Focus the image under the 4x objective (lowest objective) and view the structure of the celery stem.
Switch to the 10x objective to look a little more closely. To see amazing details of the structure of plant
tissue, use the 40x objective and the slide, carefully observing all of the parts and dierent cells.
9. Once you are able to see cells,
10. Call your teacher.
11. Make a biological drawing of your specimen as viewed under the microscope. Take note of the
magnication and draw a scale bar. Label your diagram according to the tissues you have learnt about.
Variation: Be creative and try using your favourite vegetables! Which vegetables are roots, stems and
leaves?
To prepare a slide:

Available for free at Connexions <http://cnx.org/content/col11410/1.3>


64 CHAPTER 2. LIFE PROCESSES IN PLANTS AND ANIMALS

Figure 2.9

Place the sample in the centre of the slide. Add a drop of iodine or water on top of the sample. Place
the cover slip next to the droplet as shown in the diagram.
Lower the coverslip into place with tweezers. As you lower the coverslip downwards, the drop will spread
outward and suspend the sample between the slide and the coverslip.
(Diagrams from http://www.ehow.com/how_5164819_prepare-wet-mount-slide.html )
2.1.1.1.4.4 Investigation: water uptake by stem

Aim: To examine the uptake of water by the stem


Apparatus:

• Water
• Food colouring dye (available at supermarket)
• White ower on a stem, e.g. Impatients, carnation or chrysanthemum
• Scissors
• Two jars, cups or measuring cylinders
• Plastic tray
• Sticky tape
Method:
Before starting this experiment, try to guess how the dye might move up the stem into the ower.
1. Fill one jar with plain water, and one with water containing several drops of food colouring dye.
2. Take the ower and carefully cut the stem lengthwise, either part way up the stem or right up to the
base of the ower (try both  the results will be dierent!)
Available for free at Connexions <http://cnx.org/content/col11410/1.3>
65
3. Put one half of the stem into the jar containing plain water and one half of the stem into the jar
containing food colouring dye. To make it easier to insert the stalks without breaking them, it helps to
wedge paper underneath the jars so that you can tilt them towards each other. Tape the jars or cylinders
down onto a tray so that they do not fall over.
4. Observe the owers after a few hours and the next day, and note where the dye ends up in the
owerhead. You can leave the owers up to a week but be sure to make sure that they have enough water.
Variation: Instead of using one cylinder with water and one with food dye, use two dierent colour
food dyes (e.g. blue and red). At rst the ower will show two separate colours, but as time goes by the
whole ower will show both dyes. This is because water can move sideways between xylem vessels through
openings along their length. The ability of water to move sideways between vessels is useful for when air
becomes trapped in a vessel, causing a blockage. If you cut the stem right up to the base of the ower, this
will limit movement between the xylem vessels.
Variation: Try using celery stalks with leaves. Cut open the celery stalk (cross-section) and you will
see that the little holes inside are coloured  these are the vessels.
An example of this experiment with photographs can be found
at: http://www.practicalbiology.org/areas/intermediate/cells-to-systems/transport-
in-plants/investigating-transport-systems-in-a-owering-plant,70,EXP.html
2.1.1.1.5 Secondary growth

2.1.1.1.5.1 Meristematic Tissue

• Meristematic tissue consists of small cells that are unspecialized. These cells divide by mitosis to form
new cells that can dierentiate (undergo changes in their structure) and can become specialized tissue
(e.g. xylem, phloem, epidermal cells)
• Primary meristematic tissue is found in the tips of roots, stems and buds. When it divides new cells
are produced which causes the plant to grow longer. This is referred to as primary growth.
• Secondary meristematic tissue originates from permanent tissue, usually parenchyma tissue which
divides by mitosis. Cambium is secondary meristematic tissue that is found in roots and stems. When
these cells divide by mitosis it results in the plant becoming wider. This is called secondary growth.
• Every growing season the stem of a plant increases in width  this is known as secondary thickening.
• Towards the end of the rst year of growth, the parenchyma cells between the vascular bundles become
meristematic and link up with the cambium tissue to form a cambium ring.
• The cells in the cambium ring start dividing to form secondary phloem (on the outside) and secondary
xylem (on the inside).
• Each year another ring of secondary phloem and secondary xylem is formed, making the stem grow
wider.
• It is not possible to see the layers of secondary phloem but the secondary xylem are visible. These
form rings called annular rings which can be used to work out the age of a plant.
• As new rings are formed each year, the older rings are pushed inward and the xylem vessels collapse
due to the pressure. The wood in the centre becomes denser and harder than the wood at the surface
and is called heartwood.
• The youngest annual rings found on the outside serve its function of transporting water. This wood is
not as dense and is called sapwood.
• The light-coloured rings are called spring wood. They are formed during spring and summer when the
growing conditions are favourable. The rings are therefore relatively thick and light in colour as the
xylem cell walls are thin.
• The dark-coloured rings are called autumn wood. They are formed during autumn and winter when
the growing conditions are unfavourable. The rings are therefore relatively thin and dark in colour as
the xylem cell walls are thick.
http://www.emc.maricopa.edu/faculty/farabee/biobk/biobookplantanat.html
This website provides information on plant structure and support.
Available for free at Connexions <http://cnx.org/content/col11410/1.3>
66 CHAPTER 2. LIFE PROCESSES IN PLANTS AND ANIMALS

Figure 2.10

This diagram shows the process of secondary thickening in stems

Available for free at Connexions <http://cnx.org/content/col11410/1.3>


67

Figure 2.11

This diagrams shows the annual rings of a tree trunk


2.1.1.1.5.2 Investigation - Tree rings and climate change

Every year a tree forms a new layer of xylem around the trunk. This forms tree rings, which are visible as
circles in a cross section of a tree that has been cut down. Each tree ring, or wood layer, consists of two
colours of wood; light wood that grows in spring and summer and dark wood that grows in autumn and
winter. Tree rings can be counted to give you a rough estimation of the age of a tree. Occasionally a tree
will form many rings in one year or miss forming rings in a year. The width of tree rings is greater in years
where good growing conditions occur. In years with droughts or low temperatures, the trees will produce
smaller rings. Therefore, by looking at the tree rings you can get an idea of the weather aecting a tree in
a particular year. Scientists can use this information to help determine the weather patterns of the past as
well as events such as forest res, earthquakes and volcanic eruptions. The study of past events using the
growth rings of trees is known as dendrochronology (dendros = tree, chronos = time).
Aim: to observe annual tree rings to assess age and climatic conditions
1. Find a cut or fallen tree, and count the tree rings, starting with the innermost ring. Measure the
width of each ring using a ruler, or make a note of whether a ring is narrow or wide. Make a note of any
scars caused by events such as res or pests.
2. Draw a bar graph showing the width of your tree rings for every year of the tree's life.

Available for free at Connexions <http://cnx.org/content/col11410/1.3>


68 CHAPTER 2. LIFE PROCESSES IN PLANTS AND ANIMALS

3. How old is your tree? What can you say about the climatic conditions throughout the life of your
tree?
http://www.classzone.com/books/earth_science/terc/content/investigations/es2905/
es2905page01.cfm
This is a link to an online tutorial about counting tree rings.
http://www.arborday.org/kids/carly/lifeofatree/
This is a link to a great cartoon video about the dierent tissue layers in trees (xylem, phloem, etc) and
the formation of tree rings.
http://www.worsleyschool.net/science/les/tree/rings.html
This is a link to a good website about tissue layers in trees.
2.1.1.1.6 Economic importance of plant support tissues

Plant support tissue supplies with two important resources namely wood and bre . Xylem is a source
of wood and the sclerenchyma is a source of bre.
Of course in order to obtain wood and bre we need to cut down numerous trees. This is called defor-
estation . Deforestation has escalated in the recent years due to the growing need for wood.
Activity
Collect data showing the area covered by forests in the years 1990 and 2010. Find this data for the
following countries: South Africa, Europe, Asia, North and Central America and South America.
Divide the class into two teams. One group will argue the need for us to cut down trees and the other
will be responsible for convincing us that deforestation must be reduced dramatically.
2.1.1.1.7 INDIGENOUS KNOWLEDGE

Making paper, ax, cotton, sisal. Traditional use of brous plants by san bushman using sansevaria to make
rope etc. thatching. Can we please get help with this?
2.1.1.1.8 Dicotyledonous leaf

2.1.1.1.8.1 Internal structure of the dicotyledonous leaf

Refer to chapter 1 to remind yourselves of the internal structure of a dicotyledonous leaf.

Available for free at Connexions <http://cnx.org/content/col11410/1.3>


69

Figure 2.12

This diagram shows the movement of water through a dicot leaf.


2.1.2 2.1.2 Transpiration2
2.1.2.1 Transpiration

Transpiration is the evaporation of water from the leaves of plants. Water is lost from the leaf through
special pores called stomata. Stomata are found on both surfaces of the leaf but there are usually more on
the ventral (lower surface ) of the leaf. This is to reduce the amount of transpiration that will occur because
the top of the leaf is exposed to more sunlight than the bottom.
http://education.uoit.ca/lordec/ID_LORDEC/transpiration_pull/ 3

This interactive website explains transpiration pull. Plants use the process of transpiration pull to move
water from the soil up into the leaves.
Heat from the environment causes the water in the sub-stomatal air spaces to evaporate out of the
stomata. This process is called transpiration.
Transpiration is therefore dened as the loss of water vapour from the leaves of a plant.
Transpiration only occurs during the day when the stomata are open. At night the stomata are closed.
2 This content is available online at <http://cnx.org/content/m43163/1.1/>.
3 http://education.uoit.ca/lordec/ID_LORDEC/transpiration_pull/

Available for free at Connexions <http://cnx.org/content/col11410/1.3>


70 CHAPTER 2. LIFE PROCESSES IN PLANTS AND ANIMALS

Excess water diuses into the sub-stomatal air spaces to replace that which has been lost from the leaves
Water diuses from the xylem of the leaf into surrounding mesophyll cells.
Water circulates to supply plants with their water requirements.
Water is pulled from the xylem of the stem to the xylem of the leaves. The xylem is found in the veins
of the leaf.
2.1.2.1.1 Rate of transpiration

This increases in conditions of . . .


• High light intensity (bright sunlight)
• Increased temperatures (hot weather)
• Wind
• Low humidity (dry conditions)

Light Plants transpire more rapidly in the light than in the dark. This is largely because light stimulates
the opening of the stomata. Light also speeds up transpiration by warming the leaf.
Temperature Plants transpire more rapidly at higher temperatures because water evaporates more
rapidly as the temperature rises due to the increased kinetic energy of the water molecules. At 30 ◦ C, a leaf
may transpire three times as fast as it does at 20 ◦ C.
Wind When there is no breeze, the air surrounding a leaf becomes increasingly humid thus reducing
the rate of transpiration. When a breeze is present, the humid air is carried away and replaced by drier air.
So a steep diusion gradient is maintained.
Humidity The rate of diusion of any substance increases as the dierence in concentration of the
substances in the two regions increases. When the surrounding air is dry, diusion of water out of the leaf
goes on more rapidly.
Soil water A plant cannot continue to transpire rapidly if its water loss is not made up by replacement
from the soil. When absorption of water by the roots fails to keep up with the rate of transpiration, loss
of turgor occurs, and the stomata close. This immediately reduces the rate of transpiration (as well as of
photosynthesis). If the loss of turgor extends to the rest of the leaf and stem, the plant wilts.
The volume of water lost in transpiration can be very high. It has been estimated that over the growing
season, one acre of corn (maize) plants may transpire 1.5 million litres of water. As liquid water, this would
cover the eld with a lake 38 cm deep. An acre of forest probably does even better.
This diagram shows a potometer which is used to measure the rate of transpiration. As the leafy twig
transpires, the air bubble moves to the left. The quicker the air bubble moves the faster the leafy twig is
transpiring.

Available for free at Connexions <http://cnx.org/content/col11410/1.3>


71

Figure 2.13

Suggestion: Can we have a


graph relating to
transpiration rate over time?
Also add an arrow to show
movement of the bubble
TOWARDS the plant for
greater clarity. [clark]
The diagram below shows a summary of the movement of water from the roots to the leaf.

Figure 2.14

2.1.2.1.2 Why do plants need water?

Plants need water to maintain turgor pressure. This helps provide support for the plant as when a cell
absorbs water the cell membrane pushes against the cell wall. The cell is now turgid. If there isn't enough
water in the plant the membrane moves away from the cell wall and the cell is now accid. This is when a
Available for free at Connexions <http://cnx.org/content/col11410/1.3>
72 CHAPTER 2. LIFE PROCESSES IN PLANTS AND ANIMALS

plant begins to wilt and it will eventually die. Wilting is the loss of rigidity of the non-woody parts of plants
and occurs when the turgor pressure falls towards zero. Lower water availability can results from drought
conditions, high salinity, saturated soil conditions, low temperatures or bacterial or fungal infections that
aect the vascular system of the plant,
When the environment is extremely humid (moist) the rate of transpiration is very low. In some plants,
the leaves secrete water onto the surface of the leaves through specialised pores called hydathodes. Hy-
dathodes are the open ends of xylem vessels at the edges of the leaves in certain species of plants such as
strawberries and some grasses.
2.1.2.1.2.1 Investigation  the eect of environmental conditions on transpiration rate (using
a simple potometer)

A potometer measures the rate of transpiration by measuring the movement of water into a plant. The
following experiment uses a simple hand made photometer.
Aim: To assess the eect of dierent environmental conditions (e.g. temperature) on transpiration
rate.
Apparatus

• a drinking straw
• a soft green leafy shoot
• Vaseline
• Marking pen
• Play dough / putti
• Plastic bag
• Elastic band
• Ruler
Method
Perform the following steps under water
1. Cut the stem of the leafy shoot under water .
2. Test to make sure the stem of the leafy twig will t snug tightly into the top of the straw.
3. Remove the leafy shoot from the straw and set aside.
4. Fill the straw with water. Place your nger over one end of the straw to stop the water from running
out.
5. Put the leafy shoot into the open end and seal it with play dough while removing it from water (KEEP
FINGER ON THE STRAW!)
Perform the following steps above water
1. Seal with Vaseline. Make sure it is air tight and water tight  if not, all the water will run out when
you take your nger o the straw.
2. Mark the water level on the straw.
3. Place your photometer under one of the following conditions for one hour:

• as is, in a warm, sunny place (no wind)


• as is, in a warm, windy place
• with a plastic bag tied around the leaf, in a warm, sunny place.
• A shady place

1. After an hour: use the marking pen to mark the change in water level on the straw.
2. Measure the distance the water moves.
Available for free at Connexions <http://cnx.org/content/col11410/1.3>
73
Results

• Draw a table and record the class' results.


• Plot a bar graph to compare the distances the water moved in the dierent straws.
Discussion

• Why is it important to cut the stem under water?


• What does the water movement in the straw indicate?
• Which four external environmental factors are you investigating?
• Under which condition is water loss from the leaf the greatest?
Conclusion

• What can you conclude from this investigation?


• Give two ways in which you can improve your experimental results.
More information about potometer experiments can be found on the following websites:
http://www.practicalbiology.org/areas/advanced/exchange-of-materials /transpiration-in-
plants/measuring-rate-of-water-uptake-by-a-plant-shoot- using-a-potometer,62,EXP.html 4

http://www.practicalbiology.org/areas/advanced/exchange-of-materials /transpiration-in-plants/ 5

2.1.3 2.1.3 - Movement of manufactured food6


2.1.3.1 Movement of manufactured food

Plants use carbon dioxide and water to manufacture glucose and oxygen is the waste product. Sunlight and
enzymes are necessary for photosynthesis to occur. Once the food is manufactured in the leaves it needs to
be distributed to the entire plant so that the glucose can be used by each cell for respiration (manufacture
energy).
Sunlight and enzymes
water + carbon dioxide ↔ glucose (carbohydrates) + oxygen
The glucose is manufactured mainly in the palisade cells and then passes into the phloem. Transport
of food material from leaves to other parts of the plant is called translocation. This food may be stored in
roots, stems or fruit.
Read more: Anatomy of Plants - Biology Encyclopedia - cells, body,
function, system, dierent, organs, hormone, structure, types, membrane
http://www.biologyreference.com/A-Ar/Anatomy-of-Plants.html#ixzz1an9JO8yK
Phloem tissue is made up of two dierent types of cells which are sieve tubes and companion cells. Sieve
tubes are the main conducting cells. These cells look like a sieve and phloem sap moves from cell to cell
though the phloem walls. Unlike cells of the xylem, sieve tubes are alive at functional maturity, but do
not have nuclei. Companion cells have nuclei and are closely associated with sieve tubes. Companion cells
carry out all the cellular functions of the sieve tubes. The cytoplasm of sieve tubes and companion cells
is connected through numerous channels called plasmodesmata. These cytoplasmic connections allow the
companion cells to regulate the content and activity of the sieve tube cytoplasm. The companion cells also
help load the sieve tube with sugar and the other metabolic products that they transport throughout the
plant.
4 http://www.practicalbiology.org/areas/advanced/exchange-of-materials/transpiration-in-plants/measuring-rate-of-water-
uptake-by-a-plant-shoot-using-a-potometer,62,EXP.html
5 http://www.practicalbiology.org/areas/advanced/exchange-of-materials/transpiration-in-plants/
6 This content is available online at <http://cnx.org/content/m43091/1.1/>.

Available for free at Connexions <http://cnx.org/content/col11410/1.3>


74 CHAPTER 2. LIFE PROCESSES IN PLANTS AND ANIMALS

Figure 2.15

Aphids feeding on phloem sap

Available for free at Connexions <http://cnx.org/content/col11410/1.3>


75
2.1.4 2.1.4 Summary7
2.1.4.1 Summary

Plants are made up of roots, stems, leaves and owers. The various types of tissues found within plants
work together to full functions such as support and transport of water and nutrients. In this chapter you
have learnt about the structure of roots and stems, and found out how they function. Roots consist of
a protective epidermis, a cortex to store food and water, and an endodermis surrounding an inner stele,
which is composed of vascular tissue important in transport. Xylem is responsible for transporting water
and minerals up from the roots to the stem, while phloem is responsible for transporting food (which the
plant has made by photosynthesis) from the leaves to the roots. Similarly, a stem consists of a waterproof
cuticle, a protective epidermis, a cortex for storage, and contains vascular bundles arranged in a ring (in
dicotyledonous plants) or scattered throughout the stem (in monocotyledonous plants). The most important
force that causes the movement of water through the xylem, from the roots up to the stem and leaves, is
transpiration. As water evaporates from the stomata of the leaves during transpiration, it creates a suction
force that pulls the water up through the roots. Hot, bright, dry and windy conditions can all lead to an
increase in the rate of transpiration. If plants lose too much water and do not replace it, this causes a loss of
turgor pressure, and the plant will wilt and eventually die. Water is also required for photosynthesis, which
is the process whereby plants use water, carbon dioxide and the energy from sunlight to make glucose, and
release oxygen as a by-product. Glucose is manufactured mainly in the palisade cells of the leaves, from
where it passes into the phloem and is transported to the rest of the plant. This food may be stored in the
roots, stems or fruit.

2.2 Support systems in animals


2.2.1 2.2.1 Skeletons8
2.2.1.1 Skeletons

Skeletons in animals:
http://www.clisnotes.com/WileyCDA/ClisReviewTopic/Skeletons-in- Animals.topicArticleId-
8741,articleId-8716.html 9

Skeleton  interactive animation


http://www.medtropolis.com/VBody.asp 10

2.2.1.1.1 Overview

• The word support means to hold upright or to give strength


• Aquatic organisms get support from water. Plants and animals living on land require stronger support
systems as air gives their bodies little support against the force of gravity.
• Animals have skeletons that support their bodies and enable movement to occur.
• In animals movement is brought about by the contraction or relaxation of muscles. In order to bring
about movement muscles need a medium (uid) or solid structure against which the force of contraction
can be applied. A skeletal system provides the resistance needed to cause movement.
• Animals are able to move from one point to another to look for food, shelter and mates.
• The simplest invertebrates have specialised cells and tissues to assist them to move to and from stimuli.
• Skeletons also have a protective function in that they cushion vital organs thereby preventing or limiting
damage.
7 This content is available online at <http://cnx.org/content/m43074/1.1/>.
8 This content is available online at <http://cnx.org/content/m43098/1.1/>.
9 http://www.clisnotes.com/WileyCDA/ClisReviewTopic/Skeletons-in-Animals.topicArticleId-8741,articleId-8716.html
10 http://www.medtropolis.com/VBody.asp

Available for free at Connexions <http://cnx.org/content/col11410/1.3>


76 CHAPTER 2. LIFE PROCESSES IN PLANTS AND ANIMALS

2.2.1.1.2 Hydrostatic skeleton

Figure 2.16

• It consists of a uid-lled cavity enclosed by the muscles of the body wall


• The uid presses against the muscles, that contract against the pressure of the uid
• So, a combination of the pressure of the uid and the contracting muscles, can alter the shape of the
animal and allows for movement
• If the body is segmented the pressure of the uid is localised in a few segments at a time.
• Occurs in atworms, round worms, earthworms starsh, slugs etc.
• Note that starsh and other Echinoderms have an outer skeleton of calcareous ossicles or spicules for
protection . This outer skeleton encloses a water vascular system with tube feet that are moved by
uid pressure changes i.e serves as a hydrostatic skeleton which controls movement.

2.2.1.1.2.1 Advantages:

• allow the animal to move in a more exible manner


• uid cavity stimulates circulation in the animal
• allows for change of shape e.g. earthworm

Available for free at Connexions <http://cnx.org/content/col11410/1.3>


77

Figure 2.17

2.2.1.1.2.2 Disadvantages:

• dehydration will aect the skeleton directly and the ability of the animal to move because of the loss
of shape
• does NOT provide protection for the internal organs
• does not allow for rapid movement.

2.2.1.1.3 Exoskeleton

Figure 2.18

• This forms the outer covering of the animal


• The skeleton is made of a substance called chitin, secreted by the epidermis
• It is conned to insects, spiders, scorpions, crabs etc all of which belong to the Arthropod group
(jointed legged animals
• The abdomen is soft and attached to the thorax
• The exoskeleton acts as a hard outer covering to animals and is made up of a series of plates or tubes.

Available for free at Connexions <http://cnx.org/content/col11410/1.3>


78 CHAPTER 2. LIFE PROCESSES IN PLANTS AND ANIMALS

• Muscles are attached to the inside of the exoskeleton which provides the resistance needed for muscle
action.

2.2.1.1.3.1 Advantages:

• forms the point of attachment of internal muscles needed for locomotion and ight
• supports and protects the delicate inner parts of the animal
• prevents desiccation (drying out) on land
• has a low density and is therefore lightweight, to allow for ight
• mouthparts can be modied for biting, sucking, piercing
• grasping etc.

Figure 2.19

2.2.1.1.3.2 Disadvantages:

• nal body size is limited because as the body size increases, the surface area to volume ratio decreases.
The larger the animal, the heavier the exoskeleton, making movement more dicult.
• growth is restricted, so periodic moulting is required if the animal is to grow
• very vulnerable when it is in the moulting process, as it cannot move until the exoskeleton is dry and
has hardened

2.2.1.1.4 Endoskeleton

• This skeleton is found inside the body and can consist of bone (vertebrates) or cartilage (sharks).

Available for free at Connexions <http://cnx.org/content/col11410/1.3>


79

Figure 2.20

2.2.1.1.4.1 Advantages:

• Endoskeletons consist of living tissue - so it is able to grow steadily within the animal enabling some
to reach a large size.
• It can support the weight of a large animal
• the skeleton is jointed which allows for exible movement and support
• muscles attach directly to the skeletal bones to allow for movement and support
• vital organs are protected by bone cavities like the ribcage and the pelvic girdle
• lends itself to adaptation to the environment Vertebrates move in dieret ways ( e.g. running, jumping,
swimming, ying.)

2.2.1.1.4.2 Disadvantages:

• Lack of mineral elements like calcium and phosphates will cause brittle bones and aect movement
and support
• Lack of vitamin D in the diet results in a condition/disease caused rickets. A disease characterised by
bowed legs.

2.2.1.2 Developmental progression and why skeletons are needed for terrestrial life

2.2.1.2.1 Problems that animals moving from water to land had to overcome

• Loss of body support provided by water


Water is about 1000 times denser than air and is a much more resistant medium to move through, its high
density buoys up the body providing support.. One of the major problems by animals moving from water
to land was the increase in the force of gravity. In order to counter this, animals needed to develop strong
limbs and to adapt the skeleton to support their body weight on land.
• Locomotion

Available for free at Connexions <http://cnx.org/content/col11410/1.3>


80 CHAPTER 2. LIFE PROCESSES IN PLANTS AND ANIMALS

Moving eectively on land is essential particularly if one needs to avoid predator, catch prey or adapt to a
particular habitat.
• Desiccation

Exposure to air results in evaporation of water from breathing organs and body surface. If this problem is
not overcome animals will be conned to damp habitats
Constancy of temperature
Natural bodies of water do not show much temperature uctuation, whereas on land, the range and uc-
tuation in temperatures can be large. This can cause huge problems for animals which have no temperature
control mechanisms.
• Gaseous exchange

Gills depend on water ow or movement through water to operate. To survive on land a completely new
mechanism of breathing needed to evolve.
• Opportunities for breeding on land

The provision of safe shelter for the protection of vulnerable eggs and young is far easier on land than in water
habitats.. The main problem here was to nd methods of reproduction which did not require fertilization in
water
• Variety of habitats

On land there are a tremendous variety of habitats e.g. tropical, coniferous and temperate forests, grasslands,
deserts, mountains, oceanic islands and polar regions. Aquatic habitats, despite being far less diverse contain
the greatest number and variety of living organisms on earth. Each habitat will have dierent requirements
with regard to access to food shelter, protection from enemies etc.
2.2.1.2.2 The exoskeleton in terrestrial arthropods

Arthropods moved onto the land long before the vertebrates emerged and a major factor in their success
was the exoskeleton.
• Highly protective without sacricing mobility
• Provides attachment for muscles which control movement of appendages
• In insects and spiders it is impregnated with waxes forming a waterproof barrier which prevents drying
out.
Most terrestrial Arthropods have a tracheal system of breathing whereby air is forced in and out

of a system of tubes by means of contraction and relaxation of muscles attached to the abdominal
exoskeleton.
• Most insects have one or two pairs of wings which are formed from outgrowths in the thoracic region
of the exoskeleton
• The jointed appendages have sensory hairs and can be modied and adapted for sensory functions
(antennae), food handling(mouthparts), swift and ecient walking legs and swimming appendages.
The exoskeleton because of its restriction on the size of terrestrial Arthropods was a major factor in future
prominence of the vertebrates.
2.2.1.2.3 Developmental progression in the vertebrate skeleton and associated organs.

The two major requirements for survival on land are the development of a suitable support system and an
air breathing mechanism.
Available for free at Connexions <http://cnx.org/content/col11410/1.3>
81
2.2.1.2.3.1 Fish

• Bony sh and cartilaginous sh (sharks, rays etc) , apart from the skull have a skeleton consisting of
a vertebral column to which bands of muscle are attached on either side.
• When swimming a wave of contraction is produced on one side followed by another wave on the opposite
side.
• These alternate waves of contraction gives rise to S-shaped side to side movement of the
body which together with the sweeping movement of the tail drive the sh forward. The ns keep
stability and play a role in steering.. This method of locomotion depends on the resistance of the
surrounding water and would not work on land.
• A prehistoric group of sh had lobed ns . These sh lived in swamps or ponds which often dried
up.
• It is thought that they used their ns to drag themselves from a dried up pond to a wet one. Fossil
forms seem to indicate that these ns contained bones .
• They also had primitive lungs which enabled them to inhale small amounts of air.
• Three existing sh which have similar features are the coelacanth from deep seas, the mudskipper
which occurs in mangrove swamps and uses its front lobed ns to climb trees, and the lungsh which
live in stagnant water surfacing to gulp air. None of these developed into Amphibia which arose from
one or more of the extinct forms.

2.2.1.2.3.2 Amphibia

• Amphibia: have partially adapted to land conditions.


• Firstly they have developed lung breathing in adults while tadpoles use gills.
• Secondly they have developed limbs for locomotion on land..
• One group salamanders and newts (not found in Africa) resemble small lizards. Their legs are mostly
at right angles to the body and are small . Like sh their bodies bend from side to side when
they walk.
Frogs have adapted their skeletons to land conditions in the following ways:

• skull is much lighter and more attened to allow for mobility on land.
• the exible vertebral column of sh has been shortened and acts as a r igid frame for transmitting
force to the body .
• There has been an extreme shortening of the body (9 vertebrae and there is no tail), as this would be
a hindrance when leaping and landing
• there is a pectoral girdle which serves as support for the forelimbs which are used mainly to
absorb weight during landing after a jump.
• the pelvic girdle is elongated and bear the hind limbs . When at rest the thigh, calf and foot
of a frog are each about the same length. As the frog jumps each part of the leg straightens in turn
which results in great leaping power The webbing between the toes increase the thrust in the air as
well as when swimming in water.
• the frog's leap is not only used to move from one point to another but is also a very eective way of
escaping from a predator.
Despite these skeletal developments in adapting to land, they are still dependent on water in
that:

• They have a naked skin and are restricted to moist areas as they have no protection against
desiccation.
• They have not developed an egg that is suited to terrestrial conditions and they thus have to lay
their eggs in water in order for the tadpole to mature into a frog.

Available for free at Connexions <http://cnx.org/content/col11410/1.3>


82 CHAPTER 2. LIFE PROCESSES IN PLANTS AND ANIMALS

2.2.1.2.3.3 Reptiles

The two major advantages that reptiles have over amphibians with regard to adaptations to terrestrial life
are
• The development of a tough dry scaly skin which oer protection against desiccation and physical
injury.
• The development of a shelled egg containing food and protective membranes which allow embryonic
development to take place on land.
With regard to the skeleton
• All reptiles except those that are limbless ( i.e. snakes) have better body support than the
amphibians and more eciently designed limbs for travel on land .
• Most reptiles including crocodiles, most lizards and tortoises do not have an ecient form of locomotion
as the limbs are spread out at the sides of the body and the walk is a slow and waddling one. This is
exhausting as a great deal of muscular eort is needed to keep the body o the grounds. Such reptiles
cannot move rapidly and cannot support a large size.
• Some reptiles overcame this problem by becoming bipedal i.e only using the hind limbs for locomo-
tion. Others gradually over time shifted their limbs to under the body. Many dinosaurs followed this
route and were able to support a greater weight which is why some dinosaurs became huge.
• The development of ribs with a sternum(breastbone) has allowed for larger lungs and a more
ecient method of inhaling and exhaling air by the inward and outward movement of the ribs.

2.2.1.2.3.4 Birds

Birds dier from the other vertebrates in that as a group


they are able to y and are designed for
ight. and many of their skeletal adaptations are linked to ight . The main feature which identify
a bird are its feathers and they are the only modern animals to have them
• Feathers are modied reptilian scales and there are two types
· Contour feathers which are attached to the wings in such a way that they overlap to produce
a broad at surface benecial for ight..
· Down feathers provide excellent insulation against body heat loss. This is important as birds,
like mammals are endothermic i.e. they generate heat in order to maintain a constant
body temperature .
• The paired forelimbs are usually adapted for ying ( an exception is in penguins where they
are used as paddles for swimming).
• The bones are hollow and light due to the presence of air cavities .
• Jaws are covered by horny beaks which have varied shapes according to the nature of the food source
• The sternum/breastbone is enlarged and has a keel -like extension which provides attachment
for the strong muscles used for ying.. Flightless birds such as ostriches do not have a keel.
• The feet are also adapted to the bird's mode of life e.g. talons for birds of prey, webbed feet for
waterfowl. Perching feet for songbirds.

2.2.1.2.3.5 Mammals

• Body is covered by hair but mammals dier in the amount, distribution and type of hair.
• Apart from two species, mammals do not lay eggs, instead they are retained in a uterus and are
nourished by an organ called the placenta
• After birth the young are nourished by milk from mammary glands .
Available for free at Connexions <http://cnx.org/content/col11410/1.3>
83
• Have 2 sets of teeth (milk teeth replaced by permanent teeth)
• Have 4 dierent types of teeth namely;
· Incisors : mainly for biting, snipping or gnawing  prominent in rodents and grazing animals.
Elephant tusks are incisors
· Canines : have long cone shaped crowns (the part of the tooth that s above the gums) and are
well developed in carnivores. They have sharp edges for tearing and piercing.
· Premolars : have compressed crowns and one or two cusps (ridges) used for shearing and
slicing
· Molars : with large bodies and variable cusp arrangement, are used for crushing and mastication.
• There are four limbs (reduced or absent in some) adapted for many forms of locomotion.
• In most four legged mammals the leg bones are held directly underneath the body . In this
position they act as props or struts and it is the bones rather than the muscles that take most of the
strain of the body's weight. For this reason the animal is able to support the body clear of the ground
for long periods of time without tiring.
• Like birds they are endothermic and can maintain a constant body temperature.
• Have a muscular partition or diaphragm between the thorax (chest region) and abdomen to make
breathing more eective.
• They have a highly developed cerebrum (the brain's most complex region).

2.2.2 2.2.2 Human Skeleton11


Human Skeleton
• Humans have an internal skeleton made of bone, cartilage and connective tissue.

Figure 2.21: Human skeleton from the back

11 This content is available online at <http://cnx.org/content/m43146/1.1/>.

Available for free at Connexions <http://cnx.org/content/col11410/1.3>


84 CHAPTER 2. LIFE PROCESSES IN PLANTS AND ANIMALS

Figure 2.22: Human skeleton from the front

2.2.2.1 Axial skeleton

This part of the skeleton consists of the skull comprising the cranium, facial bones, foramen magnum,
palate and jaws, vertebral column, rib cage and breastbone(sternum)
Axial skeleton animation
http://www.wisc-online.com/objects/ViewObject.aspx?ID=AP12904 12

2.2.2.1.1 The Skull: consists of the cranium and facial bones

•The cranium consists of eight at bones joined together by immovable joints called sutures. The
cranium surrounds and protects the brain.
• There is a large opening at the base of the skull called the foramen magnum through which the
spinal cord passes
• On either side of the foramen magnum is a projection or condyle which articulate with the rst
vertebra (called the atlas) to give the nodding movement of the head
• There are 15 facial bones. These are irregular bones that include cheek nasal , temple and upper
and lower jaw bones. The only movable bone is the lower jaw.
• The upper and lower jaws bear the sockets for the 32 permanent teeth.
• The number, type and arrangement of the teeth in an animal is indicated by a dental formula The
human dental formula is: 2.1.2.3
2.1.2.3
12 http://www.wisc-online.com/objects/ViewObject.aspx?ID=AP12904

Available for free at Connexions <http://cnx.org/content/col11410/1.3>


85
This formula represents the numbers of each type of tooth in half of the upper jaw and half of the lower
jaw. This formula tells us that in both the upper and lower halves there are 2 incisors, 1 canine, 2 premolars
and 3 molars. Therefore in the whole jaw there are 8 incisors, 4 canines 8 premolars and 12 molars, a total
of 32 teeth in all.
2.2.2.1.2 THE VERTEBRAL COLUMN : CONSISTS OF 33 VERTEBRAE

The vertebral columns is divided into ve regions


• A cervical (neck) region consisting of 7 vertebrae. The rst of these called the atlas supports
the skull and the joint with the skull allows for the nodding movement of the head.

The 2 nd
vertebra called the axis has a projection oo which the atlas pivots to give the side to side
movement of the head.
• A thoracic region (chest) of 12 vertebrae each of which bears a pair of ribs
• A lumbar region (lower back) the largest vertebrae as they carry the weight of the body
• A sacral region consisting of 5 fused vertebrae forming a bone called the sacrum that forms part
of the pelvic girdle which provides for the attachment of muscles and the legs.
• A coccyx made up of 4 fused bones. These bones form the tail in those mammals that have tails.
• The vertebrae join up to each other in such a way that there is a continuous spinal canal which
runs from the base of the skull to the pelvic girdle. This canal contains the spinal cord.
• Between the vertebrae are discs of brocartilage which prevent friction between vertebrae and act
as shock absorbers during walking, running and jumping.
• Spinal nerves are able to enter and leave the spinal cord through gaps between adjacent vertebrae.
• Strong ligaments and muscles around the spine stabilise the vertebrae and help to control
movement.
CURVES OF THE SPINE
When viewed from the side the vertebral column can be seen to have
four curves, with the cervical
and lumbar regions curving forwards while the thoracic and sacral regions curve backwards.
In a newborn baby the entire vertebral column curves backwards probably because of the connes of the
uterus.
Initially a baby cannot support the weight of its head.. When after about 3 months it is able to support
its head, the cervical forward curve is complete .
The lumbar forward curve is complete when the baby is able to stand on its own and ready to learn
to walk.
These curves of the vertebral column provide some of the resilience and spring so essential in balance
and movement. Abnormal curves may be due to poor posture, congenital disease or bone disease.
FUNCTIONS OF VERTEBRAL COLUMN

• Supports the skull


• Surrounds and protects the spinal cord.
• Provides attachment for ribs, girdles, and back muscles
• Separate vertebrae and S-shaped curvature provide exibility allowing humans to bend backwards,
forwards and sideways.
• Cartilage discs act as shock absorbers
RIB CAGE
The rib cage consists of 12 thoracic vertebrae, 12 pairs of ribs and the sternum or breastbone
• All 12 pair of ribs are attached to the thoracic vertebrae
• The rst 7 pairs are attached to the sternum by cartilage. These are the true ribs

Available for free at Connexions <http://cnx.org/content/col11410/1.3>


86 CHAPTER 2. LIFE PROCESSES IN PLANTS AND ANIMALS

• The next 3 pairs are each attached .to the rib above by means of cartilage. These are the false ribs.
• The last 2 pairs are not attached at all to in the front and are called oating ribs.
• Between each pair of ribs are external and internal intercostal muscles
• The sternum is a at dagger shaped bone at the front of the rib cage.
Functions of rib cage

• Protection of the heart and lungs


• With the help of the diaphragm and the intercostals muscles they increase. And decrease the volume
of the thoracic cavity thereb allowing inhalation and exhalation to take place.

2.2.2.2 APPENDICULAR SKELETON

This part consists of the pectoral girdle with arms and pelvic girdle with legs

Figure 2.23

Appendicular skeleton animation


http://www.wisc-nline.com/objects/ViewObject.aspx?ID=AP13404 13

13 http://www.wisc-nline.com/objects/ViewObject.aspx?ID=AP13404

Available for free at Connexions <http://cnx.org/content/col11410/1.3>


87
2.2.2.2.1 THE PECTORAL GIRDLE AND ARMS

The pectoral girdle consists of 2 clavicles (collar bones) and 2 scapulae (shoulder blades). Each clavicle is
attached to the sternum in the front and the scapulae at the sides and they help to support they help to
support the shoulders. The clavicle is the most frequently broken bone in the body as it often takes the full
impact of falls on outstretched arms or of blows to the shoulder. Each scapula has a socket into which the
upper arm ts
Each upper arm has a single bone called the humerus which ts into the Glenoid cavity on the
scapula to form a ball and socket joint. This cavity is very shallow which allows the arms to move in
almost any direction for forms an elbow joint with the forearm
The forearm consists of two bones namely the ulna in line with the little nger and the radius which
lies above the thumb. The joint at the elbow is a hinge joint.
The wrist consists of 8 small carpal bones arranged in two rows of four.
The palm of the hand consists of 5 metacarpal bones. Their rounded ends at the base of the ngers
form the knuckles
There are 14 nger bones or phalanges in each hand, two in each thumb and three in each of the
ngers
Unlike the pelvic girdle the pectoral girdle is not fused to the spine but is connected by muscles to the
back of the thorax so its role as a supporting structure is much less than the pelvic girdle.. Instead it gives
the shoulders greater freedom of movement which in turn allows greater mobility of the arms.. Any limit to
movement is provided by the clavicle.-
Functions of pectoral girdle

• Forms a strong support for the attachments of the arms.


• Provides large area of bone for the attachment of muscles.
• Forms ball and socket joints with the arms which allows freedom of movement

2.2.2.2.2 PELVIC GIRDLE AND THE LEGS

The pelvic girdle consists of hip bones joined in the front by the pubic symphesis and attached to the sacrum
at the back.
Each hip bone consists of three fused bones namely the:
1. ilium, which forms the upper ared portion
2. ischium, which is the lowest and strongest part and
3. pubis, which forms the anterior part.
Portions of all three bones contribute to the formation of the acetabulum, the deep socket that holds the
head of the femur (thigh bone) to form the hip joint.
The female pelvic girdle is wider and lighter than the male as an adaptation to pregnancy and childbirth.
The femur is the largest and strongest bone in the body. The upper end forms a ball and socket joint
with the hip bone while the lower end articulates with the tibia to form the hinge joint of the knee.
The patella or kneecap is a at triangular bone which is embedded in the tendon of the thigh muscle
and attached by ligament to the tibia.
There are two bones in the lower leg: the tibia or shin bone which is the larger of the two and
supports most of the mass. The upper end articulates with the femur while the lower end articulates with
one of the tarsal bones to form the ankle joint. The bula (calf bone) is smaller than than the tibia and
serves mainly for the attachment of muscles
The structure of the foot is similar to that of the hand. However the foot supports the weight of the
body, so it is stronger and less mobile than the hand.. There are seven tarsals or ankle bones only one
of which articulates with the tibia. The largest of these is the heel bone(calcaneum) to which the calf
muscle is attached and which presses rmly on the ground when one stands, walks or runs.
Available for free at Connexions <http://cnx.org/content/col11410/1.3>
88 CHAPTER 2. LIFE PROCESSES IN PLANTS AND ANIMALS

There are 5 metatarsal bones the which form the ball and arch of the foot.
The 14 phalanges of the toes are the counterparts of those in the ngers, with the big toe having two
phalanges and the other 4 having 3 phalanges each. .
2.2.2.3 Functions of Skeleton:

1. Movement  allows body to move because muscles attach to the bones to give them leverage
2. Protection  protects vital organs ( skull=brain, ribcage=heart and lungs and pelvic
bones=digestive tract and reproductive organs)
3. Support  provides shape and support to body
4. Storage of minerals - bones store minerals such as calcium and phosphate ions
5. Hearing - bones in the middle ear, called the hammer, anvil and stirrup, amplify sound waves and
assist in the hearing process
6. Red blood cells production - long bones and at bones contain red bone marrow to produce red
blood cells

2.2.2.4 Structure of a long bone

2.2.2.4.1

2.2.2.4.2 PARTS OF A LONG BONE

Epiphysis: The head of each end of a long bone covered with hyaline cartilage and consisting largely
ofspongy bone .
Diaphysis: Cylindrical shaft of a long bone composed of hard compact bone on the outside

Available for free at Connexions <http://cnx.org/content/col11410/1.3>


89
Periosteum: The membrane of dense brous connective tissue which surrounds the outside surface
of the shaft of a long bone. It has blood vessels which enables it to nourish the bone and repair injuries.
It also provides a surface for the attachment of muscles, by means of tendons and ligaments.
Marrow cavity: This is lled with yellow marrow which consists largely of fat.
Endosteum: The delicate connective tissue layer lining the inside surface of compact bone.
Cancellous/spongy bone : Found in the epiphysis of long bones and contain red marrow.
Trabeculae: The struts in the network of irregular bony plates in the epiphysis of bones which transfer
stresses from the epiphysis to the diaphysis which has a much thicker layer of compact bone and resists stress
better.
Red bone marrow: Found in the spaces between the trabeculae in spongy bone. This is where the
red blood cells are made at the rate of 2 -3 million per second. White blood cell types are also produced
here.
2.2.2.4.2.1

TYPES OF BONES

• Long bones have a central shaft and two heads, one at each end. An example is the femur ,
which is the largest bone in the body.
• Flat bones have two layers of compact bone covering a layer of spongy bone on the inside, for
example the shoulder blades .
• Irregular bones and short bones have a thin layer of compact bone covering spongy bone on the
inside, for example vertebrae of the spine and the small bones in the hands and feet.

2.2.2.4.3 PRACTICAL INVESTIGATION

Experiment A:
Aim: To investigate the role of the inorganic and organic components of bone
Apparatus: Exp A.: 2 small chicken bones
2 test tubes
Dilute hydrochloric acid
Towel
Method:

1. Label 2 test tubes with your initials and A and B. Put a bone in each tube.
2.Cover Bone A with water and Bone B with dilute hydrochloric acid. Leave for a few days. The acid will
dissolve out the mineral component of the bone leaving behind the organic part
3.Take out Bone A and dry it.
4.Use tweezers to take Bone B out of the acid. Rinse it under the tap and dry it.
5.Compare the two bones and not down how they appear and whether they are soft or hard, exible or
brittle
Experiment B
Apparatus: 1 small chicken bone
Pipe clay triangle or wire gauze on a tripod stand
Bunsen burner or Methylated spirits burner
Method:
1. Place the chicken bone (Bone C) on a pipe triangle or wire gauze on a tripod stand
2.Roast the bone strongly for 10 minutes. Roasting will burn o the organic component of bone ( mainly
the protein collagen) leaving behind the mineral part
3.Allow the bone to cool down completely before you touch it.
4.Describe the appearance of Bone C stating whether it is soft or hard, exible or brittle
Available for free at Connexions <http://cnx.org/content/col11410/1.3>
90 CHAPTER 2. LIFE PROCESSES IN PLANTS AND ANIMALS

Questions
1.What are the main inorganic components of bone?
2.What changes have occurred in Bone A?
3.What properties have been removed from the bone with the loss of it inorganic components?
4.Which deciency disease can give similar results on bones in children?
5.What is the role of Bone B in this experiment?
6.What protein makes up the main organic component of bone?
7. What changes took place in Bone C during the roasting process?
8. What properties have been removed from Bone C with the loss of its organic component?
2.2.3 2.2.3 Associated Tissues14
2.2.3.1 Associated Tissues

2.2.3.1.1 BONES

• Provide the framework and internal core structure for the attachment of muscles
• Bone is a living rigid tissue which forms the support structures for the rest of the body. The process
of bone formation is called ossication.
• The matrix of bone contains a dense arrangement of collagen bres together with mineral salts of
calcium, magnesium and phosphates.
• The calcium salts give bone its hardness and rigidity while collagen bres give bones its exibility and
strength.

2.2.3.1.1.1 FUNCTIONS OF BONE

• To serve as a rm support framework for the whole body.


• To protect such delicate structures as the brain and spinal cord
• To serve as levers, working with attached muscles to produce movement.
• To serve as a storehouse for calcium salts , which may be reabsorbed into the blood if there is
not enough calcium in the diet,
• To produce blood cells in the red marrow.
14 This content is available online at <http://cnx.org/content/m43144/1.1/>.

Available for free at Connexions <http://cnx.org/content/col11410/1.3>


91

Figure 2.24

2.2.3.1.1.2 MICROSCOPIC STRUCTURE OF A LONG BONE

• Numerous hollow tunnels called Haversian canals occur within the matrix of bone tissue and run
parallel with the length of the bone. Under the microscope they appear as black circles against a
lighter background.
• Each Haversian canal is surrounded by concentric rings of compact bone called lamellae
• Each of these layers contains a ring of uid-lled cavities called lacunae. Each of these lacuna will
contain a number of bone cells called osteocytes.
• The lacunae are linked to each other and to the Haversian canal by a system of very tiny interconnecting
canals called canaliculi. Strands of cytoplasm extend through these canals which supply the
osteocytes with oxygen and nutrients and remove waste products
• The Haversian canals, lacunae, osteocytes and canaliculi together form a unit called a Haversion
System and a number of these systems make up compact bone.

• Apart from osteocytes which are embedded in the lacunae of bone there are two other types of bone
cells
Osteoblasts : Bone forming cells. These cells allow the bone to change and remodel its shape as the
organism grows and responds to stresses. If a bone is broken or if strengthening is needed, bone cells lay
down new tissue and repair damaged tissue
Osteoclasts: Special bone cells for destroying and reabsorbing bone tissue.

2.2.3.1.2 CARTILAGE

2.2.3.1.2.1 Main features

• cartilage is a tough semi-transparent exible tissue


• it is enclosed by a brous capsule called the perichondrium
• consists of living cells called chondrocytes which secrete a rubbery protein matrix called chondrin

Available for free at Connexions <http://cnx.org/content/col11410/1.3>


92 CHAPTER 2. LIFE PROCESSES IN PLANTS AND ANIMALS

• chondrocytes occur in small uid-lled spaces called lacunae which are scattered throughout the
matrix.
• There are no blood vessels or nerves in the matrix.

2.2.3.1.2.2 Cartilage and Bone

Infant and young children do not have bones like those of adults. Their bones are made mostly of cartilage,
a rm elastic brous material.
As the individual grows and matures, the cartilage is gradually replaced by bone cells which deposit
crystals of calcium carbonate and calcium phosphate.
This process called ossication greatly increases the strength of the bone.
Bones usually continue to grow through adolescence. During this time a layer of cartilage still exists
between the head and shaft at either end of the bone. The growth of the bone does not interfere with the
way joints t together. Eventually once all the cartilage has become ossied bone growth will stop
2.2.3.1.2.3 TYPES OF CARTILAGE

Hyaline Cartilage:
Appearance: glass-like, bluish-white in colour, few bres present
Location:
• at the ends of bones as articular cartilage
• where the ribs are joined to the sternum
• forms rings in the trachea
• larynx and tip of nose
• as temporary cartilage in bones.
Functions:
• reduces friction at the joints.
• allows a degree of movement during breathing
• keeps the trachea open.
• Forms permanent structures
• Allows for bones to increase in length.
Fibrocartilage
Appearance: has numerous white collagen bres in the matrix.
Location:
as cartilaginous discs between the vertabrae

in the rim of sockets of ball and socket joints

between the pubic bones

Functions:
• act as shock absorbers
• make the cavity deeper without hampering movement
• allows for limited movement

Elastic cartilage
Appearance: has a network of yellow elastic bres in the matrix.
Location:
• in the pinna of the ear
Available for free at Connexions <http://cnx.org/content/col11410/1.3>
93
• in the epiglottis
Functions:
• maintains the shape of the ear but also allows for changes in shape.
• strengthens the epiglottis which prevents food from entering the trachea.

2.2.3.1.3 Ligaments

• Ligaments consist of white collagen bres and a network of yellow elastic bres.
• The collagen bres are less orderly and more randomly arranged than in tendons and ligaments have
varying amounts of elastic bres.
• Ligaments join bone to bone and they also control the degree of movement allowed between the two
bones. This is achieved by the amount of elasticity in a ligament i.e. a ligament will only stretch
enough to allow a particular movement to happen.
• The more elastic bres in the ligament the greater the articulation between two bones. Thus the
attachment of ligaments between bones keep the bones of a joint in position.
• By restricting bone movement ligaments will prevent any dislocation during normal actions.

2.2.3.1.4 Tendons

Attach muscles to bones and facilitate the various positions of the body related to movement and balance.
• Tendons consist of non elastic collagen bres only.
• These are densely packed, arranged in parallel bundles and are extremely strong, less exible and more
resistant to stress
• The bres give tendons a white shiny appearance.
• There is a minimal amount of matrix present.

2.2.4 2.2.4 Joints15


Joints
http://www.bbc.co.uk/science/humanbody/body/factles/joints/ball_and_socket_joi nt.shtml 16

A joint is formed when two or more bones come into contact


2.2.4.1 Types of Joints

Joints are divided into 3 groups according to their degree of movement.


1. Immovable joints such as the bones of the skull (known as sutures) which are fused.
2. Partly movable joints have cartilage between them which allows for a small degree of movement
e.g. between the vertebrae also called cartilaginous joints .
3. Synovial joints are freely movable and are divided into 4 groups
• Hinge joints - e.g elbow and knee joints which allow movement in one plane only
• Ball and Sock et joints - e.g. shoulder and hip joints allows free movement in almost all directions.
• Pivot joint - between atlas and axis vertebrae  allows for turning movement of head
• Gliding joints - between ankle bones and wrist bones  allows for rotational movements of hands and
feet
15 This content is available online at <http://cnx.org/content/m43161/1.1/>.
16 http://www.bbc.co.uk/science/humanbody/body/factles/joints/ball_and_socket_joint.shtml

Available for free at Connexions <http://cnx.org/content/col11410/1.3>


94 CHAPTER 2. LIFE PROCESSES IN PLANTS AND ANIMALS

2.2.4.2 Structure of synovial joint

• The joint is completely enclosed in a bag-like joint capsule forming a synovial cavity.
• The joint capsule is lined by a synovial membrane which secretes synovial uid lling the entire
cavity thereby reducing friction
• The ends of the bones are covered in hya l ine articular cartilage
• In addition to the joint capsule, other ligaments are present which attach bones to each other

Figure 2.25

SYNOVIAL JOINT

2.2.5 2.2.5 Human Locomotion17


Denition 2.1: Locomotion
= Movement or the ability to move from one place to another.
Denition 2.2: Human locomotion
= the ability youhave to move from one place to another ( walking from your house to
a friend's)
Harvard Outreach: Leg mechanics of playing basketball:
http://outreach.mcb.harvard.edu/animations_S09.htm 18

17 This content is available online at <http://cnx.org/content/m43153/1.1/>.


18 http://outreach.mcb.harvard.edu/animations_S09.htm

Available for free at Connexions <http://cnx.org/content/col11410/1.3>


95
2.2.5.1 What is used during locomotion?
2.2.5.1.1 1) Bones - body's supporting structure

• provide the framework


• provide internal core structure for the attachment of muscles.
• Protection of human organs
• Keeps body shape

2.2.5.1.2 2) Joints - place in your body where two bones are connected

Three types of joints in your body:


2.2.5.1.2.1 i) Fibrous joints

• join bones where no movement is allowed


• An example will be the bones of your cranium (the skull).

2.2.5.1.2.1.1 ii) Cartilaginous joints

• allows slight, restricted movement


• for example the discs between the vertebrae of the spine

2.2.5.1.2.1.2 iii) Synovial joints

• Allow free movement in one or more directions to the joints of the pelvic and pectoral girdles.
• These joints facilitate movements like standing, sitting, walking and running.

2.2.5.1.2.2 ii) Cartilaginous joints

• allows slight, restricted movement


• for example the discs between the vertebrae of the spine

2.2.5.1.2.2.1 iii) Synovial joints

• Allow free movement in one or more directions to the joints of the pelvic and pectoral girdles.
• These joints facilitate movements like standing, sitting, walking and running.

2.2.5.1.2.3 iii) Synovial joints

• Allow free movement in one or more directions to the joints of the pelvic and pectoral girdles.
• These joints facilitate movements like standing, sitting, walking and running.

2.2.5.1.3 3) Ligaments  connect bone and bone.

• Hold bone in place so that they work in a coordinated manner.

Available for free at Connexions <http://cnx.org/content/col11410/1.3>


96 CHAPTER 2. LIFE PROCESSES IN PLANTS AND ANIMALS

2.2.5.1.4 4) Tendons - connect muscles to bone.

• Attachment to the skeletal muscles move your bones


• Facilitate the various positions of the body related to movement and balance.

2.2.5.1.5 5) Antagonistic muscles

• Antagonistic = `opposite'
• Antagonistic movement of muscles

• at least two sets of muscles


• one set contracts and the other relaxes

• Contraction = stimulated muscle  becomes shorter and thicker


• Relaxation = muscle relaxes

2.2.5.1.5.1 Example: Biceps and triceps

•The biceps is an example of a exor muscle (muscle whose contraction shortens a body part)
•Whereas the triceps is an example of an extensor muscle (muscle whose contraction extends or
stretches a body part).
• Note that voluntary muscles are normally connected to at least two bones.

In the case of the biceps the two bones involved are the scapula and the humerus
• When the biceps muscle contracts only one of the bones moves ( in this case the radius). The point of
attachment to the movable bone is called the point of insertion and the biceps is attached to this
point by a single tendon. So when the biceps contracts the forearm is lifted or bent, decreasing the
angle between the forearm and humerus. and exing your arm, Thus the biceps is a exor muscle
• The biceps muscle gets its name from having two tendons attached to the scapula. The resistance. of
these two tendons prevents the contractile force of the biceps from moving the scapula and therefore
there is no movement of the bone..
• The point of attachment of a muscle to the immovable bone is called the point of origin.

Available for free at Connexions <http://cnx.org/content/col11410/1.3>


97

Figure 2.26

Figure 2.2.1: Illustration of the triceps (extensor muscle) and biceps muscles (exor mus-
cle). Found in http://commons.wikimedia.org/wiki/File:Anatomy_and_physiology_of_animals_Antagon
istic_muscles,_exion%26tension.jpg 19

Straightening of the forearm

• When the arm is bent the biceps cannot contract as it is already in a contracted state as muscles can
only cause movement by pulling as they contract not by pushing when they relax.
• Therefore the straightening of the arm is brought about by the contraction of the triceps muscle
which is an extensor muscle as it increases the angle between forearm and humerus
• The triceps has three points of origin, two on the humerus and one on the scapula, and a single point
of insertion on the ulna.
19 http://commons.wikimedia.org/wiki/File:Anatomy_and_physiology_of_animals_Antagonistic_muscles,_exion%26tension.jpg

Available for free at Connexions <http://cnx.org/content/col11410/1.3>


98 CHAPTER 2. LIFE PROCESSES IN PLANTS AND ANIMALS

2.2.5.1.5.1.1 title

Video
illustrating the mechanics of the antagonism within the biceps and triceps.
20
http://www.youtube.com/watch?v=T-ozRNVhGVg&feature=related
Antagonistic muscles:
http://www.botany.uwc.ac.za/Sci_Ed/grade10/manphys/skel_mus.htm 21

2.2.6 2.2.6 Muscles22


2.2.6.1 Muscles

Denition 2.3: Denition:


= Muscle is a contractile tissue type of animals
23 24

Khan video: Anatomy of a muscle cell


http://www.khanacademy.org/video/anatomy-of-a-muscle-cell?playlist=Biology 25

Khan video: Myosin and actin


http://www.khanacademy.org/video/myosin-and-actin?playlist=Biology 26

Khan video: Role of sarcoplasmic reticulum in muscle cell


http://www.khanacademy.org/video/role-of-the-sarcoplasmic-reticulum-in-muscle-
cells?playlist=Biology 27

20 http://www.youtube.com/watch?v=T-ozRNVhGVg&feature=related
21 http://www.botany.uwc.ac.za/Sci_Ed/grade10/manphys/skel_mus.htm
22 This content is available online at <http://cnx.org/content/m43159/1.1/>.
23 http://en.wikipedia.org/wiki/Muscle_contraction
24 http://en.wikipedia.org/wiki/Tissue_%28biology%29
25 http://www.khanacademy.org/video/anatomy-of-a-muscle-cell?playlist=Biology
26 http://www.khanacademy.org/video/myosin-and-actin?playlist=Biology
27 http://www.khanacademy.org/video/role-of-the-sarcoplasmic-reticulum-in-muscle-cells?playlist=Biology

Available for free at Connexions <http://cnx.org/content/col11410/1.3>


99
2.2.6.1.1 Three types of muscle

2.2.6.1.1.1 Smooth/ involuntary

• not by will- spontaneous


• unconscious routine tasks of the body:
· Food moving down the digestive system
· keeping the eyes in focus
· adjusting the diameter of blood vessels

2.2.6.1.1.1.1 Structure:

• spindle shaped cells with nucleus

Figure 2.27: Illustrates the structure of a smooth muscle

2.2.6.1.1.1.2 Functions:

Found in the walls of:


• blood vessels

Available for free at Connexions <http://cnx.org/content/col11410/1.3>


100 CHAPTER 2. LIFE PROCESSES IN PLANTS AND ANIMALS

• Uterus
• bladder
• Intestines

2.2.6.1.1.2 Cardiac muscle

• Responsible for your heart beat (muscle only found in the heart)
• Only found in the walls of the heart

2.2.6.1.1.2.1 Structure

• branched and contains intercalated disks


• Carry message in each cell for heart contraction

Figure 2.28: Illustrates the structure of the cardiac muscle

Available for free at Connexions <http://cnx.org/content/col11410/1.3>


101
2.2.6.1.1.3 Voluntary/skeletal

controlled by will
• Running
• Walking
• Skipping

2.2.6.1.1.3.1 Structure:

• The basic units of a muscle are called myobrils .


• These myobrils make up the muscle bre (large muscle cells).
• Numerous muscle bres (cells) are found in bundles .
• These bundles are surrounded by perimisium
· This is called fasciculus
• Numerous fasciculi are surrounded by epimysium
• This forms a muscle

Figure 2.29: Indicates the diering structural components of the voluntary muscle.

Available for free at Connexions <http://cnx.org/content/col11410/1.3>


102 CHAPTER 2. LIFE PROCESSES IN PLANTS AND ANIMALS

2.2.6.1.2 Muscle contraction

• Myobrils are responsible for the muscle contraction.


• Each myobril consists of units called sarcomeres (there are many sarcomeres in each myobril )
• Sarcomeres consist of thin actin laments and thick myosin laments.
• When muscle bres contract these laments slide across each other.
• The actin laments shorten, but the length of the myosin laments do not change.
• This causes the sarcomeres shorten,
· leading to the whole muscle to shorten
• ATP (energy) is a substance in the muscle bre that provides energy for the contracting actin lament.

IMAGE!!!Details on wish list


Video: Summary of the workings of the muscle
http://www.khanacademy.org/video/anatomy-of-a-muscle-cell?playlist=Biology 28

Muscle Exercise:
Column B

A) Attached to skeleton by tendons 1) Cardiac muscle


B) Seen in bundles 2) Blood vesels
C) They make up muscle bers 3) Muscles
D) Spindle shaped structure 4) movement
E) Causes the pumping action of the heart. 5) muscle bres
F) smooth muscles are found here 6) Fasciculus
G) specialized tissue 7) myobrils
H) contraction and relaxation 8) voluntary muscles
I) bundles surrounded by perimysium 9) epimysium
J) Numerous fasciculi are surrounded by 10) Involuntary muscle

Column B

A) Attached to skeleton by tendons 1) Cardiac muscle


B) Seen in bundles 2) Blood vesels
C) They make up muscle bers 3) Muscles
D) Spindle shaped structure 4) movement
E) Causes the pumping action of the heart. 5) muscle bres
28 http://www.khanacademy.org/video/anatomy-of-a-muscle-cell?playlist=Biology

Available for free at Connexions <http://cnx.org/content/col11410/1.3>


103

F) smooth muscles are found here 6) Fasciculus


G) specialized tissue 7) myobrils
H) contraction and relaxation 8) voluntary muscles
I) bundles surrounded by perimysium 9) epimysium
J) Numerous fasciculi are surrounded by 10) Involuntary muscle

Column B

A) Attached to skeleton by tendons 1) Cardiac muscle


B) Seen in bundles 2) Blood vesels
C) They make up muscle bers 3) Muscles
D) Spindle shaped structure 4) movement
E) Causes the pumping action of the heart. 5) muscle bres
F) smooth muscles are found here 6) Fasciculus
G) specialized tissue 7) myobrils
H) contraction and relaxation 8) voluntary muscles
I) bundles surrounded by perimysium 9) epimysium
J) Numerous fasciculi are surrounded by 10) Involuntary muscle

Column B

A) Attached to skeleton by tendons 1) Cardiac muscle


B) Seen in bundles 2) Blood vesels
C) They make up muscle bers 3) Muscles
D) Spindle shaped structure 4) movement
E) Causes the pumping action of the heart. 5) muscle bres
F) smooth muscles are found here 6) Fasciculus
G) specialized tissue 7) myobrils

Available for free at Connexions <http://cnx.org/content/col11410/1.3>


104 CHAPTER 2. LIFE PROCESSES IN PLANTS AND ANIMALS

H) contraction and relaxation 8) voluntary muscles


I) bundles surrounded by perimysium 9) epimysium
J) Numerous fasciculi are surrounded by 10) Involuntary muscle

Column B

A) Attached to skeleton by tendons 1) Cardiac muscle


B) Seen in bundles 2) Blood vesels
C) They make up muscle bers 3) Muscles
D) Spindle shaped structure 4) movement
E) Causes the pumping action of the heart. 5) muscle bres
F) smooth muscles are found here 6) Fasciculus
G) specialized tissue 7) myobrils
H) contraction and relaxation 8) voluntary muscles
I) bundles surrounded by perimysium 9) epimysium
J) Numerous fasciculi are surrounded by 10) Involuntary muscle

Column A Column B

A) Attached to skeleton by tendons 1) Cardiac muscle


B) Seen in bundles 2) Blood vesels
C) They make up muscle bers 3) Muscles
D) Spindle shaped structure 4) movement
E) Causes the pumping action of the heart. 5) muscle bres
F) smooth muscles are found here 6) Fasciculus
G) specialized tissue 7) myobrils
continued on next page

Available for free at Connexions <http://cnx.org/content/col11410/1.3>


105
H) contraction and relaxation 8) voluntary muscles
I) bundles surrounded by perimysium 9) epimysium
J) Numerous fasciculi are surrounded by 10) Involuntary muscle
Table 2.1

Choose the correct answer for column A from column B (only one correct answer per question)
2.2.6.1.2.1 Activity: Classifying Muscle Types

• Use the following story to classify the dierent muscle types. Use a coloured pen or highlighter to
underline or highlight the actions in the story which require the dierent types of muscles
• Suggested colours to use: Pink = Cardiac Muslces; Blue =Voluntary ; Yellow = Involuntary
• Then draw each structure in the space provided
Story:
BEEP BEEP BEEP!!!
6 a.m on a Monday morning Tsholo's alarm goes o.
She jumps out of bed and walks to the toilet to relieve her bladder. Tsholo is very excited for the day
and skips back to her room to get dressed and pack her school bag for the new week. In the kitchen mom
has prepared Tsholo's favourite porridge Mielie Meal *. Tsholo eats het porridge with great pleasure. After
breakfast, she brushes het teeth and skips to the car where she waits for mom to unlock the doors.
At school Tsholo runs to her friends in total excitement to tell them about her visit to her grandmother.
While chatting she sees Tom - the boy she likes a lot! He looks her way and Tsholo's starts blushing. Her
heart rate increases and her palms become sweaty.
The bell rings.
Tsholo and her friends walk to class, giggling and talking. Her heart rate slowly returns back to normal
. The week has begun. . .
Draw and label the three dierent muscle types
Cardiac:
Voluntary:
Involuntary:
2.2.6.1.3 Interesting facts  Skeleton

• A baby is born with more bones (360 bones) than an adult (average 206 bones). Bones making up the
skull and the spine fuse together as the body grows making it less.
• The femur/thigh bone is the largest in your body. The femur is approximately one quarter of a person's
overall height.
• Strengthen your skeleton by drinking milk and eating leafy greens (such as brussels sprouts and kale).
They contain calcium which keeps bones healthy and strong.
• A broken bone produces many new cells to rebuild the bone. These cells cover both ends of the broken
part of the bone and close up the break.
• Your bones are alive! In your body bones have their own nerves and blood vessels.
• Your bone is 50% water and 50% solid material
• You have 14 bones are in your face.
• There are 8 bones in each of your wrists
• You have 23 bones in each foot ( this includes the ankle)
• Your skull is made up of fused bones which acts like a hard protective helmet for your brain.

Available for free at Connexions <http://cnx.org/content/col11410/1.3>


106 CHAPTER 2. LIFE PROCESSES IN PLANTS AND ANIMALS

2.2.7 2.2.7 Diseases29


2.2.7.1 Diseases

Bones like any other part of your body require proper care and can get diseased in older people and mal-
nourished children. Common bone problems include Rickets, Osteoporosis and arthritis.
2.2.7.2 Rickets

• Childhood disease
• Delay in mineral (calcium phosphate) deposition to harden bones  soft bones
• causing characteristic bowed legs
• Caused by Vitamin D deciency in the diet or when the body does not receive enough sunlight as the
body requires sunlight to make Vitamin D!
An X-ray picture of an adult who suered from rickets as a child (
http://depts.washington.edu/bonebio/ASBMRed/diseases/rickets/rickets.html ) 30

2.2.7.3 Osteoporosis

• Greek for bones with holes.


• Osteo bone Porosis  Holes/Passages.
• Common in older people.
• When there is a shortage of calcium in the body or when a bone is inactive e.g leg in plaster or
imobilised, calcium is withdrawn from the leg for use in other parts of the body.. This together with
a decrease in bone protein, will result in an increased breakdown of bone tissue without an increase in
deposit of new bone by osteoblasts
and the development of holes in the bone ( hence the name of the disease
• Decrease in bone density which makes it weak and prone to fractures
• The most typical fractures are of the spine, wrist and hip.

2.2.7.4 Arthritis:

• The most common form of arthritis aects the bone and is known as Osteoarthritis.
• Commonly known as wear-and-tear arthritis.
• Degradation of the soft cartilage in joints between bones causes the end of the bones to rub against
each other which is painful!
Visualizing osteoarthritis: http://www.youtube.com/watch?v=a1d8qK4BEx0&feature=related 31

Further reading on the web: http://depts.washington.edu/bonebio/ASBMRed/diseases.html 32

Types of arthritis: http://www.vimovo.com/types-of-arthritis.aspx 33

29 This content is available online at <http://cnx.org/content/m43104/1.1/>.


30 http://depts.washington.edu/bonebio/ASBMRed/diseases/rickets/rickets.html
31 http://www.youtube.com/watch?v=a1d8qK4BEx0&feature=related
32 http://depts.washington.edu/bonebio/ASBMRed/diseases.html
33 http://www.vimovo.com/types-of-arthritis.aspx

Available for free at Connexions <http://cnx.org/content/col11410/1.3>


107
2.2.8 2.2.8 Summary34
2.2.8.1 Summary

• There are 3 types of skeletons:


1. Hydrostatic skeleton
2. Endoskeleton
3. Exoskeleton
• When animals moved from water to land, there was a need for the development of strong limbs and a
skeleton to provide support to the bodies, which before water had provided
• Humans have an endoskeleton consisting of:
1. Axial skeleton (cranium, facial bones, foramen magnum, palate and jaws, vertebral column, rib cage
and breastbone(sternum)
2. Appendicular skeleton (pectoral girdle with arms and pelvic girdle with legs)
• Functions of the human skeleton are:
1. Movement
2. Protection
3. Support
4. Storage of minerals
5. Hearing
• There tissues associated with the human skeleton are bone, cartilage, tendons and ligaments
• Joints
• A joint is formed when two or more bones come into contact
• There are three types of joints
1. Immovable joints
2. Partly movable joints
3. Synovial joints (Hinge joints, ball and socket joints, pivot joints, gliding joints)
• Human locomotion requires the use and coordination of bones, joints, ligaments, tendons and antago-
nistic muscles
• Muscles
- There are three types:
1) Smooth/involuntary
2) Skeletal/voluntary
3) Cardiac muscle
• Myobrils are responsible for muscle contraction

• There are many diseases that aect the skeleton, such as rickets, osteoporosis and arthritis
34 This content is available online at <http://cnx.org/content/m43078/1.1/>.

Available for free at Connexions <http://cnx.org/content/col11410/1.3>


108 CHAPTER 2. LIFE PROCESSES IN PLANTS AND ANIMALS

2.2.9 2.2.9 Exercises35

Figure 2.30

Figure 2.31

2.2.9.1 Exercises

2.2.9.1.1 Question 1

1. Name three types of skeletons and provide one advantage and one disadvantage of each. (9)
2. State where the Haversian canal is located and state its function. (3)
3. Name four functions of bone tissue. (4)
4. Tabulate two dierences between tendons and ligaments. (5)

2.2.9.1.2 Question 2

1. Supply the biological term for each of these bones:


a. thigh bone
b. knee cap
c. shin bone
d. ankle bones
e. heel bone
f. upper arm bone
g. wrist bones
h. breast bone (8)
2. Name FOUR functions of the human skeleton. (4)
3. State the number of:
a. bones in the human vertebral column. (1)
b. pairs of true ribs (1)
c. lumbar vertebrae (1)
35 This content is available online at <http://cnx.org/content/m43169/1.1/>.

Available for free at Connexions <http://cnx.org/content/col11410/1.3>


109
2.2.9.1.3 Question 3

Study the following diagrams showing the main bones of the pectoral girdle and the human arm (forelimb)
and answer the questions that follow:

Figure 2.32

1. Identify bone X. (1)


2. Parts of some of these bones meet at certain joints. By using the letters
3. (A  H) only, state which parts of the bones form the shoulder joint. (2)
4. Name the type of synovial joint that is located at the following parts of the body:
a. At the elbow (1)
b. Where the lower limb joins the pelvis (1)
c. In the wrist (1)

2.2.9.1.4 Question 4

The diagram below shows the legs of an athlete while he is waiting for a race to start. The letters A to F
show some of the muscles as well as joints that will be used during the race.

Available for free at Connexions <http://cnx.org/content/col11410/1.3>


110 CHAPTER 2. LIFE PROCESSES IN PLANTS AND ANIMALS

Figure 2.33

1. When the Starter's gun is red, the athlete's right leg will straighten, pushing the athlete upwards and
forwards. Which of the letters (A to F) indicate muscles that will:
a. Relax (2)
b. Contract (2)
2. The leg shown in the diagram has dierent types of joints. Which of the following letters (A to F)
indicates:
a. A hinge joint (1)
b. A ball and socket joint (1)

2.2.9.1.5 Question 5

Skeleton and Movement  True or False?


1. The skeleton's role is to provide support, protection and capacity for movement.
2. The skeleton is divided into the axial and appendicular skeleton.
3. The axial skeleton consists of the pectoral and pelvic girdles and their attached limbs.
4. Carpals are found in the ankles and tarsals in the wrists.
5. The biceps muscle raises the arm while the triceps lowers it in an antagonistic pair.
6. Synovial liquid lubricates joints and keeps them friction free.
7. Bone joints in the cranium are examples of brous joints.
Available for free at Connexions <http://cnx.org/content/col11410/1.3>
111
8. The neck contains 7 lumbar vertebrae.
9. Tendons join muscles to bone and are elastic while ligaments join bone to bone and are non-elastic.
10. Bone is composed of exible minerals such as Calcium and Phosphate with rigid bres of Collagen.
11. Osteocyte is another word for bone cell. (11)

2.2.9.1.6 Question 6

Compare the biceps and triceps muscles with respect to:


1. Point of origin (4)
2. Point of insertion (2)
3. Function (2)

2.2.9.1.7 ANSWERS

2.2.9.1.7.1 Question 1

1)

SKELETON ADVANTAGE DISADVANTAGE


Hydrostatic - Allows animal to be very exi- - Not very strong  easily dam-
ble  moves easily; not restricted aged or lost if the enclosed cav-
in terms of possible movements.- ity around it is pierced.- Gener-
Give support without adding ally not suitable to terrestrial an-
much weight.- Allows rapid dif- imals and oers not protections
fusion of gases through the body against dehydration.- Limits the
wall, so a transport system is of- size of the animal  large animals
ten unnecessary, e.g. jellysh. would not be feasible.
Exoskeleton - Very strong and provides good - Heavy, so it prevents the an-
protection against damage.- Can imal getting very large. (small
be present on great variety of animals are easy prey)- Necessi-
colours to provide protection via tates moulting, making the ani-
camouage.- Oer good protec- mal very vulnerable.- Movement
tion against dehydration. is only possible at thinner joints,
but these are more vulnerable
than thick areas.
continued on next page

Available for free at Connexions <http://cnx.org/content/col11410/1.3>


112 CHAPTER 2. LIFE PROCESSES IN PLANTS AND ANIMALS

Endoskeleton - Bone is very hard, so gives ex- - Broken bones take a long time
cellent protection of vital organs to heal and are painful.- Bones in-
e.g. brain.- Bone marrow inside side the body oer no protection
bones forms blood cells.- Allows to some soft tissues, e.g. intes-
animal to become bigger  large tine.
animals have fewer enemies
Table 2.2

(One mark per skeleton, one for an advantage and disadvantage each) [9]
2) Haversian canal is in the centre of a Haversian systemΠ in compact bone. It contains a nerve to carry
impulsesΠ, blood vessels to transport gasesΠ, food and wastes and a lymph duct to drain tissue uid. (one
for where it is, + 2 for any two functions of parts in it [3]
3) Any four of the following:
• Is hard to support the body part and protect vital organs
• Forms a store of calcium and phosphorus in the body
• Can undergo mitosis to repair damage, e.g. breaks
• Grows to make the body bigger as we age
• Provides rm attachment place for muscles
• Protects bone marrow that produces blood cells
• Allows fast diusion to and from osteoblasts via liquid-lled canaliculi [4]

4)

TENDONS LIGAMENTS
Attach muscles to bonesContain more collagen Attach bones to other bones or to brocartilage-
compared to elastin, so they are very inelasticFibres Contain less collagen compared to elastin, so they
in tendons are all along the long axis for strength have slight elasticityThe bres in ligaments are wo-
ven together, not arranged longitudinally
Table 2.3

(One for the table and one each for two dierences between them) [5]
2.2.9.1.7.2 Question 2

1)
1. femur
2. patella
3. tibia
4. tarsals
5. calcanum
6. humerus
7. carpals
8. sternum [8]
2)
• Protection of vital organs, e.g. brain, heart
• Attachment place for muscles and resistance for muscle contraction
Available for free at Connexions <http://cnx.org/content/col11410/1.3>
113
• Gives shape to the body, e.g. facial features
• Supports body parts and keeps us upright
• Bones form levers for locomotion
• Bone marrow forms blood cells
• Middle ears bones are vital for hearing [4]
3)
1. 33
2. 7
3. 5 [3]
Scapula [1]
Bone end B joins to part F
hinge joint
ball and socket joint
gliding joint [3]
2.2.9.1.7.3 Question 4

a) B and F [2]
b) C and E [2]
2a) D
b) A [2]
2.2.9.1.7.4 Question 5

1. True
2. True
3. False (should be appendicular skeleton)
4. False (other way round  carpals are in the wrist, tarsals in the ankle)
5. False (the biceps and triceps respectively raise and lower the LOWER arm or forearm, not whole arm)
6. True (not entirely `friction-free', but close)
7. True
8. False (neck has cervical vertebrae)
9. False (joining functions are correct, but tendons are inelastic and ligaments are more elastic)
10. False (Ca and P are inexible minerals and collagen is a exible protein)
11. True (but bone cells can also be called osteoblasts) [11]

2.2.9.1.7.5 Question 3

1. Biceps and triceps BOTH originate at the shoulder (remember the origin is the part that DOES NOT
MOVE when the muscle contracts). Biceps has one tendon at shoulder, triceps has two (4)
2. Biceps is inserted on the radius and triceps on the ulna (remember the insertion is the part that
MOVES when the muscle contracts) (2)
3. The biceps contracts to bend the arm at the elbow, the triceps contracts to straighten the arm at the
elbow. (2)

Available for free at Connexions <http://cnx.org/content/col11410/1.3>


114 CHAPTER 2. LIFE PROCESSES IN PLANTS AND ANIMALS

2.3 Transport systems in mammals (humans)


2.3.1 2.3.1 Blood circulatory system36
2.3.1.1 Useful links

Cardiovascular system: http://www.biologyinmotion.com/cardio/index.html 37

Khan: Red blood cells http://www.khanacademy.org/video/red-blood-cells?playlist=Biology 38

Khan: Circulatory system and the heart http://www.khanacademy.org/video/circulatory-system-and-the-


heart?playlist=Biology 39

YouTube video: Circulatory system http://www.youtube.com/watch?v=D3ZDJgFDdk0 40

Blood ow animation http://health.howstuworks.com/human-body/systems/circulatory/adam-200078.htm


41

Blood Flow animation http://www.sumanasinc.com/webcontent/animations/content/human_heart.html


42

The heart and circulation (interactive activity): http://www.kett6.net/adulteducation/heartanimations.html


43

Circulation animation: http://www.bbc.co.uk/schools/gcsebitesize/pe/appliedanatomy/0_anatomy_circulator


ysys_rev1.shtml 44

2.3.1.2 Overview

All living cells require nutrients and oxygen to survive. Cells produce metabolic waste, which must be re-
moved and excreted. The circulatory system is responsible from providing nutrients and removing metabolic
waste.
Unicellular organisms have a simple system to allow for this and it is by diusion where substances
move from a high concentration to a low concentration.
Most invertebrates like a grasshopper have an open circulatory system , where blood
(haemolymph) bathes the body organs.
By comparison, mammals have a closed circulatory system since blood is contained within blood
vessels.
2.3.1.3 Pulmonary and Systemic circulatory systems

Open circulatory system


blood is pumped into a heamocoel (an open space or cavity) that surrounds to organs. Muscle movement
also helps to pump then blood. Blood diuses back the heart. Blood movement is sluggish. There is no
dierence between the blood and the interstitial uid. Interstitial uid is the uid that surrounds the cells.
Blood is not contained within capillaries.
Closed circulatory system
blood is pumped from the heart through arteries and returns to the heart via veins. Blood never leaves
the vascular system (arteries, veins and capillaries). Nutrients, water and metabolic waste diuses out of
the vascular system and into the interstitial uid. Interstitial uid and blood are separated, by the vascular
system. Interstitial uid returns to circulation through the lymphatic system.
36 This content is available online at <http://cnx.org/content/m43150/1.1/>.
37 http://www.biologyinmotion.com/cardio/index.html
38 http://www.khanacademy.org/video/red-blood-cells?playlist=Biology
39 http://www.khanacademy.org/video/circulatory-system-and-the-heart?playlist=Biology
40 http://www.youtube.com/watch?v=D3ZDJgFDdk0
41 http://health.howstuworks.com/human-body/systems/circulatory/adam-200078.htm
42 http://www.sumanasinc.com/webcontent/animations/content/human_heart.html
43 http://www.kett6.net/adulteducation/heartanimations.html
44 http://www.bbc.co.uk/schools/gcsebitesize/pe/appliedanatomy/0_anatomy_circulatorysys_rev1.shtml

Available for free at Connexions <http://cnx.org/content/col11410/1.3>


115
2.3.1.3.1 The Human Circulatory System

All mammals have a closed blood circulatory system - blood always ows inside blood vessels.
A double circulatory system = blood passes through the heart twice:
1. Pulmonary circulation: the blood is pumped from the heart to the lungs to oxygenate the blood
and then back to the heart.
2. Systemic circulation (to all the systems): the blood is pumped from the heart to all parts of
the body and back to the heart again.
3. Coronary circulation : is a circulatory system that supplies the heart muscle with the blood it
required in order to function.
Very simple simulation of blood ow through the systemic and pulmonary circulatory systems. The illus-
tration shows each of these circulatory systems to be separate loops leaving from one side of the heart and
returning to the other.
http :// www .
45 46 47 48
biologyinmotion . 49
com / cardio / index .
50 51 52 53 54 55

56
html 57

Figure: Simplied Diagrammatic sketch of the entire circulatory system. Blood ows to every inch of
the body, even to the tips of the ngers and toes. Lungs provide oxygen to the blood. The digestive system
supplies nutrients. The kidneys lter the blood.
45 http://www.biologyinmotion.com/cardio/index.html
46 http://www.biologyinmotion.com/cardio/index.html
47 http://www.biologyinmotion.com/cardio/index.html
48 http://www.biologyinmotion.com/cardio/index.html
49 http://www.biologyinmotion.com/cardio/index.html
50 http://www.biologyinmotion.com/cardio/index.html
51 http://www.biologyinmotion.com/cardio/index.html
52 http://www.biologyinmotion.com/cardio/index.html
53 http://www.biologyinmotion.com/cardio/index.html
54 http://www.biologyinmotion.com/cardio/index.html
55 http://www.biologyinmotion.com/cardio/index.html
56 http://www.biologyinmotion.com/cardio/index.html
57 http://www.biologyinmotion.com/cardio/index.html

Available for free at Connexions <http://cnx.org/content/col11410/1.3>


116 CHAPTER 2. LIFE PROCESSES IN PLANTS AND ANIMALS

2.3.1.4 The Heart and Associated Blood Vessels

Figure 2.34

Available for free at Connexions <http://cnx.org/content/col11410/1.3>


117
Figure 1 : Heart overlayed on a body so show the location of the heart within the chest.
http://en.wikipedia.org/wiki/File:Surface_anatomy_of_the_heart.png 58

• The heart is situated in your thorax just behind your breastbone and is about the size of your st.
• It is a large muscle that pumps through repeated rhythmic contractions and therefore requires lots of
nutrients and oxygen.
• On the surface of the heart are coronary arteries that are arteries that branch o the aorta and
supply the heart with oxygen and nutrients.
• The heart is made up of 4 chambers and divided by a septum into a right and left half.
• The right half of the heart pumps deoxygenated blood up into the pulmonary artery, towards the
lungs (pulmonary circulation), where it is oxygenated.
• Oxygenated blood returns from the lungs via the pulmonary veins and enters the left side of the heart.
• The left side of the heart then pumps oxygenated blood up through the aorta, and into the general
circulation (systemic circulation) and the oxygen is consumed by the body.
• Deoxygenated blood returns to the right side of the heart via the inferior vena cava which drains blood
from below the heart and superior vena cava, which brings blood from the head and arms.
• The human circulatory system is a double circulatory system, because blood travels to the heart twice
during circulation, once before going to the lungs and once before circulating throughout the body.
• Blood only ows in one direction, through the circulatory system.
• All vessels that ow A way from the heart are called A rteries.
• All blood vessels entering the heart are called V eins.
• The terms artery and vein are not determined by what the vessel transports (oxygenated blood or
deoxygenated) but by whether the vessel ows to or from the heart.
58 http://en.wikipedia.org/wiki/File:Surface_anatomy_of_the_heart.png

Available for free at Connexions <http://cnx.org/content/col11410/1.3>


118 CHAPTER 2. LIFE PROCESSES IN PLANTS AND ANIMALS

Figure 2.35

Figure 2 : General structure of the heart and associated blood vessels


(http://en.wikipedia.org/wiki/File:Anatomy_Heart_English_Tiesworks.jpg)
2.3.1.5 internal structure of the heart

• The heart is made up of 4 chambers. There are 2 atria at the top of the heart which receives
blood and 2 ventricles at the bottom of the heart which pumps blood out of the heart.
• The septum divides the left and right side of the heart.
• The valves of the heart ensure that blood only ows one way through the heart.

The tricuspid valve is found between the right atrium and the right ventricle.
The mitral valve is found between the left atrium and the left ventricle.
Strong tendinous chords attached to valves prevent them from turning inside out when they close.
The semi-lunar valves are located at the bottom of the aorta and pulmonary artery.

Available for free at Connexions <http://cnx.org/content/col11410/1.3>


119

Figure 2.36: Internal structure of the heart

Interesting facts : Humans, birds, and mammals have a four-chambered heart. Fish have a two-
chambered heart, one atrium and one ventricle. Amphibians have a three- chambered heart with two atria
and one ventricle. The advantage of a four chambered heart is that there is no mixture of the oxygenated
and deoxygenated blood.

Available for free at Connexions <http://cnx.org/content/col11410/1.3>


120 CHAPTER 2. LIFE PROCESSES IN PLANTS AND ANIMALS

Figure 2.37

Available for free at Connexions <http://cnx.org/content/col11410/1.3>

Figure 2.38
121

Figure 2.39

continued on next page

Available for free at Connexions <http://cnx.org/content/col11410/1.3>


122 CHAPTER 2. LIFE PROCESSES IN PLANTS AND ANIMALS

Figure 2.40

continued on next page

Available for free at Connexions <http://cnx.org/content/col11410/1.3>


123
Figure 4. The relationship of the heart and circulatory system to major visceral organs.

Table 2.4

59
The circulatory song http://www.youtube.com/watch?v=q0s-1MC1hcE&NR=1

2.3.1.6 The Cardiac Cycle

• The human heart will undergo over 3 billion contraction cycles, as shown in Figure 5, during a normal
lifetime.
• The heart beats in a rhythmic cycle. A complete cardiac cycle is one round of the heart pumping
blood.
• The top half of the heart works as one unit.
• The bottom half of the heart works as one unit.
• The sino-atrial node (pacemaker) starts and regulates the process.
• The cardiac cycle consists of two parts: systole (contraction of the heart muscle) and diastole
(relaxation of the heart muscle).
Atrial systole (0.1s)

• Atria contract simultaneously, pushing blood into the ventricles


• Ventricles are relaxed
• Atrio ventricular valves open
• Semi lunar valves close
Ventricular systole (0.3s)

• Atria relax
• Ventricles contract simultaneously
• Blood enters aorta and pulmonary artery
• Semi lunar valves open
• Atrio ventricular valves close
Diastole (0.4s)

• Atria and ventricles relax


• Atrio ventricular valves open
• Blood enters atria and ventricles
• Semi lunar valves close
• Back ow in arteries prevented
The heart beat can be heard as a sound that the valves make when they close. The `lub' sound is made
when the atrio ventricular valves close and the `dub' sound is made when the semi lunar valves close.
59 http://www.youtube.com/watch?v=q0s-1MC1hcE&NR=1

Available for free at Connexions <http://cnx.org/content/col11410/1.3>


124 CHAPTER 2. LIFE PROCESSES IN PLANTS AND ANIMALS

Figure 2.41

Figure 5
from mindset  (please check permission from this, found it in Biology 6th edition Campbell and Reece)
Cardiac Cycle: ow of blood through the heart
Excellent simple video illustrating the heart cycle.
60
http://www.youtube.com/watch?v=D3ZDJgFDdk0

2.3.1.6.1 Blood Pressure

• The blood pressure is produced by the left ventricle contractions.


• The rhythm of ventricle diastole, often just referred to as diastole, causes the pulse, which can be felt
by holding two nders to the side of the throat.
60 http://www.youtube.com/watch?v=D3ZDJgFDdk0

Available for free at Connexions <http://cnx.org/content/col11410/1.3>


125
• Blood pressure oscillates with the contraction of the left ventricle.
Ideal blood pressure for an adult is:
Systolic pressure: 120 mm HG
Diastolic blood pressure: 80 mm HG
A usual rule is that systolic pressure should be 100 plus your age but never more than 140 and diastolic
pressure should not be over 90.

Figure 2.42

Table 2.5

Figure 6 The cardiac cycle. Image from Purves et al., Life: The Science of Biology , 4th Edition, by
Sinauer Associates ( www.sinauer.com ) and WH Freeman ( www.whfreeman.com ),(please get
61 62

permission)
Normal Heart Sounds
http://upload.wikimedia.org/wikipedia/commons/7/72/HROgg.ogg 63

61 http://www.sinauer.com/
62 http://www.whfreeman.com/
63 http://upload.wikimedia.org/wikipedia/commons/7/72/HROgg.ogg

Available for free at Connexions <http://cnx.org/content/col11410/1.3>


126 CHAPTER 2. LIFE PROCESSES IN PLANTS AND ANIMALS

2.3.1.7

2.3.1.8 Lung and pulmonary system

Khan Academy video on the pulmonary system. Overview on breathing.


http://www.khanacademy.org/video/the-lungs-and-pulmonary-system?playlist=Biology 64

• The lungs serve as the air-blood interface.


• Blood from the lungs is pumped into the pulmonary arteries.
• From the pulmonary arteries the vascular system branches into smaller and smaller vessels until the
blood is owing through thin pulmonary capillaries.
• These capillaries surround the alveoli in the lungs.
• At this point there are only two layers of cells separating the blood from the air.
• Carbon dioxide in deoxygenated blood diused out of the blood.
• Oxygen in the lungs diuse in to the blood oxygenating it Oxygen is absorbed.
• Oxygenated blood then returns to the heart vial the pulmonary veins.

Figure : Details arteries and veins connecting the heart to the lungs Red blood has been oxygenated, blue
blood is deoxygenated. . (Wikipedia - http://en.wikipedia.org/wiki/File:Illu_pulmonary_circuit.jpg)

Figure 2.43

64 http://www.khanacademy.org/video/the-lungs-and-pulmonary-system?playlist=Biology

Available for free at Connexions <http://cnx.org/content/col11410/1.3>


127

Figure 2.44

Figure : Very detailed image of the lungs, it is not necessary or required


to know all this detail but this is a fantastic image of the lungs  wikipedia
(http://en.wikipedia.org/wiki/File:Respiratory_system_complete_en.svg).

Available for free at Connexions <http://cnx.org/content/col11410/1.3>


128 CHAPTER 2. LIFE PROCESSES IN PLANTS AND ANIMALS

2.3.1.9 Major organs and systemic system: associated major blood vessels the brain, small

intestines, liver, kidney.

Figure : Detailed illustration of the systemic circulation. http://en.wikipedia.org/wiki/File:Circulatory_System_en.svg


65

• All the organs of the body are supplied by blood.


• Each has an artery supplying the organ with blood from the heart, and veins returning blood to the
heart.
• Arteries and veins have been named according to the organ which they supply blood to.
The circulatory system forms a closed system.
• Nutrients enter the circulatory system from the digestive system.
• These nutrients rst move to the liver via the hepatic portal vein, the liver then controls the nutrient
composition of the blood.
• Blood passes from the liver to the heart through the hepatic vein.
• Nutrients are then circulated throughout the body.
• Cells consume the nutrients in the blood and produce metabolic waste. T
• his metabolic waste is circulated in the blood, if it remains in the blood the blood would eventually
become toxic.
• The kidneys are supplied with blood via the renal arteries and they remove metabolic waste from the
blood, passing it to urine.
• The Brain is supplied with blood via the carotid arteries and the vertebral arteries. The blood is
drained via the jugular veins. The brain is supplied with 15% of the total amount of blood pumped
by the heart.

2.3.1.10 Mechanisms for controlling cardiac cycle and heart rate (pulse)

• The cardiac cycle is controlled by nerve bers extending from nodes of nerve bundles through the heart
muscle.
65 http://en.wikipedia.org/wiki/File:Circulatory_System_en.svg

Available for free at Connexions <http://cnx.org/content/col11410/1.3>


129
• An electrical signal is triggered in the node.
• The electrical signal then spreads through the bers and causes the heart muscle to contract.

2.3.1.10.1 There are two nodes:

1. The sinoatrial node (SA), which initiates the heart cycle. Electrical signals spread from the SA
across the atria causing it to contact.
2. The electrical signal also reaches the Atrioventricular node (AV) . Here the signal pauses, before
spreading through the ventricles causing them to contract.

• The SA is able to initiate the electrical signal without any stimulation for the nervous system, but it
can be controlled by the nervous system.

• The brain does not need to tell the heart to beat; it is able to beat on its own.

• The brain can make the heart rate increase, when for instance you are scared or are running.
• Hormones are also able to increase the heart rate.
Simple simulation of how electrical activity spreads over the heart.
Link : http://en.wikipedia.org/wiki/File:ECG_Principle_fast.gif 66

Measuring pulse rate: http://www.nlm.nih.gov/medlineplus/ency/article/003399.htm 67

2.3.1.10.2 How the Nodes cause contraction

• Human heartbeats originate from the sinoatrial node (SA node) near the right atrium.
• Modied muscle cells contract, sending a signal to other muscle cells in the heart to contract.
• The signal spreads to the atrioventricular node (AV node).
• Signals carried from the AV node, slightly delayed, through bundle of His bers and Purkinjie bers
cause the ventricles to contract simultaneously. Figure 13 illustrates several aspects of this.

66 http://en.wikipedia.org/wiki/File:ECG_Principle_fast.gif
67 http://www.nlm.nih.gov/medlineplus/ency/article/003399.htm

Available for free at Connexions <http://cnx.org/content/col11410/1.3>


130 CHAPTER 2. LIFE PROCESSES IN PLANTS AND ANIMALS

Figure 13. The contraction of the heart and the action of the nerve nodes located on the heart. Images
from Purves et al., Life: The Science of Biology , 4th Edition, by Sinauer Associates ( www.sinauer.com
68
) and WH Freeman ( www.whfreeman.com ), (please get permission)
69

Figure 2.45

Available for free at Connexions <http://cnx.org/content/col11410/1.3>


131

Figure 2.47

Table 2.6

2.3.1.10.3 Electrical activity

• The electrical activity in the heart is so strong that is can be measured from the surface of the body
as an electrocardiaogram (ECG).
• A normal heart has a very regular rhythm.
• An abnormal heart may have an arrhythmia, or abnormal rhythm as shown in the gures.

Figure 15. Normal cardiac pattern (top) and some abnormal patterns (bottom). Images from Purves et
al., Life: The Science of Biology , 4th Edition, by Sinauer Associates ( www.sinauer.com ) and WH 70

Freeman ( www.whfreeman.com ), (please contact for permission).


71

continued on next page

68 http://www.sinauer.com/
69 http://www.whfreeman.com/

Available for free at Connexions <http://cnx.org/content/col11410/1.3>


132 CHAPTER 2. LIFE PROCESSES IN PLANTS AND ANIMALS

Figure 2.48

Available for free at Connexions <http://cnx.org/content/col11410/1.3>

Figure 2.49
133
Table 2.7

Investigation: Heart Health and Measuring Heart rate


Part 1: Investigating your cardiovascular tness
Aim :To investigate your heart rate before, during and after strenuous aerobic exercise.
Method :
1. Work in pairs on the eld and ensure you have a stop watch.
2. One partner performs the experiment and the other records the results. Partners then swap roles.
3. Take the resting pulse rate before exercising.
4. One partner runs quickly around the eld twice.
5. Immediately after the run take his pulse.
6. Continue to take his pulse every minute for 5 minutes.
7. Record the results and plot a graph using the data pertaining to you.
Results : Record results in a table like the one indicated below

TIME HEART RATE (BEATS PER MINUTE)


Before exercise (resting)
0 min (immediately after exercise)
1 min (after exercise)
2 min
3 min
4 min
5 min
Table 2.8

Draw a line graph to illustrate your results on the following axis (show the resting pulse rate as a separate
dotted line on the axis).
Mark allocation: heading [U+F0FC][U+F0FC]x-axis scale [U+F0FC]x-axis label [U+F0FC]
y-axis scale [U+F0FC]y-axis label [U+F0FC]plotting graph [U+F0FC][U+F0FC][U+F0FC]
neat and done in pencil [U+F0FC]
Questions:
1.Write a hypothesis for this investigation.
2.Write down the independent variable.
3.Write down the dependent variable.
4.Name ONE factor that must be kept constant during this investigation.
5.Write down TWO ways in which the accuracy of this investigation can be
improved.
6.What conclusions can be made about your cardiovascular tness?
7.Explain why the heart rate increases during exercise?
Part 2: Investigating your family's heart health:
Instructions:
1. Draw up a table to record the answers to the following yes/no questions:
70 http://www.sinauer.com/
71 http://www.whfreeman.com/

Available for free at Connexions <http://cnx.org/content/col11410/1.3>


134 CHAPTER 2. LIFE PROCESSES IN PLANTS AND ANIMALS

i. Do you smoke?
ii. Are you overweight?
iii. Do you exercise regularly?
iv. Do you follow a healthy diet (low fat, low salt)
v. Do you have your blood pressure checked regularly?
vi. Do you have a family history of heart and circulatory disease?
1. Survey two adult male family member (father, grandfather or uncle) and two adult female family
members (mother, grandmother or aunt). Include the adults' rst name, gender, age and relationship
to you.
3.Record the results in your table. Also indicated the score they obtained:
i. yes=0; no=5
ii. yes=0; no=5
iii. yes=5; no=0
iv. yes=5; no=0
v. yes=5; no=0
vii. yes=0; no=5
4.Analyse the results by comparing the total score with the following descriptors:
30 marks- you take very good care of your heart. Well done!
25 marks- you take good care of your heart. Keep it up!
20 marks- you take reasonably good care of your heart but need to work
on a few aspects where you scored 0.
15 marks- you need to take better care of your heart.
0-10 marks- you do not look after your heart at all. It's time to make a
change to a healthier lifestyle.
Assessment Rubric

5
• Results

0- not done1- poorly presented.


2- average presentation of results,
but missing some detail.3- aver-
age presentation of results, in-
cluding all salient features and
information.4- good presentation
of results, but missing some
detail.5- good presentation of re-
sults, including all salient fea-
tures and information.
Table 2.9

Rich media:
Khan Academy

Available for free at Connexions <http://cnx.org/content/col11410/1.3>


135
http :// www . khanacademy . org / video / circulatory -
72 73 74 75 76 77 78 79 80 81 82

83
system - and - the - heart ?
84 85 86 87 88 89
playlist = Biology
90 91 92 93 94

Cardiac Magnetic Resonance imaging of Beating heart: Large magnets are used to create images of the
heart inside the body, without the need for surgery.
http://upload.wikimedia.org/wikipedia/commons/7/73/Four_chamber_cardiovascular_m
agnetic_resonance_imaging.gif 95

View from the top


http://commons.wikimedia.org/wiki/File:Beating_Heart_axial.gif 96

View from the side


http://commons.wikimedia.org/wiki/File:Cardiac_mri_ani_sagittal_bionerd.gif 97

2.3.1.11 Blood Vessels

2.3.1.11.1 Structure and functioning of arteries, veins, capillaries and valves

2.3.1.11.1.1 Arteries

• Arteries carry blood from away from the heart. The pressure created by the pumping heart forces
blood down the arteries.
• Arteries have three layers.

1. Outside layer  connective tissue


2. Middle layer  smooth muscle, allows contraction of the arteries to regulate blood ow and pressure
3. Inside layer  single layer of tightly connected simple squamous endothelial cells

• The large arteries close to the heart branch into smaller arterioles (smaller arteries) and eventually
branch into capillaries.
72 http://www.khanacademy.org/video/circulatory-%20system-and-the-heart?playlist=Biology
73 http://www.khanacademy.org/video/circulatory-%20system-and-the-heart?playlist=Biology
74 http://www.khanacademy.org/video/circulatory-%20system-and-the-heart?playlist=Biology
75 http://www.khanacademy.org/video/circulatory-%20system-and-the-heart?playlist=Biology
76 http://www.khanacademy.org/video/circulatory-%20system-and-the-heart?playlist=Biology
77 http://www.khanacademy.org/video/circulatory-%20system-and-the-heart?playlist=Biology
78 http://www.khanacademy.org/video/circulatory-%20system-and-the-heart?playlist=Biology
79 http://www.khanacademy.org/video/circulatory-%20system-and-the-heart?playlist=Biology
80 http://www.khanacademy.org/video/circulatory-%20system-and-the-heart?playlist=Biology
81 http://www.khanacademy.org/video/circulatory-%20system-and-the-heart?playlist=Biology
82 http://www.khanacademy.org/video/circulatory-%20system-and-the-heart?playlist=Biology
83 http://www.khanacademy.org/video/circulatory-%20system-and-the-heart?playlist=Biology
84 http://www.khanacademy.org/video/circulatory-%20system-and-the-heart?playlist=Biology
85 http://www.khanacademy.org/video/circulatory-%20system-and-the-heart?playlist=Biology
86 http://www.khanacademy.org/video/circulatory-%20system-and-the-heart?playlist=Biology
87 http://www.khanacademy.org/video/circulatory-%20system-and-the-heart?playlist=Biology
88 http://www.khanacademy.org/video/circulatory-%20system-and-the-heart?playlist=Biology
89 http://www.khanacademy.org/video/circulatory-%20system-and-the-heart?playlist=Biology
90 http://www.khanacademy.org/video/circulatory-%20system-and-the-heart?playlist=Biology
91 http://www.khanacademy.org/video/circulatory-%20system-and-the-heart?playlist=Biology
92 http://www.khanacademy.org/video/circulatory-%20system-and-the-heart?playlist=Biology
93 http://www.khanacademy.org/video/circulatory-%20system-and-the-heart?playlist=Biology
94 http://www.khanacademy.org/video/circulatory-%20system-and-the-heart?playlist=Biology
95 http://upload.wikimedia.org/wikipedia/commons/7/73/Four_chamber_cardiovascular_magnetic_resonance_imaging.gif
96 http://commons.wikimedia.org/wiki/File:Beating_Heart_axial.gif
97 http://commons.wikimedia.org/wiki/File:Cardiac_mri_ani_sagittal_bionerd.gif

Available for free at Connexions <http://cnx.org/content/col11410/1.3>


136 CHAPTER 2. LIFE PROCESSES IN PLANTS AND ANIMALS

2.3.1.11.1.1.1 Capillaries

• Capillaries are little more than a single layer or endothelial cells.


• Capillaries form intricate networks throughout the tissues.
• They allow water, nutrients and gasses to diuse out of the blood and waste materials to diuse into
the blood.
• This exchange occurs between the blood and the interstitial uid.
• The interstitial uid is the uid surrounding the cells.
• The blood never comes into contact with the cells.
• The blood and interstitial uid exchange material, and the interstitial uid then exchanges material
with the cells.

2.3.1.11.1.1.2 Veins

• The intricate networks formed by the capillaries eventually converge to form venules, (small veins)
• The venules then converge to form veins which return the blood to the heart.
• Veins only consist of two layers.

1. The outer layer is made up of connective tissue


2. The inner layer is made up of endothelial cells.

2.3.1.11.1.1.3 Valves

• Once the blood has passed through the capillaries very little blood pressure remains to return blood
to the heart.
• Instead of pressure from the heart veins use a series of valves to force blood to return to the heart.
• Contraction of the muscles squeezes the veins, pushing the blood through them.
• The valves cause the blood to ow in only one direction, back to the heart.

Available for free at Connexions <http://cnx.org/content/col11410/1.3>


137

Figure 2.50

Figure showing capillaries as the transition between arteries and veins


http://en.wikipedia.org/wiki/File:Illu_capillary.jpg
Interactive diagram illustrating arterial and venous structure.
http://www.phschool.com/science/biology_place/biocoach/cardio2/structure.html 98

98 http://www.phschool.com/science/biology_place/biocoach/cardio2/structure.html

Available for free at Connexions <http://cnx.org/content/col11410/1.3>


138 CHAPTER 2. LIFE PROCESSES IN PLANTS AND ANIMALS

Figure 2.51

Figure
Shows the how valves cause blood to only ow one way though veins
http://upload.wikimedia.org/wikipedia/commons/thumb/4/4a/Venous_valve.svg /2000px-
Venous_valve.svg.png
2.3.1.11.1.2 Capillaries

• Capillaries are little more than a single layer or endothelial cells.


• Capillaries form intricate networks throughout the tissues.
• They allow water, nutrients and gasses to diuse out of the blood and waste materials to diuse into
the blood.
• This exchange occurs between the blood and the interstitial uid.
• The interstitial uid is the uid surrounding the cells.
• The blood never comes into contact with the cells.
• The blood and interstitial uid exchange material, and the interstitial uid then exchanges material
with the cells.

2.3.1.11.1.2.1 Veins

• The intricate networks formed by the capillaries eventually converge to form venules, (small veins)
• The venules then converge to form veins which return the blood to the heart.
• Veins only consist of two layers.

Available for free at Connexions <http://cnx.org/content/col11410/1.3>


139
1. The outer layer is made up of connective tissue
2. The inner layer is made up of endothelial cells.

2.3.1.11.1.2.2 Valves

• Once the blood has passed through the capillaries very little blood pressure remains to return blood
to the heart.
• Instead of pressure from the heart veins use a series of valves to force blood to return to the heart.
• Contraction of the muscles squeezes the veins, pushing the blood through them.
• The valves cause the blood to ow in only one direction, back to the heart.

Figure 2.52

Figure showing capillaries as the transition between arteries and veins


http://en.wikipedia.org/wiki/File:Illu_capillary.jpg
Interactive diagram illustrating arterial and venous structure.
http://www.phschool.com/science/biology_place/biocoach/cardio2/structure.html 99

99 http://www.phschool.com/science/biology_place/biocoach/cardio2/structure.html

Available for free at Connexions <http://cnx.org/content/col11410/1.3>


140 CHAPTER 2. LIFE PROCESSES IN PLANTS AND ANIMALS

Figure 2.53

Figure
Shows the how valves cause blood to only ow one way though veins
http://upload.wikimedia.org/wikipedia/commons/thumb/4/4a/Venous_valve.svg /2000px-
Venous_valve.svg.png
2.3.1.11.1.3 Veins

• The intricate networks formed by the capillaries eventually converge to form venules, (small veins)
• The venules then converge to form veins which return the blood to the heart.
• Veins only consist of two layers.

1. The outer layer is made up of connective tissue


2. The inner layer is made up of endothelial cells.

2.3.1.11.1.3.1 Valves

• Once the blood has passed through the capillaries very little blood pressure remains to return blood
to the heart.
• Instead of pressure from the heart veins use a series of valves to force blood to return to the heart.
• Contraction of the muscles squeezes the veins, pushing the blood through them.
• The valves cause the blood to ow in only one direction, back to the heart.

Available for free at Connexions <http://cnx.org/content/col11410/1.3>


141

Figure 2.54

Figure showing capillaries as the transition between arteries and veins


http://en.wikipedia.org/wiki/File:Illu_capillary.jpg
Interactive diagram illustrating arterial and venous structure.
http://www.phschool.com/science/biology_place/biocoach/cardio2/structure.html 100

100 http://www.phschool.com/science/biology_place/biocoach/cardio2/structure.html

Available for free at Connexions <http://cnx.org/content/col11410/1.3>


142 CHAPTER 2. LIFE PROCESSES IN PLANTS AND ANIMALS

Figure 2.55

Figure
Shows the how valves cause blood to only ow one way though veins
http://upload.wikimedia.org/wikipedia/commons/thumb/4/4a/Venous_valve.svg /2000px-
Venous_valve.svg.png
2.3.1.11.1.4 Valves

• Once the blood has passed through the capillaries very little blood pressure remains to return blood
to the heart.
• Instead of pressure from the heart veins use a series of valves to force blood to return to the heart.
• Contraction of the muscles squeezes the veins, pushing the blood through them.
• The valves cause the blood to ow in only one direction, back to the heart.

Available for free at Connexions <http://cnx.org/content/col11410/1.3>


143

Figure 2.56

Figure showing capillaries as the transition between arteries and veins


http://en.wikipedia.org/wiki/File:Illu_capillary.jpg
Interactive diagram illustrating arterial and venous structure.
http://www.phschool.com/science/biology_place/biocoach/cardio2/structure.html 101

101 http://www.phschool.com/science/biology_place/biocoach/cardio2/structure.html

Available for free at Connexions <http://cnx.org/content/col11410/1.3>


144 CHAPTER 2. LIFE PROCESSES IN PLANTS AND ANIMALS

Figure 2.57

Figure
Shows the how valves cause blood to only ow one way though veins
http://upload.wikimedia.org/wikipedia/commons/thumb/4/4a/Venous_valve.svg /2000px-
Venous_valve.svg.png
2.3.1.11.2 Dierences between arteries and veins

Artery Vein
Small lumen Large lumen
Blood under high pressure Blood under low pressure
Valves absent Valves present
Carries blood away from heart Carries blood towards the heart
Carries oxygenated blood except pulmonary artery Carries deoxygenated blood except pulmonary vein
Table 2.10

2.3.1.11.3 Indigenous Knowledge Systems

Use and symbology of blood and heart in traditional black culture

Available for free at Connexions <http://cnx.org/content/col11410/1.3>


145
2.3.1.12 Fun facts about your heart

1. The average adult heart beats:

• 72 times a minute
• 100,000 times a day
• 3,600,000 times a year
• A billion times during a lifetime.

1. Each day your heart creates enough energy to drive a truck for 32 kilometres.
2. Your left lung is smaller than your right one to make room in your chest cavity for your heart.
3. Clench your st - the size of your st is more or less the size of your heart.
4. Laughing is good exercise for your heart. Whenever you laugh, the blood ow in your heart is increased,
keeping your heart healthy.
Investigation: Practical investigation of sheep's heart
Video: Doing a dissection
http://www.hometrainingtools.com/images/videos/Dissection_Video/dissection_vpl
ayer.html?TB_iframe=true&height=390&width=405 102

Equipment:

Figure 2.58

• 1 sheep heart • Cotton


• Cutting board • water
• Scalpel • funnel
• textbook • scissors

Table 2.11

1. EXTERNAL
(a)How would you describe the general shape of the heart?
(b)Note the grooves on the surface of the heart. In which direction do they run.
What do you observe in these grooves.
(c)Identify the atria and ventricles. How do they dier from each other in
appearance. What dierence do you notice between the atria and ventricles.
2. If the venae cavae are suciently long, insert a funnel into the superior vena cava and tie o
the inferior vena cava with a piece of cotton . When water is added through the superior vena cave
into the right atrium:
(a)What happens to the wall of the right ventricle?
(b)Press the right ventricle. What do you observe?
102 http://www.hometrainingtools.com/images/videos/Dissection_Video/dissection_vplayer.html?TB_iframe=true&height=390&width=405

Available for free at Connexions <http://cnx.org/content/col11410/1.3>


146 CHAPTER 2. LIFE PROCESSES IN PLANTS AND ANIMALS

(c)Release the pressure. What happens?


(d)Now press the left ventricle a few times. What do you notice?
(e)Now attach funnel to one of the pulmonary veins and tie o the others
(if possible). Pour water down the funnel and press the left ventricle.
What do you observe?
(f)Release the pressure and press the right ventricle. What do you observe?
Remove the funnel and tubes.
3. Cut the superior vena cava from the atrium and cut open the wall of the atrium. Dothe same
with the pulmonary vein and left atrium.
(a)Describe the appearance of the inner atrial surface.
(b)Determine the position of the pulmonary artery and the aorta by inserting a
glass rod through these vessel into the chambers of the heart.
Name the artery that leaves the right ventricle.
Name the artery that leaves the left ventricle.
4. Make an incision in the right side of the left ventricle from the oblique groove to the a
pex of the heart.
(a)What do you observe between the left atrium and left ventricle?
(b)How many aps do you see?
(c)What is the function of these aps?
5. Similarly, make an incision in the left wall of the right ventricle from the oblique groove.
(a)How many aps do you see between the atrium and the ventricle?
(b)What do these aps collectively form?
6. Compare the muscular walls of the:
(a) atria and the ventricles
(b) left and right ventricles
7. What do you observe between the two halves of the heart.
8. Examine the tendinous cords .
(a)Where are their points of attachment?
(b)What is their function
9. If the pulmonary artery and aorta are long enough, do this question. Using a funnel, pour water
into the pulmonary artery and the aorta.
(a)What do you notice?
(b)What do you see at the base of these arteries?
10. Cut the aorta and pulmonary arteries open longitudinally and examine the valves.
(a)How many parts are there to each of these valves?
(b)Compare the walls of the aorta and the pulmonary artery and suggest a
reason for any dierence you many nd.
2.3.2 2.3.2 Lymph circulatory system103
2.3.2.1 Lymph Circulatory System

Comparison of lymph and blood system: http://www.lymphnotes.com/article.php/id/150/ 104

Short videos on lymph: http://www.nlm.nih.gov/medlineplus/ency/article/002247.htm 105

YouTube video on lymph system: http://www.youtube.com/watch?v=qTXTDqvPnRk 106

103 This content is available online at <http://cnx.org/content/m43110/1.1/>.


104 http://www.lymphnotes.com/article.php/id/150/
105 http://www.nlm.nih.gov/medlineplus/ency/article/002247.htm
106 http://www.youtube.com/watch?v=qTXTDqvPnRk

Available for free at Connexions <http://cnx.org/content/col11410/1.3>


147
2.3.2.1.1 Composition of the lymphatic system

• The lymphatic system is composed of lymph vessels, lymph nodes, and organs.
• The organs associated with the lymphatic system are the spleen and thymus.
• The spleen is the boundary between the blood and the lymphatic system.
• Knots of lymphatic tissue in the spleen add lymphocytes to the blood.
• The spleen also acts as a lter for the blood, and helps to destroy worn out red-blood cells.
• In the event of damage to the spleen, it can be removed and its functions will be carried out by the
liver, the bone marrow and the lymph nodes.
• Most of the disease-ghting function of the adult mammal is carried out by the lymph nodes which
occur along the lymph ducts.
• Lymph nodes are small irregularly shaped masses through which lymph vessels ow.
• Clusters of nodes occur in the armpits, groin, and neck.
• Cells of the immune system line channels through the nodes and attack bacteria and viruses traveling
in the lymph so they basically act as tiny lters.
So. . .
• Lymph vessels are located as a network throughout all tissues in the body.
• Lymph vessels assist the circulatory system and all the cells of the body by removing wastes, germs
and excess water from the tissue uid.
• Lymph vessels carry lymph uid in ONE direction only , from the bottom of the body up towards
the heart.
• Valves prevent the lymph uid from owing backwards.
• Muscle contractions push the uid upwards.

2.3.2.1.2 Functions of lymphatic system

The main functions of the lymphatic system are as follows:


• the main function of the lymphatic system is to collect and transport tissue uids from the
intercellular spaces in all the tissues of the body, back to the veins in the blood system;
• it plays an important role in returning plasma proteins to the bloodstream ;
• digested fats are absorbed and then transported from the villi in the small intestine to the
bloodstream via the lacteals and lymph vessels.
• new lymphocytes are manufactured in the lymph nodes ;
• antibodies and anti (manufactures in the lymph nodes) assist the body to build up an eective
immunity to infectious diseases ;
• lymph nodes play an important role in the defence mechanism of the body . They lter out
micro-organisms (such as bacteria) and foreign substances such as toxins, etc.
• it transports large molecular compounds (such as enzymes and hormones) from their manufactured
sites to the bloodstream.
Youtube videos on lymph http://www.youtube.com/embed/Kh-XdNnTZUo 107

2.3.2.1.3 Relationship between blood system and lymphatic system

Table of comparison between the cardiovascular system and the lymphatic system

107 http://www.youtube.com/embed/Kh-XdNnTZUo

Available for free at Connexions <http://cnx.org/content/col11410/1.3>


148 CHAPTER 2. LIFE PROCESSES IN PLANTS AND ANIMALS

Cardiovascular System (Blood) Lymphatic System (Lymph)

Blood is responsible for collecting and distributing Lymph is responsible for collecting and removing
oxygen, nutrients and hormones to the tissues of waste products left behind in the tissues.
entire body.
Blood ows in the arteries, capillaries, and veins. Lymph ows in an open circuit from the tissues
into lymphatic vessels.
Blood ows towards the heart and away from the Lymph ows in one direction only (towards the
heart heart)
Blood is pumped by the heart to all parts of the Lymph is not pumped. It passively ows from the
body tissues into the lymph capillaries.
Blood consists of the liquid plasma that transports Lymph that has been ltered and is ready to re-
the red and white blood cells and platelets. turn to the cardiovascular system is a clear or milky
white uid.
Blood is visible and damage to blood vessels causes Lymph is invisible and damage to the lymphatic
obvious signs such as bleeding or bruising. system is dicult to detect until swelling occurs.
Blood is ltered by the kidneys. Lymph is ltered by lymph nodes located through-
out the body..
Table 2.12

2.3.3 2.3.3 Diseases of the heart and circulatory system108


2.3.3.1 Diseases of the heart and circulatory system

Khan: Heart diseases and heart attacks


http://www.khanacademy.org/video/heart-disease-and-heart- attacks?playlist=Biology 109

Khan: Thrombo-emboli and Thromboembolisms


http://www.khanacademy.org/video/thrombo-emboli-and- thromboembolisms?playlist=Biology 110

Khan: Stenosis, ischemia and heart failure


http://www.khanacademy.org/video/stenosisischemia-and-heart- failure?playlist=Biology 111

2.3.3.2 Cardiovascular diseases

• Cardiovascular disorders are a major cause of death in modern societies like heart attacks and
strokes .
• Angina is caused when the heart muscle receives an inadequate supply of blood, resulting in very
painful cramps in the chest area.

2.3.3.2.1 Heart attack

Cardiac muscle cells are provided with oxygenated blood by a system of coronary arteries . During
exercise the ow through these arteries is up to ve times normal ow. Blocked ow in coronary arteries can
result in death of heart muscle, leading to a heart attack .
Blockage of coronary arteries is usually the result of gradual buildup of lipids and cholesterol in the inner
wall of the coronary artery. Occasional chest pain can result during periods of stress or physical exercise.
108 This content is available online at <http://cnx.org/content/m43156/1.1/>.
109 http://www.khanacademy.org/video/heart-disease-and-heart-attacks?playlist=Biology
110 http://www.khanacademy.org/video/thrombo-emboli-and-thromboembolisms?playlist=Biology
111 http://www.khanacademy.org/video/stenosisischemia-and-heart-failure?playlist=Biology

Available for free at Connexions <http://cnx.org/content/col11410/1.3>


149
Angina indicates that oxygen needs are greater than the ability to deliver it and that a heart attack may
occur in the future. Heart muscle cells that die are not replaced since heart muscle cells do not divide. Heart
disease and coronary artery disease are the leading causes of death.

The gure below shows development of arterial plaque. Images from Purves et al., Life: The Science
of Biology , 4th Edition, by Sinauer Associates ( www.sinauer.com ) and WH Freeman (
112

www.whfreeman.com ), used with permission.


113

Figure 2.59

continued on next page

Available for free at Connexions <http://cnx.org/content/col11410/1.3>


150 CHAPTER 2. LIFE PROCESSES IN PLANTS AND ANIMALS

Figure 2.60

continued on next page

Available for free at Connexions <http://cnx.org/content/col11410/1.3>


151

Figure 2.61

Table 2.13

2.3.3.2.2 Hypertension

Hypertension , high blood pressure (the silent killer), occurs when blood pressure is consistently
above 140/90. Causes in most cases are unknown, although stress, obesity, high salt intake, and smoking
can add to a genetic tendency. Luckily, when diagnosed, the condition is usually treatable with medicines
and diet and exercise
112 http://www.sinauer.com/
113 http://www.whfreeman.com/

Available for free at Connexions <http://cnx.org/content/col11410/1.3>


152 CHAPTER 2. LIFE PROCESSES IN PLANTS AND ANIMALS

Figure 2.62

2.3.3.2.3 Stroke

What Is a Stroke? What Causes a Stroke?


A stroke is a condition where a blood clot or burst artery or blood vessel interrupts blood ow to an
area of the brain. A lack of oxygen and glucose (sugar) owing to the brain leads to the death of brain cells
and brain damage, often resulting in an impairment in speech, movement, and memory.
The outcome after a stroke depends on where the stroke occurs and how much of the brain is aected.
Smaller strokes may result in small problems, such as weakness in an arm or leg. Larger strokes may lead to
paralysis or death. Many stroke patients are left with weakness on one side of the body, diculty speaking,
and bladder problems.

Available for free at Connexions <http://cnx.org/content/col11410/1.3>


153
2.3.4 2.3.4 Summary114
2.3.4.1 Summary

• Nutrients and oxygen are required by cells for cellular respiration. These are transported by blood to
the various cells.
• Carbon dioxide and other wastes need to be transported from the cells to the exterior. This is also
transported via blood.
• Blood is pumped through the heart under high pressure to the various parts of the body.
• The heart is divided into the left side of the heart and the right side of the heart.
• The left side receives deoxygenated blood from the body via veins and sends it to the lungs to be
oxygenated.
• The right side of the heart receives oxygenated blood from the lungs and sends it via arteries to all
parts of the body.
• The lymphatic system is composed of lymph vessels, lymph nodes, and organs.
• Lymph vessels assist the circulatory system and all the cells of the body by removing wastes, germs
and excess water from the tissue uid.

114 This content is available online at <http://cnx.org/content/m43080/1.1/>.

Available for free at Connexions <http://cnx.org/content/col11410/1.3>


154 CHAPTER 2. LIFE PROCESSES IN PLANTS AND ANIMALS

Available for free at Connexions <http://cnx.org/content/col11410/1.3>


Chapter 3

Environmental studies

3.1 Biospheres to ecosystems


3.1.1 3.1.1 - Biosphere1
3.1.1.1 Concept of the Biosphere

In the past scientists have studied the various parts of the Earth. They have looked at botany (plants),
zoology (animals), geology (rocks), and physics (forces), but few have studied how all of these work together.
Now we are discovering that the Earth is much more than a bunch of parts. It is a whole. The Earth is a
whole system that works together. This means that there is an interconnection between all of Earth's living
and non-living parts. Everything works together in important ways. Scientists divide the Earth's System
into four sub-systems:
•biosphere (life)
•lithosphere (land)
•hydrosphere (water)
•atmosphere (air)
To see how the sub-systems of the Earth interact, watch the video: The Earth as a System:
http://www.oercommons.org/courses/earth-as-a-system/view 2

3.1.1.1.1 Biosphere

From: http://cnx.org/content/m16693/latest/?collection=col10548/latest
The biosphere is the region of the earth that encompasses all living organisms: plants, animals and
bacteria. It is a feature that distinguishes the earth from the other planets in the solar system. "Bio"
means life, and the term biosphere was rst coined by a Russian scientist (Vladimir Vernadsky) in the
1920s. Another term sometimes used is ecosphere ("eco" meaning home). The biosphere includes the outer
region of the earth (the lithosphere) and the lower region of the atmosphere (the troposphere). It also
includes the hydrosphere, the region of lakes, oceans, streams, ice and clouds comprising the earth's water
resources. Traditionally, the biosphere is considered to extend from the bottom of the oceans to the highest
mountaintops, a layer with an average thickness of about 20 kilometers. Scientists now know that some
forms of microbes live at great depths, sometimes several thousand meters into the earth's crust.
Nonetheless, the biosphere is a very tiny region on the scale of the whole earth, analogous to the thickness
of the skin on an apple. The bulk of living organisms actually live within a smaller fraction of the biosphere,
from about 500 meters below the ocean's surface to about 6 kilometers above sea level.
1 This content is available online at <http://cnx.org/content/m43182/1.1/>.
2 http://www.oercommons.org/courses/earth-as-a-system/view

Available for free at Connexions <http://cnx.org/content/col11410/1.3>

155
156 CHAPTER 3. ENVIRONMENTAL STUDIES

Dynamic interactions occur between the biotic region (biosphere) and the abiotic regions (atmosphere,
lithosphere and hydrosphere) of the earth. Energy, water, gases and nutrients are exchanged between the
regions on various spatial and time scales. Such exchanges depend upon, and can be altered by, the envi-
ronments of the regions. For example, the chemical processes of early life on earth (e.g. photosynthesis,
respiration, carbonate formation) transformed the reducing ancient atmosphere into the oxidizing (free oxy-
gen) environment of today. The interactive processes between the biosphere and the abiotic regions work
to maintain a kind of planetary equilibrium. These processes, as well as those that might disrupt this
equilibrium, involve a range of scientic and socioeconomic issues.
The study of the relationships of living organisms with one another and with their environment is the
science known as ecology. The word ecology comes from the Greek words oikos and logos, and literally means
"study of the home." The ecology of the earth can be studied at various levels: an individual (organism),
a population, a community, an ecosystem, a biome or the entire biosphere. The variety of living organisms
that inhabit an environment is a measure of its biodiversity.
3.1.1.1.2 Lithosphere

From: http://www.curriki.org/xwiki/bin/view/Coll_NROCscience/Lesson14TheLithosphereand
PlateTectonics_0 3

The layer of the mantle above the asthenosphere plus the entire crust make up a region called the
lithosphere. The lithosphere, and therefore, the earth's crust, is not a continuous shell, but is broken into
a series of plates that independently "oat" upon the asthenosphere, much like a raft on the ocean. These
plates are in constant motion, typically moving a few centimeters a year, and are driven by convection in
the mantle. The scientic theory that describes this phenomenon is called plate tectonics. According to the
theory of plate tectonics, the lithosphere is comprised of some seven major plates and several smaller ones.
Because these plates are in constant motion, interactions occur where plate boundaries meet.
http://www.ickr.com/photos/izzymunchted/3436486360/sizes/l/in/photostream/ 4

Figure 3.1

3.1.1.1.3 Hydrosphere

From Open Source Earth Science Course ( www.opencollegetextbook.org ) 5

The Hydrosphere contains all the water on Earth. As groundwater, the hydrosphere penetrates the soil
as far down as bedrock, mostly limestone, or other impermeable layers. It is found in aquifers as groundwater
and also between soil particles. As surface water, it is found in wetlands, marshes, estuaries, lakes, streams,
rivers, lakes, seas, and oceans. In the atmosphere, water is found as a gas throughout the dierent regions.
Water appears to permeate all the other spheres.
3 http://www.curriki.org/xwiki/bin/view/Coll_NROCscience/Lesson14TheLithosphereandPlateTectonics_0
4 http://www.ickr.com/photos/izzymunchted/3436486360/sizes/l/in/photostream/
5 http://www.opencollegetextbook.org/

Available for free at Connexions <http://cnx.org/content/col11410/1.3>


157
The Hydrosphere extends upward to about 15 kilometers in the Earth's atmosphere and downward to
depths on the order of ve kilometers in its crust. Indeed, the abundance of water on Earth is a unique
feature that clearly distinguishes our "Blue Planet" from others in the solar system. Not a drop of liquid
water can be found anywhere else in the solar system.
Though it cannot be found on any other planet, water is the most abundant inorganic substance at the
surface of the Earth. About 1.4 billion cubic kilometers of water in liquid and frozen form make up the
oceans, lakes, rivers, streams, glaciers, and groundwater.

Figure 3.2

Figure 3.3

Available for free at Connexions <http://cnx.org/content/col11410/1.3>


158 CHAPTER 3. ENVIRONMENTAL STUDIES

Figure 3.4

Figure 3.5

3.1.1.1.4 Atmosphere

The atmosphere, the gaseous layer that surrounds the earth, formed over four billion years ago. The earth's
atmosphere extends outward to about 1,000 kilometers where it transitions to interplanetary space. However,
most of the mass of the atmosphere (greater than 99 percent) is located within the rst 40 kilometers. The
sun and the earth are the main sources of radiant energy in the atmosphere. The sun's radiation spans the
infrared, visible and ultraviolet light regions, while the earth's radiation is mostly infrared.
The vertical temperature prole of the atmosphere is variable and depends upon the types of radiation
that aect each atmospheric layer. This, in turn, depends upon the chemical composition of that layer
(mostly involving trace gases). Based on these factors, the atmosphere can be divided into four distinct
layers: the troposphere, stratosphere, mesosphere, and thermosphere.
The troposphere is the atmospheric layer closest to the earth's surface. It extends about 8 - 16 kilometers
from the earth's surface. The thickness of the layer varies a few km according to latitude and the season of
the year. It is thicker near the equator and during the summer, and thinner near the poles and during the
winter. The troposphere contains the largest percentage of the mass of the atmosphere relative to the other
layers. It also contains some 99 percent of the total water vapor of the atmosphere.
The temperature of the troposphere is warm (roughly 17º C) near the surface of the earth. This is due to
the absorption of infrared radiation from the surface by water vapor and other greenhouse gases (e.g. carbon

Available for free at Connexions <http://cnx.org/content/col11410/1.3>


159
dioxide, nitrous oxide and methane) in the troposphere. The concentration of these gases decreases with
altitude, and therefore, the heating eect is greatest near the surface. The temperature in the troposphere
decreases at a rate of roughly 6.5º C per kilometer of altitude. The temperature at its upper boundary is
very cold (roughly -60º C).
Because hot air rises and cold air falls, there is a constant convective overturn of material in the tropo-
sphere. Indeed, the name troposphere means region of mixing. For this reason, all weather phenomena
occur in the troposphere. Water vapor evaporated from the earth's surface condenses in the cooler upper
regions of the troposphere and falls back to the surface as rain. Dust and pollutants injected into the tro-
posphere become well mixed in the layer, but are eventually washed out by rainfall. The troposphere is
therefore self cleaning.
A narrow zone at the top of the troposphere is called the tropopause. It eectively separates the under-
lying troposphere and the overlying stratosphere. The temperature in the tropopause is relatively constant.
Strong eastward winds, known as the jet stream, also occur here.
The stratosphere is the next major atmospheric layer. This layer extends from the tropopause (roughly
12 kilometers) to roughly 50 kilometers above the earth's surface. The temperature prole of the stratosphere
is quite dierent from that of the troposphere. The temperature remains relatively constant up to roughly
25 kilometers and then gradually increases up to the upper boundary of the layer. The amount of water
vapor in the stratosphere is very low, so it is not an important factor in the temperature regulation of the
layer. Instead, it is ozone (O3) that causes the observed temperature inversion.
The third layer in the earth's atmosphere is called the mesosphere. It extends from the stratopause (about
50 kilometers) to roughly 85 kilometers above the earth's surface. Because the mesosphere has negligible
amounts of water vapor and ozone for generating heat, the temperature drops across this layer. It is warmed
from the bottom by the stratosphere. The air is very thin in this region with a density about 1/1000 that of
the surface. With increasing altitude this layer becomes increasingly dominated by lighter gases, and in the
outer reaches, the remaining gases become stratied by molecular weight.
The fourth layer, the thermosphere, extends outward from about 85 kilometers to about 600 kilometers.
Its upper boundary is ill dened. The temperature in the thermosphere increases with altitude, up to 1500º
C or more. The high temperatures are the result of absorption of intense solar radiation by the last remaining
oxygen molecules. The temperature can vary substantially depending upon the level of solar activity.
The lower region of the thermosphere (up to about 550 kilometers) is also known as the ionosphere.
Because of the high temperatures in this region, gas particles become ionized. The ionosphere is important
because it reects radio waves from the earth's surface, allowing long-distance radio communication. The
visual atmospheric phenomenon known as the northern lights also occurs in this region. The outer region
of the atmosphere is known as the exosphere. The exosphere represents the nal transition between the
atmosphere and interplanetary space. It extends about 1000 kilometers and contains mainly helium and
hydrogen. Most satellites operate in this region.

Available for free at Connexions <http://cnx.org/content/col11410/1.3>


160 CHAPTER 3. ENVIRONMENTAL STUDIES

Figure 3.6

Figure 3.7

3.1.1.1.5 Interconnectedness with, and components of a global ecosystem

Concept: the earth is a system


Text from Open Source Earth Science Course
While studying the parts of the Earth System it is important to look for the emergent properties of the
Earth System. How do the parts of the Earth System come together to form a sum that is greater than the
sum of its parts? This question is best answered by focusing on the Earth's matter, energy, and life.
A system has two distinguishing characteristics. The rst is that it has SYNERGY. Synergy means that
the whole is greater than the sum of the parts. This sounds a lot more complicated than it is. What it
means is that when all of the pieces of a system are put together they are more valuable than all of the pieces
would be if they were considered separately. A home is a good example. If you were to lay all the pieces and
Available for free at Connexions <http://cnx.org/content/col11410/1.3>
161
parts of your home in a pile you would have a big pile of wood, insulation, pipes, wires, drywall, etc. Your
pile of house stu would be worth something but not nearly as much as your home is worth when all the
house stu is organized into a system.
The second distinguishing characteristic of a system is that it has EMERGENT PROPERTIES. Emergent
properties are properties that emerge as a result of how the system works together; properties that do not
exist without the system. In other words, emergent properties are characteristics that are unique to the
system as a whole. Let us consider the example of your home once again. Some emergent properties of your
home may be its comfort and its safety. The comfort of your home is a function of the materials used to
build it, the architectural design, and the furniture inside. The home's safety is a property dependent on the
design, the strength and location of its doors and windows, and the neighbourhood in which it was built.
Both the safety and comfort of your home are properties of the home that are a result of the home system;
they are not dependent on just one aspect of the home.
Text from Earth as a System. " Teachers' Domain. 17 Dec. 2005. Web. 15 Oct. 2011.
http://www.teachersdomain.org/resource/ess05.sci.ess.earthsys.hologlobe/
Understanding our planet as an integrated system of components and processes is a fundamental part of
Earth and space science research. Just as the human body is composed of interrelated systems that control
specic bodily functions, Earth's four principal components  the atmosphere (air), lithosphere (land),
hydrosphere (water), and biosphere (life)  perform critical roles that, together, support and sustain life on
the planet.
Nothing inuences the subsystems that contribute to Earth's dynamic behaviour more than heat. Heat
comes from two sources: solar energy and radioactivity in the Earth's core. Because of the angle at which
the Sun strikes Earth, Earth's surface is heated unevenly. This creates Earth's three major climate zones 
tropical, temperate, and polar  which then inuence what types of life ourish in dierent locations.
The uneven heating also controls weather systems. The heat absorbed by the oceans and carried by
its currents is constantly being released into the atmosphere. This heat and moisture drive atmospheric
circulation and set weather patterns in motion. The weather patterns then inuence vegetation, as well as
erosion and sediment transport.
The other heat source, deep within Earth's core, is responsible for plate tectonics, which gives the Earth
its physical character: mountain ranges and valleys, ocean basins and lake beds, and islands and trenches.
The heat from Earth's core generates convection cells within its mantle, which help drive plate activity.
Ever since the rst photos were sent back from space, our view of Earth has changed. Remote sensing
instruments, such as satellites, allow us to better understand the interrelationships between the dierent
subsystems. For instance, recordings made by remote and Earth-based instruments show that signicant
surface warming has occurred over the past three decades. Knowing this, scientists are working to determine
how this will aect  and already is aecting  the entire Earth system.
Possible slide-shows:
http://www.slideshare.net/Alyssa10/earth-science-biosphere-ppt 6

http://www.slideshare.net/shoreyl/3-biosphere 7

Video: The Earth as a System: http://www.oercommons.org/courses/earth-as-a-system/view 8

Questions
What are the parts of Earth's System?
What are the properties of the Earth's System?
How is the Earth's System part of a larger system?
6 http://www.slideshare.net/Alyssa10/earth-science-biosphere-ppt
7 http://www.slideshare.net/shoreyl/3-biosphere
8 http://www.oercommons.org/courses/earth-as-a-system/view

Available for free at Connexions <http://cnx.org/content/col11410/1.3>


162 CHAPTER 3. ENVIRONMENTAL STUDIES

3.1.2 3.1.2 Biomes9


3.1.2.1 Biomes

3.1.2.1.1 Key concepts

In this session we will focus on summarising what you need to know about:
• Terrestrial and aquatic biomes of Southern Africa.
• How climate, soil and vegetation inuence the organisms found in each.
• Location of the dierent biomes in South Africa.

3.1.2.1.2 Terminology & Denitions

Biomes can be dened as the major climatic regions of the world, classied according to their predominant
vegetation and characterised by adaptations of organisms to that particular environment.
3.1.2.1.3 Content

The biosphere can be divided into relatively large regions called biomes. A biome has a distinct climate
and certain living organisms (especially vegetation) characteristic to the region and may contain many
ecosystems. The key factors determining climate are average annual precipitation and temperature. These
factors, in turn, depend on the geography of the region, such as the latitude and altitude of the region, and
mountainous barriers. The major types of biomes include: aquatic , desert , forest , grassland and
tundra . Biomes have no distinct boundaries. Instead, there is a transition zone called an ecotone, which
contains a variety of plants and animals. For example, an ecotone might be a transition region between a
grassland and a desert, with species from both.
You will be required to learn about terrestrial and aquatic biomes.
3.1.2.1.4 Aquatic Biomes

Water covers a major portion of the earth's surface, so aquatic biomes contain a rich diversity of plants and
animals. Aquatic biomes can be subdivided into two basic types: freshwater and marine .
A freshwater region has a low salt concentration, usually less than 1 percent, and occurs in several
types of regions: ponds and lakes, streams and rivers, and wetlands.
• Ponds and lakes range in size, and small ponds may be seasonal. They sometimes have limited species
diversity due to isolation from other water environments. They can get their water from precipitation,
surface runo, rivers, and springs.
• Streams and rivers are bodies of owing water moving in one general direction (i.e., towards the sea).
Streams and rivers start at their upstream headwaters, which could be springs, snowmelt or even lakes.
They continue downstream to their mouths, which may be another stream, river, lake or ocean. The
environment of a stream or river may change along its length, ranging from clear, cool water near the
head, to warm, sediment-rich water near the mouth. The greatest diversity of living organisms usually
occurs in the middle region.
• Wetlands are places of still water that support aquatic plants, such as cattails, pond lilies and
cypress trees. Types of wetlands include marshes, swamps and bogs. Wetlands have the highest
diversity of species with many species of birds, mammals, amphibians and reptiles. Some wetlands,
such as salt marshes, are not freshwater regions.
Marine regions cover nearly three-fourths of the earth's surface. Marine bodies are salty, having approx-
imately 35 grams of dissolved salt per litre of water (3.5 percent). Oceans are very large marine bodies
9 This content is available online at <http://cnx.org/content/m43240/1.1/>.

Available for free at Connexions <http://cnx.org/content/col11410/1.3>


163
that dominate the earth's surface and hold the largest ecosystems. They contain a rich diversity of living
organisms. Ocean regions can be separated into four major zones: intertidal, pelagic, benthic and abyssal
.
• The intertidal zone is where the ocean meets the land. Sometimes, it is submerged and at other times
exposed, depending upon waves and tides.
• The pelagic zone includes the open ocean further away from land.
• The benthic zone is the region below the pelagic zone, but not including the very deepest parts of
the ocean. The bottom of this zone consists of sediments.
• The deepest parts of the ocean are known as the abyssal zone . This zone is very cold (near freezing
temperatures), and under great pressure from the overlying mass of water. Mid-ocean ridges occur on
the ocean oor in abyssal zones.

Figure 3.8

Figure Showing zonation


(Source from http://studentweb.cortland.edu/knowles86/Intertidalzone.gif ) 10

Coral reefs are found in the warm, clear, shallow waters of tropical oceans around islands or along
continental coastlines. They are mostly formed from calcium carbonate produced by living coral. Reefs
provide food and shelter for other organisms and protect shorelines from erosion.
10 http://studentweb.cortland.edu/knowles86/Intertidalzone.gif

Available for free at Connexions <http://cnx.org/content/col11410/1.3>


164 CHAPTER 3. ENVIRONMENTAL STUDIES

Figure 3.9

Coral Reef. (Source: Coral Reef Alliance Photobank)


Estuaries are partially enclosed areas where fresh water and silt from streams or rivers mix with
salty ocean water. They represent a transition from land to sea and from freshwater to saltwater. Estuaries
are biologically very productive areas and provide homes for a wide variety of plants, birds and animals.
3.1.2.1.5 Terrestrial Biomes

Terrestrial biomes characterise ecosystems on land, and are usually identied by the growth form of the
dominant vegetation, climate, and/or where they are located on the earth. The major terrestrial biomes
include the tundra biome , the forest biome , the grassland biome , and the desert biome . Note that
forests and grasslands are dened based on the growth form of the dominant vegetation whereas deserts are
classied based on the dominant climatic conditions. The geographic distribution of terrestrial biomes is
mostly inuenced by climatic conditions such as rainfall and temperature. The most recent classication of
the biomes in South Africa divides the region into the following seven biomes:
• Grassland
• Savanna
• Succulent Karoo
• Nama Karoo
• Forest
• Fynbos
• Desert.
• Thicket

Available for free at Connexions <http://cnx.org/content/col11410/1.3>


165

Figure 3.10

This map shows the dierent biomes of South Africa


(Source from http://cnx.org/content/m20153/latest/graphics1.png )
3.1.2.1.6 Grassland

Grasslands cover regions where moderate rainfall is sucient for the growth of grasses, but not enough for
stands of trees. There are two main types of grasslands: tropical grasslands (savannas) and temperate
grasslands . Tropical grasslands occur in warm climates such as Africa and very limited regions of Australia.
They have a few scattered trees and shrubs, but their distinct rainy and dry seasons prevent the formation of
tropical forests. Most temperate grasslands are treeless, relatively at and have rich soil, have been replaced
by farmland.

Available for free at Connexions <http://cnx.org/content/col11410/1.3>


166 CHAPTER 3. ENVIRONMENTAL STUDIES

Figure 3.11

(Source from http://www.ickr.com/photos/takver/5884439290/sizes/o/in/photostream 11


)
The information shown below shows the eect of climate change on the grassland biome
11 http://www.ickr.com/photos/takver/5884439290/sizes/o/in/photostream

Available for free at Connexions <http://cnx.org/content/col11410/1.3>


167

Available for free at Connexions <http://cnx.org/content/col11410/1.3>


Figure 3.12
168 CHAPTER 3. ENVIRONMENTAL STUDIES

Figure Reduction in grassland biome


(Source from http://maps.grida.no/go/graphic/changing_biomes_in_south_africa 12
)
Activity: Write down the advantages and disadvantages of burning grassland,

ADVANTAGES OF BURNING DISADVANTAGES OF BURNING

Table 3.1

Table 1
See memorandum at the end of this section:
3.1.2.1.7 Savanna

The Savanna Biome is the largest biome in Southern Africa. Mainly found in Mpumalanga and Limpopo
provinces but also in the coastal belt of KwaZulu Natal and the Eastern Cape Province. Summers are hot
and wet and the winters are cool with little or no rain. This biome is also known as the bushveld where
grasses are mainly found and regular res prevent the trees from dominating.
12 http://maps.grida.no/go/graphic/changing_biomes_in_south_africa

Available for free at Connexions <http://cnx.org/content/col11410/1.3>


169

Figure 3.13

(Source from http://edu.glogster.com/media/2/11/47/83/11478364.jpg 13


)
3.1.2.1.8 Succulent Karoo

The Succulent Karoo biome can be found in the west coast of the Northern Cape Province and the northern
parts of the Western Cape Province. This biome is hot in summer and cold in winter, although the rainfall
in this area is very low. 40% of plant species found here are endemic to this biome. The Namaqualand region
of this biome is famous for its colourful wild owers. Succulent plants are able to live through dry seasons
by using water stored in their leaves or stems.

Figure 3.14

13 http://edu.glogster.com/media/2/11/47/83/11478364.jpg

Available for free at Connexions <http://cnx.org/content/col11410/1.3>


170 CHAPTER 3. ENVIRONMENTAL STUDIES

(Source from http://planet.uwc.ac.za/nisl/BDC321/ekapa%20Cape%20Towns%20lowlands/biomes/image


s/succulentkaroo-01.jpg ) 14

Figure 3.15

This map shows the succulent karoo region within the orange and the nama karoo within the pink.
(Source from http://images-mediawiki-sites.thefullwiki.org/01/2/0/1/97579812567978300.jpg ) 15

3.1.2.1.9 Nama Karoo

The Nama Karoo is the second largest biome in South Africa. It forms the major part of the Northern Cape
Province. It is regarded as a semi-desert area receiving very little rain. The summers are very hot and the
winters are very cold. The dominant vegetation type is grasses.
14 http://planet.uwc.ac.za/nisl/BDC321/ekapa%20Cape%20Towns%20lowlands/biomes/images/succulentkaroo-01.jpg
15 http://images-mediawiki-sites.thefullwiki.org/01/2/0/1/97579812567978300.jpg

Available for free at Connexions <http://cnx.org/content/col11410/1.3>


171

Figure 3.16

(Source from http://www.plantzafrica.com/vegetation/vegimages/namakaroo2.jpg)


3.1.2.1.10 Forest

The forest biome in South Africa occurs in patches in areas such as Knysna of the Western Cape as well as
KwaZulu Natal, the Eastern Cape, Limpopo and Mpumalanga. Some of these forests experience rain only
in winter, while others get rainfall throughout the year.
Forests are dominated by trees of which the Yellowwood is the largest. There are many herbaceous and
bulbous plants that also occur.

Available for free at Connexions <http://cnx.org/content/col11410/1.3>


172 CHAPTER 3. ENVIRONMENTAL STUDIES

Figure 3.17

(Source from http://www.ickr.com/photos/jlascar/4460866346/sizes/o/in/photostream/ Jorge Las-


16

car's photostream)
Trees are not only producers , but as a result of their size they also create a habitat for certain
species. The leaf cover of trees provides shelter for animals, while the bark and ssures in the trees also
provide a habitat for numerous insect species. The leaf cover also creates a shady environment in which
shade-loving, low-growing plants can ourish.
When leaves or fruit fall from the trees and collect at the feet of the trees, another series of organisms
can appear. The decomposers , such as micro-organisms that cause the dead material to decay and
decompose, contribute to the decomposition of the nutrients so that they may return to the soil. Humus
is formed in this way. Humus is dead organic material. Other creatures that live o decayed organic
material, namely the detrivores , also promote this process of decomposition.
ACTIVITY: Do a poster to illustrate the Role-players in a Tree Ecosystem.
Bring pictures of animals, trees and other plants to class. The teacher will divide the class into groups.
Each group will prepare a poster to illustrate the mutual dependence of the trees, other plants and
animals. Each group must present its poster to the rest of the class.
Answer the following questions / follow the instructions arising from the class discussion:
QUESTIONS / INSTRUCTIONS
1. Supposing the tree on your poster was to fall over.
16 http://www.ickr.com/photos/jlascar/4460866346/sizes/o/in/photostream/

Available for free at Connexions <http://cnx.org/content/col11410/1.3>


173
a. Which organisms would die?
b. Which organisms would move away?
c. Which organisms would increase in number?
2. Describe the role played by trees in an ecosystem.
3. Ecologically speaking, why is it bad practice to rake up leaves under trees?
4. Name three more examples where humans harm ecosystems.

3.1.2.1.11 Fynbos

Fynbos grows in a 100-to-200-km-wide coastal belt stretching from Clanwilliam on the West coast to Port
Elizabeth on the Southeast coast  forms part of the Cape oral kingdom , where it accounts for half of
the surface area and 80% of the plant varieties. The fynbos in the western regions is richer and more varied
than in the eastern regions of South Africa.
Of the world's six oral kingdoms , this is the smallest and richest per area unit. Contrast it in size with
the Holarctic kingdom, which incorporates the whole of the northern hemisphere apart from the tropical
regions. The diversity of fynbos plants is extremely high, with over 9000 species of plants occurring in the
area, around 6200 of which are endemic , i.e. they do not grow anywhere else in the world.
The Cape Fynbos is the term given to a collection of plants that are mainly shrubs and is comprised of
species belonging to South Africa's south western and southern Cape. Fynbos can be dened as a shrubland
with an unusual mixture of plant types of dierent shapes and sizes that have been termed, "growth forms".
There are four of these growth forms; the proteoids - tall protea shrubs with large leaves; the ericoids 
heath-like shrubs; reed-like plants  the restoids; and bulbous herbs  the geophytes.
The mountain fynbos is found from Nieuwoudtville and Vanrhynsdorp south along the main mountain
ranges to Cape Hangklip and the Cape Peninsula and then eastwards on the mountains to near Grahamstown.
The vegetation is characterised by ericoids (heather), restoids (reeds) and proteoid shrubs like proteas and
conebushes.
Coastal fynbos is found along the western and southern coastlines of the Cape Province from sea-level to
about 150m above sea-level, where the soil is usually alkaline to sour. There are two types:
Coastal fynbos of the West Coast sands, from the Cape Flats to Redelinghuys on the West Coast, and
Coastal fynbos of the south coast limestone, from Danger Point to Mossel Bay.
The Strandveld veld type is found mainly on the lower parts of the western coastal plains and could be
regarded as a transitional stage between coastal fynbos and Karooveld. Some patches may be found on the
southern coast as far as close to Port Elizabeth.
The coastal rhenosterbosveldveld type is found on the lowlands along the coast on shale and granite,
from sea level to 400 m above. As a veld type, it is rich in a wide variety of species and dominated by the
rhenosterbush and the characteristic wealth of spring owers.

Available for free at Connexions <http://cnx.org/content/col11410/1.3>


174 CHAPTER 3. ENVIRONMENTAL STUDIES

Figure 3.18

(Source from http://www.itmonline.org/image/honey1.gif)

Available for free at Connexions <http://cnx.org/content/col11410/1.3>


175

Figure 3.19

(Source from http://4.bp.blogspot.com/_Dt7LueKRwF0/SduqKc1ZUPI/AAAAAAAABQ0/CgbnLf8LYX8/s400/2


009_03290148.jpg )17

ACTIVITY: Read the passage below.


The astonishing richness and diversity of the Western Cape's natural resources is matched only by the
resourcefulness and diversity of its many people. Historical patterns of unsustainable use of resources have
led to the Cape Floristic Region (CFR) being listed as one of the world's threatened bioregions, and the
scars are deeply etched in the land and its people.
Now the people of the Western Cape are exploring new and sustainable ways to value and benet from
these globally important assets.
South Africa's Cape Floristic region is legendary, and the unique nature of the fynbos biome has been
celebrated by biologists, conservationists, development experts, and ecologist worldwide.
(Adapted from speech by Tasneem Essop the Western Cape Provincial Minister for Environment, Plan-
ning and Economic Development)
Write an essay on the `Fynbos' biome and discuss the following aspects.
• What is the meaning of the term `fynbos'?
17 http://4.bp.blogspot.com/_Dt7LueKRwF0/SduqKc1ZUPI/AAAAAAAABQ0/CgbnLf8LYX8/s400/2009_03290148.jpg

Available for free at Connexions <http://cnx.org/content/col11410/1.3>


176 CHAPTER 3. ENVIRONMENTAL STUDIES

• Identify features of families/indicator species that make up this vegetation type


• Describe it ecological role in the environment
• Biological impacts on the environment of destroying this type of vegetation.
• Economical importance of it for the people of the Western Cape.
• Management strategies involved in protecting it.
Your essay may be written or typed . Marks will be awarded for originality and own interpretation.
Include a bibliography of three of more resources. No marks will be awarded for plagiarism.
Factual info : 5x5 (25)
Synthesis: (5)
Total: 30
3.1.2.1.12 Thicket

The thicket biome occurs along the coasts of KwaZulu Natal and the Eastern Cape. Most thickets occur in
river valleys. Thickets develop in areas where the rainfall is fairly high however; there may be dry periods
that prevent the vegetation from developing into forests. The vegetation of this biome includes short tress,
low intertwining shrubs and vines. There are no distinct layers of trees and shrubs with many large open
spaces as found in the forest biome.

Figure 3.20

Thickets in the Eastern Cape are comprised of dense impenetrable vegetation dominated by spiny, often
succulent trees and shrubs such as seen in this photograph taken near Uitenhage, E.Cape.
(Source from http://www.plantzafrica.com/vegetation/thicket.htm ) 18

18 http://www.plantzafrica.com/vegetation/thicket.htm

Available for free at Connexions <http://cnx.org/content/col11410/1.3>


177
3.1.2.1.13 Desert

The Desert Biome is found largely as the Namib Desert along the coast of Namibia. The Deserts are dry
areas where evaporation usually exceeds precipitation. Rainfall is low  less than 25 centimeters per year 
and can be highly variable and seasonal. The low humidity results in temperature extremes between day and
night. Deserts can be hot or cold. Hot deserts (e.g. the Namib and Kalahari) are very hot in the summer
and have relatively high temperatures throughout the year and have seasonal rainfall. This combination of
low rainfall and high temperatures keeps the air very dry, increasing its evaporating power. Deserts have
relatively little vegetation and the substrate consists mostly of sand, gravel or rocks. The transition regions
between deserts and grasslands are sometimes called semi-arid deserts.

Figure 3.21

Distribution map of Welwitschia mirabilis and a detail of Welwitschia mirabilis.


Activity
Knowledge Are : Diversity, change and continuity
Topic: Advertisements on South African Biomes
You work for an Advertising Agency that is bidding for the account of a top travel agency. The bid
includes designing a full page advert (A4) for the Getaway Magazine. Presentation, appeal and accuracy
will therefore be of top priority.
(DON'T FORGET TO CHECK MAPS, REFERENCE BOOK, ADVERTS, and BROCHURES FOR
IDEAS. DO NOT CUT AND PASTE OR COPY OTHER PEOPLE'S WORK )
The travel agency has specied that they would like the following to be included in the ad, which is
geared towards people looking for a dierent and fascinating holiday in a specic biome :
• A region in the biome of your choice, including cities and/or towns worth a visit
• Climate (of interest to tourists)
• Well-known geographical features in the region
• Mention of some interesting wildlife (i.e. birds, animals, plants) that may be seen

Available for free at Connexions <http://cnx.org/content/col11410/1.3>


178 CHAPTER 3. ENVIRONMENTAL STUDIES

• Pictures
• Tour dates
• The name of the travel agency, with contact information
You should also include, very discreetly, the name of your own agency . (Study some ads for ideas). Re-
member  THE SKILL IS IN THE CRISP, RELEVANT AND SUCCINCT WAY IN WHICH THE INFOR-
MATION IS PRESENTED.
ARE YOU THE BEST? CAN YOUR AD AGENCY WIN THE BID?
We have discussed the following six biomes from which you will select one:
Savannah, Forest, Nama-Karoo, Succulent-Karoo, Fynbos, Grassland.
You will have two periods and homework time to investigate and complete this assignment.
Mark
Name of Travel Agency and Biome /3
Climate information /2
Wildlife /2
Geographical features and towns/cities /4
Local crafts and use of resources /4
Size of advert Layout is neat. Good use of space, Font shape & size is appropriate. /4
Eye-catching. Colorful Use of diagrams/pictures, etc. /4
Use of language. Age-appropriate. Own words. /4
Only relevant information included. /4
Interesting information throughout /4
Followed all instructions /4
Adequate Bibliography supplied on back of this page /4
TOTAL
/40

Mark
Name of Travel Agency and Biome /3
Climate information /2
Wildlife /2
Geographical features and towns/cities /4
Local crafts and use of resources /4
Available for free at Connexions <http://cnx.org/content/col11410/1.3>
179

Size of advert Layout is neat. Good use of space, Font shape & size is appropriate. /4
Eye-catching. Colorful Use of diagrams/pictures, etc. /4
Use of language. Age-appropriate. Own words. /4
Only relevant information included. /4
Interesting information throughout /4
Followed all instructions /4
Adequate Bibliography supplied on back of this page /4
TOTAL
/40

Mark
Name of Travel Agency and Biome /3
Climate information /2
Wildlife /2
Geographical features and towns/cities /4
Local crafts and use of resources /4
Size of advert Layout is neat. Good use of space, Font shape & size is appropriate. /4
Eye-catching. Colorful Use of diagrams/pictures, etc. /4
Use of language. Age-appropriate. Own words. /4
Only relevant information included. /4
Interesting information throughout /4
Followed all instructions /4
Adequate Bibliography supplied on back of this page /4
TOTAL
/40

Available for free at Connexions <http://cnx.org/content/col11410/1.3>


180 CHAPTER 3. ENVIRONMENTAL STUDIES

MARK SCHEME Mark


Name of Travel Agency and Biome /3
Climate information /2
Wildlife /2
Geographical features and towns/cities /4
Local crafts and use of resources /4
Size of advert Layout is neat. Good use of space, Font shape & size is appropriate. /4
Eye-catching. Colorful Use of diagrams/pictures, etc. /4
Use of language. Age-appropriate. Own words. /4
Only relevant information included. /4
Interesting information throughout /4
Followed all instructions /4
Adequate Bibliography supplied on back of this page /4
TOTAL /40

Table 3.2

Activity
The following activity is to be done in groups of four
INSTRUCTIONS:
Brainstorm a suitable set of criteria for assessment for poster and verbal report
Select ONE biome from the list given and do the following:
Use suitable references to obtain as much information as possible on the plant and animals found in your
selected biome
Make notes about the climate, landscape, stating how some of these are adapted to their environment
Design an attractive poster to illustrate the landscape as well as the dominant plants and animals that
make up a food chain.
Display your poster on the classroom wall and
Each person of the group is to give a verbal presentation on an aspect of the biome you studied.
Assessment criteria:
Written:
Size A2 and bigger, heading font size
Layout  organisation aesthetic appeal, use of colour, creativity, eye- catching
Visual  drawings, diagrams
Information  relevant, factual, main points
Oral:
Condence
Subject knowledge
Poise
Eort
RUBRIC FOR ORAL PRESENTATION

Available for free at Connexions <http://cnx.org/content/col11410/1.3>


181

ASSESSMENTCRITERIAPERFORMANCE INDICATORS
3 2 1 0
Poise and con- Very condent, Less condent, No condence, No condence at
dence stands up straight, leans against desk, does not look up, all, ill prepared
makes good eye shues around nervous, shues
contact, does not
shue
Communication Stimulating, clear Clear at times Not clear, slightly None
and concise confusing
Body of presenta- Coherent, excel- Clear at times, Unclear, little sub- No subject knowl-
tion lent subject knowl- good subject ject knowledge edge, reads word
edge,information knowledge, accu- for word from
accurate and rate but not as notes, nonsense
detailed detailed
Time Eective use of Too long Too short
time
Enthusiasm /Ef- Very enthusiastic, Very enthusiastic, Little eortLittle No eortNo enthu-
fort worthwhile eort Lacks eort enthusiasm siasm
Table 3.3

(15)
Memo Activity 1: Discussing the value of a grassland ecosystem

ADVANTAGES OF BURNING DISADVANTAGES OF BURNING

• Hard seedpods that cover seeds crack open • Air pollution  smoke

• Species are rediscovered, e.g., the mountain • Animals and plants are injured and damaged,
rose or killed

• Plants that grow aggressively are restricted • Organisms in the soil are destroyed  humus
is reduced
continued on next page

Available for free at Connexions <http://cnx.org/content/col11410/1.3>


182 CHAPTER 3. ENVIRONMENTAL STUDIES

• Younger plants provide better nutrition • Grasses are weakened if burning is practised
(green grass after a severe winter) or occurs at the wrong time

Table 3.4

3.1.2.1.14 Rich media

http://www.curriki.org/xwiki/bin/view/Coll_NROCscience/Lesson25VideoBiosphere 19

The video of this lesson focuses on the biosphere, ecospheres, the lithosphere, the troposphere, the hy-
drosphere, ecology, individuals, populations, communities, ecosystems, biomes, biospheres, and biodiversity.
http://www.oercommons.org/courses/biomes/view 20

This interactive resource adapted from NASA features some of the physical and biological characteristics
of seven of the world's biomes.
http://www.southafrica.info/about/animals/ora.htm#ixzz1aqVbqENT 21

This classication of dierent biomes corresponds to decreasing average temperatures.


3.1.3 3.1.3 Environment22
3.1.3.1 Concept of an environment

Throughout history humans have inuenced and been inuenced by the natural world. While much of our
impact has been detrimental to the natural environment, we have preserved and protected certain resources
that are important to us. The environment, while highly valued by most, is used and altered by a wide
variety of people with many dierent interests and values. Diculties remain on how best to ensure the
protection of our environment and natural resources, and how to divide the earth's resources equally so that
all benet, and not just a select few.
There are currently many uncertainties regarding the future of the natural environment. However it is
clear that humans have had a role in its destruction and will be responsible in the future for its conservation.
Environmental problems are becoming more and more complex, especially as issues arise on a more global
level, such as that of atmospheric pollution or global warming. There is a realization that such complex
problems will demand complex solutions and the participation of all.
The challenge is to nd approaches to environmental management that give people the quality of life they
seek while protecting the environmental systems that are also the foundations of our well being. In order to
face these challenges, students today will need more than supercial knowledge or awareness of disconnected
environmental issues. A multidisciplinary approach to learning can build upon the strengths of a wide range
of elds of study, providing a deeper understanding of the technological, political, and social options and
strategies for both studying and managing the relationship between our society and the environment. There
will always be tradeos and, many times, unanticipated or unintended consequences.
3.1.3.2 Abiotic and biotic factors: eects on the community

There a number of characteristics of your local environment that can be classied into three broad categories,
which can be called the ABC's of the environment.
In the ABC's of the environment,
A-refers to the abiotic (physical, non-living) features of the area
B- identies the biotic (plant and animal) component of the environment.
19 http://www.curriki.org/xwiki/bin/view/Coll_NROCscience/Lesson25VideoBiosphere
20 http://www.oercommons.org/courses/biomes/view
21 http://www.southafrica.info/about/animals/ora.htm#ixzz1aqVbqENT
22 This content is available online at <http://cnx.org/content/m43180/1.1/>.

Available for free at Connexions <http://cnx.org/content/col11410/1.3>


183
C-C is the cultural (human) inuences.
Some ecologists think of the ABC's as forming a triangle with inter- relating sides. In a civilization as
complex as ours, no single side can exist uninuenced by others.
3.1.3.3 How Humans have an impact on the environment

3.1.3.3.1 The eect of humans on the composition of the atmosphere and the climate

This section will summarize some of the impacts that human's have had on the environment. In particular
we will look at the eect of humans on our abiotic (non-living) environment.
Summary
Ozone Depletion: the ozone layer which prevents UV light from entering the earth's surface is being
destroyed by CFC's and pollution
Greenhouse Eect: The UV light is being trapped within the earth's atmosphere by a blanket of
carbon dioxide and other greenhouse gasses, such as methane and nitrous oxide. The main greenhouse gases
are carbon dioxide and methane. They absorb much of the energy radiated from Earth and then re-radiate
it back to Earth. The levels of these gases in the atmosphere are slowly rising. Carbon dioxide is produced
when fossil fuels are burnt. Methane is released by cattle and from rice elds. An increase of only a few
degrees Celsius may cause big changes in the Earth's climate, upsetting weather patterns and aecting the
types of crops that can be grown in dierent parts of the world. Polar ice caps may melt, causing the sea
level to rise.

Figure 3.22

Global warming: refers to increase in earth's temperature due to the greenhouse eect
Deforestation: removal of natural forests. This destroys biodiversity, and also removes plants which
help to decrease the carbon dioxide burden.

Available for free at Connexions <http://cnx.org/content/col11410/1.3>


184 CHAPTER 3. ENVIRONMENTAL STUDIES

Acid Rain Burning fossil fuels may also produce sulphur dioxide and nitrogen oxide gases. These gases
rise up in the atmosphere from factories, power stations and vehicles and dissolve in the water in clouds
forming acid rain. Acid rain can damage trees directly.If the water in rivers and lakes becomes too acidic,
plants and animals cannot survive. Acid rain also damages buildings
Destruction of biodiversity - hunting, pollution, destruction of the natural environment and intro-
duction of alien species can result in the extinction of native species.
3.1.3.3.1.1 Denitions and terms

Greenhouse gases: Methane, nitrous oxide, carbon dioxide


Fossil Fuels: Natural substances made deep within the earth by the decomposition of plants and
animal remains over millions of years
3.1.3.3.1.2 The Greenhouse Eect

http://www.curriki.org/xwiki/bin/view/Coll_Athabasca/Unit5-Lesson3TheGreenhouseE ect 23

With the rise to prominence of the issue of global warming, it is important to discuss the greenhouse
eect. The name comes from the everyday concept of a greenhouse, where sunlight is allowed to pass
through transparent panels and shine on the plants inside. This provides energy to the plants, but also
warms everything inside the greenhouse. With the sealed layer of transparent panels, the warmth is trapped
inside and the greenhouse becomes much warmer than the environment outside.
The Earth's atmosphere functions exactly like this, except there are no transparent panels. When sunlight
shines down on the Earth, most of it is absorbed on the surface, giving us warmth and energy. Some of the
light is absorbed by the atmosphere before it hits the surface, and a very small amount of the light is also
reected back o the surface toward outer space. Additionally, the surface of the Earth releases heat into
the atmosphere, such as can be seen over a road on a hot day.
The greenhouse eect is not limited to Earth. Any planet that has a signicant atmosphere has some
kind of greenhouse eect. Venus has a signicant greenhouse eect that keeps the surface of the planet
extremely hot, averaging around 460C. A probe that was sent to study the planet survived for only two
hours before melting, even though it was designed with durable metals.
23 http://www.curriki.org/xwiki/bin/view/Coll_Athabasca/Unit5-Lesson3TheGreenhouseEect

Available for free at Connexions <http://cnx.org/content/col11410/1.3>


185

Figure 3.23

With the reection of light o the surface and the surface radiation of heat, much of the energy from
sunlight would be lost back to space. Fortunately the atmosphere acts like the transparent panels from the
greenhouse trapping the heat. Natural gases in our atmosphere called greenhouse gases (such as carbon
dioxide and water vapour) are extremely good at absorbing various kinds of sunlight. So, rather than
escaping back into space, much of this reected light and heat is actually absorbed by the greenhouse gases.
This has a signicant warming eect on our atmosphere.
Many people associate the greenhouse eect with global warming. In fact, there is so much confusion,
that these terms are sometimes used interchangeably. The greenhouse eect is naturally occurring on most
planets, and it is necessary on Earth to maintain life as we know it.
Without the greenhouse eect, the temperature of the Earth might be as much as 30C cooler! That
would alter the surface of the Earth signicantly, covering much of it with ice. We need the greenhouse eect
to survive on Earth.
However, there can be too much of a good thing. Human beings have begun adding a large amount
of greenhouse gases, primarily carbon dioxide, into our atmosphere. This came mostly with the industrial
revolution when we began to burn coal and gasoline, and now many other fossil fuels (such as propane,
natural gases), and even wood, in great quantities. With this increase in carbon dioxide in our atmosphere,
Available for free at Connexions <http://cnx.org/content/col11410/1.3>
186 CHAPTER 3. ENVIRONMENTAL STUDIES

there is more gas to absorb energy. With more energy being absorbed, the temperature of the atmosphere is
beginning to increase, causing changes within our weather patterns, and other inuences on the ecosystems
of the Earth. This is called climate change.
In the past few decades the population of the Earth has doubled to over six billion people. These six billion
people foster a large increase in automobile transportation; the major source of the increase in greenhouse
gases. The greater population has also required more resources such as land. Large amounts of forest have
been cut down. Trees are one of the most important organisms that actually remove carbon dioxide from the
atmosphere during photosynthesis. So not only are humans adding more carbon dioxide to the atmosphere,
but they are also destroying trees that would otherwise be helping to absorb excess carbon dioxide from the
atmosphere.
We will not know the full impact of global warming until perhaps the middle of this century. This is
because it takes so long for the full impact to be felt. You may remember that water vapour and carbon
dioxide are a very small part of the makeup of our original atmosphere (see Module 5, Tutorial 1). So as
we add carbon dioxide from burning fossil fuels there is only a very small change in the makeup of our
atmosphere. In fact it takes a long time for the atmosphere to mix in the added greenhouse gases fully.
Scientists say that even if we halted the release of greenhouse gases today, the climate would continue to
warm until about the year 2050 as the atmosphere reaches a new stable state.
3.1.3.3.1.3 The eect of humans on biodiversity

This section will summarize some of the impacts that human's have had on the natural environment, focussing
on our impact on the biotic (living) environment.
Summary
Destruction of biodiversity - hunting, pollution, destruction of the natural environment and intro-
duction of alien species can result in the extinction of native species.
Introduction of alien invasive species: These are plants and animals that are NOT indigenous to
a particular country. The plants and animals grow rapidly as they are alien and have no natural predators
to feed on them.
Chemical control: use of insecticides to kill insects that damage food crops. This also harms the
environment and gets washed into rivers, dams and seas, negatively impacting on food chains.
Biological control: use natural predators to eradicate (get rid of) of pests that feed and damage on
food crops. This ensures that the useful plants and animals as well as the soil as a natural resource is not
damaged.
Deforestation: removal of natural forests. This destroys biodiversity, and also removes plants which
help to decrease the carbon dioxide burden.
Acid Rain : Burning fossil fuels may also produce sulphur dioxide and nitrogen oxide gases. These
gases rise up in the atmosphere from factories, power stations and vehicles and dissolve in the water in clouds
forming acid rain. Acid rain can damage trees directly.If the water in rivers and lakes becomes too acidic,
plants and animals cannot survive. Acid rain also damages buildings
Eutrophication: Pollution of water by fertilisers may cause eutrophication. The fertiliser causes
rapid growth of aquatic plants such as algae. The increased amount of algae on the surface of the water
blocks out sunlight, preventing plants at the bottom of river from photosynthesising. These plants die and
begin to decompose. Micro-organisms feed and decompose dead plants using up oxygen in the water. Fish
and other aquatic organisms die due to oxygen shortage.
Sewage pollutes the sea and fresh water, unless it is treated properly.This is a health hazard, but it
also upsets the balance of organisms living in the water. Untreated sewage provides food for micro-organisms
and causes eutrophication.
Toxic chemicals are released from industrial plants and by farmers. These can pollute the land and
water.Some toxins e.g. DDT can accumulate along food chains until the top predator has very high, often
lethal, amounts in its body.DDT is carcinogenic (cancer causing)

Available for free at Connexions <http://cnx.org/content/col11410/1.3>


187
3.1.3.3.1.3.1 Denitions and terms

Pollution: an undesirable change in the natural resources due to negative inuences of human activities.
Thermal pollution: hot waste water released by industries and mining into rivers and lakes increases
temperature of water `driving' out oxygen killing sh and other aquatic life.
Contamination: not t for human consumption as it contains bacteria
Sewage: waste water containing faeces
Flora: refers to all plant life
Fauna: refers to all animal life

3.1.3.3.1.4 Activities

3.1.3.3.1.4.1 The Greenhouse eect

To see how greenhouse gases aect the climate try this simulation from PhET. Explore the atmosphere during
the ice age and today. What happens when you add clouds? Change the greenhouse gas concentration and
see how the temperature changes. Then compare to the eect of glass panes. Zoom in and see how light
interacts with molecules. Do all atmospheric gases contribute to the greenhouse eect?
Phet: The Greenhouse Eect: http://phet.colorado.edu/en/simulation/greenhouse 24

3.1.3.3.1.4.2 Human's inuence on greenhouse gas concentrations

Take a look at http://www.breathingearth.net/ to see how much CO2 is currently been released into the
atmosphere.
Watch for 4 minutes. How many people were born in that time? How many people died?
If the current grade 9's repeated this exercise exactly one year from today, at exactly the same time of
day, by how much will the world's population have grown? Do you think this is a problem? Why?
How much CO2 will have been added to the atmosphere by that time? How does South Africa compare to
the rest of the world? Do you think all South African's contribute equally to CO2 emissions in our country?
3.1.3.3.1.4.3 Discovering your impact

What Is A Carbon Footprint?


A carbon footprint is a measure of the impact our activities have on the environment, and in particular
climate change. It relates to the amount of greenhouse gases produced in our day-to-day lives through
burning fossil fuels for electricity, heating and transportation etc.
The carbon footprint is a measurement of all greenhouse gases we individually produce and has units of
tonnes (or kg) of carbon dioxide equivalent.
A carbon footprint is made up of the sum of two parts, the primary footprint and the secondary footprint.
The primary footprint is a measure of our direct emissions of CO2 from the burning of fossil fuels
including domestic energy consumption and transportation (e.g. car and plane). We have direct control of
these.
The secondary footprint is a measure of the indirect CO2 emissions from the whole lifecycle of
products we use - those associated with their manufacture and eventual breakdown. To put it very simply
 the more we buy the more emissions will be caused on our behalf.
To work out what your carbon footprint is visit: http://www.carbonfootprint.com/calculator.aspx 25

To discover how to reduce your carbon footprint visit: http://www.carbonfootprint.com/minimisecfp.html


26

24 http://phet.colorado.edu/en/simulation/greenhouse
25 http://www.carbonfootprint.com/calculator.aspx
26 http://www.carbonfootprint.com/minimisecfp.html

Available for free at Connexions <http://cnx.org/content/col11410/1.3>


188 CHAPTER 3. ENVIRONMENTAL STUDIES

3.1.3.3.1.4.4 Climate Change

Student's guide to climate change


http://www.epa.gov/climatechange/kids/index.html 27

http://climate.nasa.gov/ 28

http://climate.nasa.gov/imagesVideo/climateReel/index.cfm 29

3.1.3.3.1.5 Assignment

Identify the ABC's (abiotic, biotic and cultural characteristics) of a natural environment near you. To make
your ABC prole, follow the instructions below.
1. Select an area that is undeveloped (i.e. no buildings, no pavement, no bulldozing, no spraying of
pesticides, no farming, no grazing, etc.). Your area must be at least the size of a soccer eld. For some this
will be an easy walk from their homes. Others will have to travel quite a distance[U+2011][U+2011]lucky
you! You can think of it as a eld trip. Make a map of your province and show, approximately, where your
area is located.
2. Identify the at least 10 A (abiotic) features of your area. Consider factors such as:
• Landforms (mesa, mountain, valley, bench, etc..
• Altitude

3. Identify at least 15 B (biotic) features of the area. (You may use common names.) Consider things such
as:
• Plants (trees, shrubs, grasses, owers, etc.)
• Insects (ants, bees, praying mantis, etc.)
• Amphibians, reptiles, and/or sh

4. Identify at least 3 C (cultural) components. Look for evidence of human inuence. Consider things
such as:
• *Recycling, conservation eorts
• *Pollution
• *Introduced species

ANALYSIS
NB- Come back and use South African examples for the model answer examples
Examine the data you collected when making your ABC prole. Use your collected data to answer the
following questions.
1. What eect does the environment (abiotic) have on the organisms (biotic) living there? Give FIVE
specic examples from your prole. [For example: Lily pads (biotic) are able to grow in my area because
it is a natural wetland that has standing, stagnant water (abiotic) all year long.]
2. What eect do the organisms (biotic) have on the environment (abiotic)? Give THREE specic
examples from your prole. [For example: The area is heavily shaded by spruce trees (biotic). The shade
keeps the soil moist (abiotic) and reduces the air temperature.]
3. How do natural forces aect the area? Give ONE specic example from your prole. Consider the
direction of the prevailing winds, the direction from which the sun's rays come, gravity (if you are on a
slope), etc. . .
4. How have humans aected your area? Give ONE specic example.
5. Predict how your area would change if the amount of rainfall doubled. Be sure to mention how this
increase in rainfall would aect the abiotic and biotic factors.
27 http://www.epa.gov/climatechange/kids/index.html
28 http://climate.nasa.gov/
29 http://climate.nasa.gov/imagesVideo/climateReel/index.cfm

Available for free at Connexions <http://cnx.org/content/col11410/1.3>


189
3.1.3.3.2 Questions

Question 1
Read the following text and answer the questions that follow :
South Africa has oered to slow the growth of its greenhouse gas emissions by 34% by 2020. The
biggest producers of greenhouse gases in South Africa are Eskom, the producers of electricity, and Sasol, the
producers of petrol and diesel from fossil fuels.
1.1.Which is the most important greenhouse gas?(1)
1.2.Explain why Eskom and Sasol produce the most greenhouse gases. (2)
1.3.You are a scientist who has been employed by the government to check that the target is reached.
Briey explain the methodology you would use to do this. (3)
1.4.Suggest two strategies that Eskom and Sasol could use to reduce their greenhouse gas emissions. (2)
Question 2
Study the table below on ozone depletion and answer the questions that follow:
Amount of ozone depletion of stratosphere between 1982 & 1996

Year Ozone depletion (in tons)


1982 2000
1984 3000
1986 4000
1988 3500
1990 6000
1992 7000
1994 8000
1996 7000
Table 3.5

2.1.Comment on the ozone depletion during the period 1982 to 1996.(2)


2.2.How much of the ozone was lost from the stratosphere in 1990? (1)
2.3.CFC's can stay around for about 100 years. What is the implication of this for the ozone layer? (1)
2.4.Give ONE possible reason for the decrease in ozone depletion after 1994.(2)
Question 3
The dying of trees due to acid rain is brought about by changes in the pH of the soil. Tshepo observed
that trees in his town were gradually dying. He decided to investigate the eect of acid rain on germinating
bean seeds. He did the following:
• Placed cotton wool on 6 saucers
• Poured a solution with a dierent pH (varying from pH 2 to pH 7) onto the cotton wool in each saucer
• Placed 50 bean seeds onto the cotton wool in each saucer
• Covered the seeds and left them on the shelf for a week
• The following results were obtained:

Number of germinating seeds

2 0
3 4
Available for free at Connexions <http://cnx.org/content/col11410/1.3>
190 CHAPTER 3. ENVIRONMENTAL STUDIES

4 10
5 15
6 28
7 35

Number of germinating seeds

2 0
3 4
4 10
5 15
6 28
7 35

Number of germinating seeds

2 0
3 4
4 10
5 15
6 28
7 35

pH Number of germinating seeds

2 0
3 4
4 10
5 15
6 28
7 35
Available for free at Connexions <http://cnx.org/content/col11410/1.3>
191
Table 3.6

3.1.Name the independent variable in this investigation.(1)


3.2.Name the dependent variable in this investigation.(1)
3.3.Name TWO ways in which the investigation could be improved to obtain accurate and valid results.(2)
3.4.Describe the relationship between pH and the number of germinating seeds. (2)
3.5.Suggest THREE strategies that the government can use to help reduce the formation of acid rain.
(3)
QUESTION 4
The graph below shows the eect of biocontrol on an invasive alien plant population.
Graph showing the eect of biocontrol on an invasive alien plant population.

Figure 3.24

4.1.Explain what is meant by the term `biocontrol'.(1)


4.2.Give two reasons that you can see from the graph to support the fact that biocontrol has been
successful in controlling the alien plant population. (2)
4.3.Why is host specicity so important when introducing a biocontrol agent?(2)
Read the following text and answer the questions that follow:
QUESTION 5:
GLOBAL BURDEN OF DISEASE DUE TO INDOOR AIR POLLUTION
In the year 2000, indoor air pollution from solid fuel use was responsible for more than 1,6 million annual
deaths. Smoke and gases that come from the burning of fossil fuels, such as coal, charcoal and wood, to heat
homes and to cook contributes to indoor pollution.
Dependence on burning fossil fuels to meet basic energy needs is one of the underlying causes of asthma,
bronchitis, pneumnia and other respiratory illnesses among children.
Adapted from: http://www.who.int/indoorair/health_impacts/burden_global/enprint.html 30

30 http://www.who.int/indoorair/health_impacts/burden_global/enprint.html

Available for free at Connexions <http://cnx.org/content/col11410/1.3>


192 CHAPTER 3. ENVIRONMENTAL STUDIES

5.1 Name THREE illnesses that are thought to be made worse by exposure to indoor pollution.(3)
5.2 Why do you think indoor pollution causes so many medical problems?(2)
Suggest TWO ways in which communities can try to decrease the negative eects of indoor pollution.(2)
Studies show that the average South African has a carbon footprint that is higher than the world
average because most of our electricity is generated from burning coal.
5.3 State THREE ways to solve this problem.(3)
Question 6
(NSC Additional Exemplar 2008 Life Science Paper 2 Question 1)
Study the table below showing the amount of medical waste produced by three provinces over a number
of years
Amount of medical waste (tones)

1995
1997
1999
2001
2003

Province A 357 398 410 426 450

Province B 283 290 300 312 330

Province C 230 240 245 270 290

Amount of medical waste (tones)

Available for free at Connexions <http://cnx.org/content/col11410/1.3>


193

1995
1997
1999
2001
2003

Province A 357 398 410 426 450

Province B 283 290 300 312 330

Province C 230 240 245 270 290

Amount of medical waste (tones)

1995
1997
1999
2001
2003

Province A 357 398 410 426 450

Available for free at Connexions <http://cnx.org/content/col11410/1.3>


194 CHAPTER 3. ENVIRONMENTAL STUDIES

Province B 283 290 300 312 330

Province C 230 240 245 270 290

Year Amount of medical waste (tones)

1995 1997 1999 2001 2003

Province A 357 398 410 426 450


Province B 283 290 300 312 330
Province C 230 240 245 270 290
Table 3.7

6.1Which province has shown the most rapid increase in the amount of medical waste produced?(1)
6.2What was the percentage increase of medical waste produced by the province named in QUESTION
2.1 over the period 1995 to 2003? Show ALL workings. (3)
6.3Give TWO negative eects of dumping medical waste.(2)
Question 7
The graph below shows the eect of dierent amounts of sewage discharged into a river, on the amount
of dissolved oxygen in the water. Study the graph and answer the questions that follow.

Available for free at Connexions <http://cnx.org/content/col11410/1.3>


195

Figure 3.25

7.1.Why does the sewage discharge cause oxygen levels to decrease?(2)


7.2.Why does the minimum amount of oxygen occur some distance downstream of the point of sewage
discharge?(2)
7.3.Why does the level of dissolved oxygen in the river then increase again further downstream?(2)
7.4.Give TWO possible reasons why sewage would be discharged into river systems in a developing urban
environment.(4)
7.5.Name one positive and one negative eect that quite heavy pollution could have on a rural farming
community living at point X down the river.(2)
Question 8
8.1Humans can inuence the environment in ONE of the following positive ways.
ABurning fossil fuels
BConservation
CPollution
DDeforestation
8.2Pollutant gases have an adverse eect on the environment and on our health. The amounts of these
gases have been steadily increasing over the years. The table below shows the source and amount of some
pollutant gases produced by human activities.
Source
Amount produced per year (millions of tons)

Carbon dioxide Vehicle exhausts 350


Available for free at Connexions <http://cnx.org/content/col11410/1.3>
196 CHAPTER 3. ENVIRONMENTAL STUDIES

Sulphur dioxide Burning coal and oil, industry 200

Nitrogen oxide Vehicle exhaust 55

Source
Amount produced per year (millions of tons)

Carbon dioxide Vehicle exhausts 350

Sulphur dioxide Burning coal and oil, industry 200

Nitrogen oxide Vehicle exhaust 55

Source
Amount produced per year (millions of tons)

Carbon dioxide Vehicle exhausts 350

Sulphur dioxide Burning coal and oil, industry 200

Nitrogen oxide Vehicle exhaust 55

Pollutant gas Source Amount produced per year (millions of tons)

Carbon dioxide Vehicle exhausts 350


Sulphur dioxide Burning coal and oil, industry 200
Nitrogen oxide Vehicle exhaust 55
Table 3.8

8.3.Which ONE of the following statements is correct?


A150 million tons of nitrogen oxide is produced.
BCarbon monoxide is produced by industry.
Available for free at Connexions <http://cnx.org/content/col11410/1.3>
197
CSulphur dioxide is produced by vehicle exhaust.
DCarbon monoxide is the gas produced in the largest amount.
8.4.Possible measures for the reduction of air pollution would NOT include
Aproviding subsidies for solar panels
Bproviding subsidies for truck drivers
Crewards for cleaner production by industries
Dimproving the public transport system
8.5Adding harmful substances to the environment is termed
Aexploitation
Bforestation
Cpollution
Ddetoxication
Question 9
The map below shows a region of coastline close to where a giant oil tanker was wrecked at sea. The
shallow waters of the coastline provided a rich source of edible crabs. Oil does not kill the crabs but harm
their esh, making them inedible and they cannot be sold. Samples of crabs were collected at sites A to
D. The number of crabs is indicated by the size of the circle. The extent of the shaded part at each site
represents the proportion of crabs with diseased esh after the disaster.

Figure 3.26

9.1Which sample site (A to D) had the highest number of crabs? (1)


9.2.In which sample site was the crabs only rarely found?(1)
9.3Name the agent of pollution that aected the crabs.(1)
9.4In which sample site was the most crabs aected compared to the population size? (1)
9.5Explain your answer to QUESTION 2.4(2)
9.6.List TWO strategies that could reduce the eects of oil pollution at sea. (2)

Available for free at Connexions <http://cnx.org/content/col11410/1.3>


198 CHAPTER 3. ENVIRONMENTAL STUDIES

3.1.3.3.3 Discussion Points

3.1.3.3.3.1 The Tragedy of the Commons

From: http://cnx.org/content/m16743/latest/?collection=col10548/latest (AP Environmental Science:


Environmental Ethics) from Connexions
In his essay, The Tragedy of the Commons, Garrett Hardin (1968) looked at what happens when humans
do not limit their actions by including the land as part of their ethic. The tragedy of the commons develops
in the following way: Picture a pasture open to all. It is to be expected that each herdsman will try to
keep as many cattle as possible on the commons. Such an arrangement may work satisfactorily for centuries,
because tribal wars, poaching and disease keep the numbers of both man and beast well below the carrying
capacity of the land. Finally, however, comes the day of reckoning (i.e., the day when the long- desired
goal of social stability becomes a reality). At this point, the inherent logic of the commons remorselessly
generates tragedy.
As a rational being, each herdsman seeks to maximize his gain. Explicitly or implicitly, more or less
consciously, he asks: "What is the utility to me of adding one more animal to my herd?" This utility has
both negative and positive components. The positive component is a function of the increment of one animal.
Since the herdsman receives all the proceeds from the sale of the additional animal, the positive utility is
nearly +1. The negative component is a function of the additional overgrazing created by one more animal.
However, as the eects of overgrazing are shared by all of the herdsmen, the negative utility for any particular
decision-making herdsman is only a fraction of -1.
The sum of the utilities leads the rational herdsman to conclude that the only sensible course for him to
pursue is to add another animal to his herd, and then another, and so forth. However, this same conclusion
is reached by each and every rational herdsman sharing the commons. Therein lies the tragedy: each man
is locked into a system that compels him to increase his herd, without limit, in a world that is limited. Ruin
is the destination toward which all men rush, each pursuing his own best interest in a society that believes
in the freedom of the commons. Freedom in the commons brings ruin to all.
Hardin went on to apply the situation to modern commons. The public must deal with the overgrazing
of public lands, the overuse of public forests and parks and the depletion of sh populations in the ocean.
Individuals and companies are restricted from using a river as a common dumping ground for sewage and
from fouling the air with pollution. Hardin also strongly recommended restraining population growth.
The "Tragedy of the Commons" is applicable to the environmental problem of global warming. The
atmosphere is certainly a commons into which many countries are dumping excess carbon dioxide from
the burning of fossil fuels. Although we know that the generation of greenhouse gases will have damaging
eects upon the entire globe, we continue to burn fossil fuels. As a country, the immediate benet from the
continued use of fossil fuels is seen as a positive component. All countries, however, will share the negative
long-term eects.
3.1.3.3.4 Additional Resources

Plants can tell us about climate change


See how the general public are helping scientists monitor climate change by observing the timing of
leang, owering, and fruiting of plants (plant phenophases).
Project Budburst http://neoninc.org/budburst/ 31

Ecology site
http://www.ecology.com 32

The Story of Stu


To see how humans can aect the environment: watch The story of stu:
http://youtu.be/9GorqroigqM
31 http://neoninc.org/budburst/
32 http://www.ecology.com/

Available for free at Connexions <http://cnx.org/content/col11410/1.3>


199
3.1.3.3.5 Answers

Question 1
1.1B
1.2D
1.3B
1.4.C
Question 2
2.1.Sample site A
2.2.Sample site D
2.3.Oil
2.4.Sample B
2.5.Sample B has greater shaded portion
2.6.Regular service of oil tankers
Legislation on the amount of oil the oil tankers transport
Monitoring of travel routes
Use bacteria to dissolve the oil spill
EXCELLENT RESOURCES, QUESTIONS, ACTIVITIES IN MINDSET LEARN MA-
TERIAL!!!!!

3.1.4 3.1.4 Ecosystems33


3.1.4.1 Key concepts

In this session we will focus on summarising what you need to know about:
• Concept of an ecosystem
• Structure and ecosystem function related to abiotic factors and biotic factors
• Factors that inuence terrestrial and aquatic primary production

3.1.4.2 Terminology & Denitions

Ecosystem - a community plus the physical environment that it occupies at a given time
Abiotic factors  non-living factors . Eg soil, air.
Biotic factors  living factors. Eg trees, insects.
Producer  organism that manufactures food by photosynthesis.
Herbivore  an organism that feeds on plants only.
Carnivore  an organism that feeds on animals only.
Omnivore  an organism that feeds on both plants and animals.
Saprotroph  an organism that feeds on dead and decaying matter. (fungi)
Decomposers  an organism that causes the decay of dead and dying organisms. (bacteria)
Scavenger  an organism that feeds on remains of animals.
Predator  an animal that hunts for its food.
Prey  an animal that is hunted and is food for the predator.
Food chain  a chain showing feeding relationships between organisms.
33 This content is available online at <http://cnx.org/content/m43207/1.1/>.

Available for free at Connexions <http://cnx.org/content/col11410/1.3>


200 CHAPTER 3. ENVIRONMENTAL STUDIES

3.1.4.3 Content

An ecosystem is a community of living organisms interacting with each other and their environment. Ecosys-
tems occur in all sizes. A tidal pool, a pond, a river, and a yellowwood forest are all examples of ecosystems.
The living organisms (plants and animals) are referred to as biotic factors.The non-living components make
up the abiotic factors. Organisms living in a particular ecosystem are adapted to the prevailing abiotic and
biotic conditions.
3.1.4.4 Abiotic Components

Abiotic factors may be grouped together as:


• Physical / Climatic factors: sunlight, water, temperature and gases.
• Edaphic factors: soil type, soil pH.
• Physiographic factors: slope, aspect and altitude and latitude.

3.1.4.4.1 Climatic Factors:

3.1.4.4.1.1 Sunlight

• Light is a fundamental necessity.


• The amount of sunlight an ecosystem receives is called insolation. This is critical for plant survival,
especially in aquatic ecosystems. Light decreases and water depth increases.
• Amount of light changes from season to season.
• In summer  days are longer, ecosystems gets more hours of sunlight than during short winter days.
• The amount of sunlight aects other features of the ecosystem such as temperature, and the types of
plants and animals that are found there.

3.1.4.4.1.2 Temperature

• Temperature increases with an increase in both latitude and altitude.


• Temperature aects the rate at which photosynthesis, respiration and decomposition take place. The
higher the temperature, the faster the processes.

3.1.4.4.1.3 Water

• Water is one of the most important factors in the ecosystem.


• It is the main component of living cells and is essential for all living organisms.
• About 80% of the human body and 90% of the plant body consists of water.
• Water is evenly distributed over the earth.
• It is abundant in aquatic ecosystems and least in deserts
• Plants are adapted to the available amount of water in the following ways:
· Xerophytes are plants that are able to live in dry habitats, or in regions with low annual rainfall.
These plants are resistant to drought, have to cope with shortage of water, high temperatures and
light intensities and dry warm winds. ie Cactus, Aloe
· Hydrophytes are plants that are able to live entirely or partially in submerged water ot in very
wet soil. These plants have to cope with a water surplus. ie Water lily, Water hyacinth
· Mesophytes are plants that need an average, regular supply of water. ie Fruit trees, wheat

Available for free at Connexions <http://cnx.org/content/col11410/1.3>


201
3.1.4.4.1.4 Air / Gases

• The movement of air is called wind.


• Wind aects many things in the ecosystem ie speeds up evaporation, assist in pollination of plants and
the dispersal of their seeds.
• Air is composed of 78% Nitrogen, 21% Oxygen, 4% Carbon dioxide and water vapour.
· Oxygen  used in cellular respiration and combustion and is returned to the environment by the
process of photosynthesis.
· Carbon dioxide  is a product of cellular respiration and decayed organic matter. It is removed
from the atmosphere by plants during the process of photosynthesis.
· Nitrogen  is needed by all living organisms for the synthesis of proteins.
· Water vapour - the amount of water vapour found in the air remains constant on average, however,
it can vary greatly from one place to another. Some parts of the Earth are prone to high humidity
levels, why other locations have very dry air. Much of what we consider weather is caused by
water vapour. The clouds in the sky are largely made up of it, and it is the condensation of this
vapour into droplets that creates rain and snow.

3.1.4.4.2 Edaphic Factors:

3.1.4.4.2.1 Soil Structure

• Humus content is the decomposed remains of dead organic matter. It gives


• the topsoil its dark colour. It supplies plants with nutrients and helps the soil
• absorb (retain) water. Soils rich in humus are fertile soils.
• Texture of soil is determined by the size of the soil particles.

3.1.4.4.2.2 Soil Properties

• How acid or alkaline a soil is, can be measured by the pH scale .


• pH is a measure of the acidity or alkalinity of a solution.
• The pH scale ranges from 0 to 14. Neutral solutions have a pH value of 7 . Acid
solutions have a pH value of less than 7 and alkaline solutions greater than 7 .
• Litmus paper or universal indicator can be used to determine whether a solution is acid or
alkaline.

3.1.4.4.3 Physiographic Factors:

3.1.4.4.3.1 Aspect

• Refers to the position of an area in relation to the sun or wind or wave action.
• It is the direction that the slope faces ie North, South, East, West .
• In Southern Hemisphere -
· North facing habitats receive:
* more solar radiation than a South facing one.
* are warmer and drier
* greater variety of animals
· South facing habitats are:
* cooler
* have a higher water capacity
* plant growth is luxurious

Available for free at Connexions <http://cnx.org/content/col11410/1.3>


202 CHAPTER 3. ENVIRONMENTAL STUDIES

3.1.4.4.3.2 Slope

• Also referred to as the gradient which represents the steepness of a slope.


• Aects the rate of water run-o
· Ie Steep slope  encourages fast run-o of water and causes soil erosion. Soils tend to be shallow
and infertile and plant growth is reduced. Plants are small and few animals present
• Gentle slope  favours slower ow of surface water, reduces erosion, and increases availability of
water to plants.

3.1.4.4.3.3 Altitude

• Is the height of the land above sea level.


• At high altitudes :
· the temperature is lower ,
· the wind speed is greater,
· and the rainfall less and more likely to get snow.
· Less plant and animal species found
· Plants more stunted than at lower elevations.

Available for free at Connexions <http://cnx.org/content/col11410/1.3>


203
3.1.4.4.4 BIOTIC COMPONENTS

Figure 3.27

Living organisms in an ecosystem are usually grouped according to how they obtain food.
• Autotrophs that make their own food are known as producers, while
• Heterotrophs that eat other organisms, living or dead, are known as consumers.
• The producers include land and aquatic plants, algae and microscopic phytoplankton in the ocean.

Available for free at Connexions <http://cnx.org/content/col11410/1.3>


204 CHAPTER 3. ENVIRONMENTAL STUDIES

• They all make their own food by using chemicals and energy sources from their environment.
For example, plants use photosynthesis to manufacture sugar (glucose) from carbon dioxide and water.
Using this sugar and other nutrients (e.g., nitrogen, phosphorus) assimilated by their roots, plants produce
a variety of organic materials. These materials include: starches, lipids, proteins and nucleic acids. Energy
from sunlight is thus xed as food used by themselves and by consumers.
The consumers are classed into dierent groups depending on the source of their food. Herbivores (e.g.
deer, squirrels) feed on plants and are known as primary consumers. Carnivores (e.g. lions, hawks, killer
whales) feed on other consumers and can be classied as secondary consumers. They feed on primary
consumers. Tertiary consumers feed on other carnivores. Some organisms known as omnivores (e.g., bears,
rats and humans) feed on both plants and animals. Organisms that feed on dead organisms are called
scavengers (e.g., vultures, ants and ies). Detritivores (detritus feeders, e.g. earthworms, termites, crabs)
feed on organic wastes or fragments of dead organisms.
Decomposers (e.g. bacteria, fungi) also feed on organic waste and dead organisms, but they digest the
materials outside their bodies. The decomposers play a crucial role in recycling nutrients, as they reduce
complex organic matter into inorganic nutrients that can be used by producers. If an organic substance can
be broken down by decomposers, it is called biodegradable.
In every ecosystem, each consumer level depends upon lower-level organisms (e.g. a primary consumer
depends upon a producer, a secondary consumer depends upon a primary consumer and a tertiary consumer
depends upon a secondary consumer).
3.1.4.4.4.1 Activity:

Let's study an example of an ecosystem:


(a) What is a forest?
A forest is much more than a large area of land covered with trees. Shrubs, vines, ferns, mosses and
toadstools live in the shade of trees. The forest also swarms with birds, mammals, reptiles, amphibians and
insects. A forest is therefore all these plants and animals living together.
The animals depend on plants for food, while plants use sunlight, carbon dioxide, water and minerals to
make food for themselves and other organisms in the forest.
The living organisms (plants and animals) together with the non-living environment (air, water, sun and
soil) constitute an ecosystem.
(b) Tabulate an example of each of the following found in a forest.

Species My example of...


Bird
Mammal
Reptile
Insect
Amphibian
Table 3.9

(c) Now sketch these ve examples in the drawing to complete it!
(d) The Ecosystem concept
The organism-environment interaction leads to the ecosystem concept, elaborating the interaction be-
tween matter, energy and organism.
(e) How big is an ecosystem?
An ecosystem can be of any size, from a puddle of water on the pavement to the entire rain forest in the
Amazon or an even larger area.
Available for free at Connexions <http://cnx.org/content/col11410/1.3>
205
A forest with its trees, plants, insects, birds, etc., is an ecosystem of certain kinds of organisms that
occupy a certain environment. On the other hand, a rock in the shade of a forest with its mosses and other
rock plants, insect larvae and centipedes, is also an ecosystem. The system is therefore integrated, with parts
that are intimately related to one another. Anything aecting a part of the system will also aect the rest.
Assignment 1:
(f) Study the sketch of the forest ecosystem below:
Name the:
• producer
• primary consumer
• secondary consumer
• tertiary consumer

Primary means rst.


Secondary means second.
Tertiary means the third in this case.
2. The ecosystem is the living organisms together with the
____________________________________
invisibly linked and mutually interactive.
The living part can be divided into:
(a) food producers - mainly green plants
(b) food consumers - which are macro consumers (animals which eat other organisms) or decomposers
(bacteria or fungi which break up dead organisms)
3.1.4.4.4.2 Activity:

To discuss rock pool ecosystems


This ecosystem is extremely sensitive. The main factor that plays a role here is TIDES.
The tide changes twice every 24 hours. As a result of the wave action, cold, oxygen-rich sea water washes
over the rock pool communities. During low tide the water in the rock pools heats up and evaporation takes
place. These changes contribute to the adaptation of the organisms that live in the rock pools so that they
may survive the various challenges of nature.
1. Describe what you understand by a rock pool.
2. List all the abiotic factors that have an impact on rock pools.
Plants in rock pools
The most common plants are sea-weeds or sea-algae. They are red, green or brown.
All contain chlorophyll but in some the green is masked by red / brown pigments. Although they are not
always green, they can also produce nutrients through photosynthesis. Therefore they are also producers.
They also provide nutrition for a wide range of other organisms that live o them, such as mussels and
some sea-snails.
1. Do research in order to describe what each of the following is:
(a) lter feeders:
(b) scavengers:
2. What would the purpose of tentacles be in sea-anemones?
3. What kind of mutual dependence exists between the organisms in a rock pool?
Assessment of the interpretation of the SKETCHES
Could you distinguish the basic rock pool components?
3.1.4.4.4.3 Environmental Investigation

Brief  you are going to investigate the number of grass plants growing on the lawns of the north side and
the south side of South High.
Investigate  design your investigation.

Available for free at Connexions <http://cnx.org/content/col11410/1.3>


206 CHAPTER 3. ENVIRONMENTAL STUDIES

Your method must be fair, reliable, precise, valid and accurate.


You should record your results in an appropriate table.
Write up  write up your investigation.
Use the following headings 
• Hypothesis
• Method
• Justication
• Results
• Analysis
• Evaluation

Hypothesis  Write a Hypothesis for your investigation. It is a statement of your expected results (2)
Method  Write a bullet pointed Method for your investigation procedure. Remember to include the
steps taken to make it Fair, Reliable, Precise and Valid. (8)
Justication  Explain what you did to make your experiment Fair, Reliable, Precise and Valid. (4)
Results  Display your results in a table and in a graph if appropriate. (8)
Analysis  Use maths to analyse your results and state clearly what the results show. (4)
Evaluation - Evaluate your experiment. What was good about it and why? What could you improve
and how? (6)
Practical Technique (8)
Total 40 marks
Memo: Environmental Investigation
Hypothesis  Write a Hypothesis for your investigation. What are you trying to prove? (2)
• There is a dierence/ no dierence One side has more grass plants/greater density of grass plants than
the other (2)
Method  Write a bullet pointed Method for your investigation procedure. Remember to include the steps
taken to make it Fair, Reliable, Precise and Valid. (8)
• Use random sampling/Construct square/Count stems/Record/Repeat/Find average/Repeat for other
side/in exactly the same way (Max 7)
• Bullet pointed or numbered (1)

Justication  Explain what you did to make your experiment Fair, Reliable, Precise and Valid. (4)
• Fair linked to doing the same to both sides
• Reliable linked to repeats
• Precise linked to using ruler/precise counting
• Valid linked t no bias/random
Results  Display your results in a table and in a graph if appropriate. (8)
• Caption (2)
• Table headings( 2)
• Correct data (2)
• Total and average (2)
Analysis  Use maths to analyse your results and state clearly what the results show. (4)
• Correct calculation of total shown (2)
• Correct calculation of average shown (1) AND LINKED TO HYPOTHESIS (1)

Available for free at Connexions <http://cnx.org/content/col11410/1.3>


207
Evaluation - Evaluate your experiment. What was good about it and why? What could you improve and
how? (6)
• 3 Good points mentioned (3) eg Reliable, Precise, Fair, Unbiased
• 3 Improvements mentioned (3) More samples, Bigger square, Extend to other areas
Practical Technique (8)
• Candidate fully involved in all aspects of investigation (2)
• Candidate often involved (1)
• Candidate lets others do the work (0)
• Candidate counted patiently and accurately (2)
• Candidate did some counting (1)
• Candidate did no counting (0)
• Candidate fully involved in planning (2)
• Candidate involved t some extent in planning (1)
• Candidate did no planning (0)
• Candidate ensured results were recorded properly by group (2)
• Candidate involved t some extent in recording (1)
• Candidate did no recording (0)

Total 40 marks
3.1.4.4.4.4 Investigative Project

Background information on the state of our environment


Population growth  The increase in population no's together with the world unsustainable consump-
tion pattern is putting ever-increasing stress on the land, water, air, energy and other essential resources of
the planet. Rapid population growth is usually accompanied by serious environmental degradation, including
soil erosion, desertication and deforestation.
Biological diversity  is essential for human survival. Biological resources provide food, medicines,
clothing, housing and a wide range of raw materials. They are important for the future improvement of
livestock and crops, and for the development of new medicines and products. Plants and animals are also
essential in the maintenance of soil productivity, the degradation of waste, and in pest and ood control.
Fresh water  is essential for the maintenance of life on earth. It is vital for drinking, sanitation, indus-
try, food production, urban developments, power generation, transportation inland sheries and recreation.
Adequate supplies of fresh water are available to satisfy projected worldwide demands into the twenty-rst
century if the resource is eciently used.
Energy stands as one of the most essential of all the earth's resources. Human life would not exist
without the heat, light and food that depend on it. The world consumption of energy has quadrupled in the
past ve decades. There has been a very great increase in the use of non-renewable energy sources, notably
fossil fuels, yet most energy is ineciently used.
Your presentations must focus on any of the following themes for creating a sustainable
city

• Conserving : including terrestrial, freshwater and marine ecosystems; restoration of; monitoring
and conserving plants, animals and ecosystems.
• Water conservation : monitoring and reducing water use; water harvesting & re-use
• Waste management : waste / litter audits; reducing, reusing and recycling; composting; avoid-
ing/appropriately disposing hazardous wastes
• Energy conservation : monitoring and reducing energy use; alternative energy; global climate
change
Available for free at Connexions <http://cnx.org/content/col11410/1.3>
208 CHAPTER 3. ENVIRONMENTAL STUDIES

• Environmental health : nutrition and food gardens; controlling air and water pollution; hygiene
and sanitation; creating a healthy environment
• Transport : reducing the use of fossil fuels and pollution through public / shared transport, walking
and cycling programmes,
Procedural Guideline :

• Decide on a topic. Choose a topic in which you have a particular interest and about which you would
like to know more. It is quite probable that your eld of interest has nothing to do with your school
subject syllabus. The important thing is the investigative process that you follow. Get ideas from
magazines/ TV/ attached list.
• Find a question that needs an answer. Read available literature as background information. Start
planning your investigations/experiments that will hopefully answer your question.
• Now consult with your teacher and possibly some expert in the eld. Phone a relevant Com-
pany/University/Technicon Department. Ask for advice. Be polite when asking!!
• Your typed report:

1. This should be a detailed account of your research and investigation .


2. It should be neat , attractive and illustrated if possible. Concise. Use photocopies, scanned
pictures or preferably use your own illustrations.
3. Computer printed reports, using graphics are recommended.
4. The format for the report should be as follows:
TITLE : Name of your project
AIM : What problem are you trying to solve?
METHOD : Show programmes followed to solve the problem. Include experiments, clearly illustrated.
This could be done in a diary format.
RESULT : Preferably tabulate results of experiments and observations. Include graphs of results, if
appropriate.
CONCLUSIONS : Very important
THEORY : Any theory, which needs explanation, can be included at any stage.
APPLICATION : Can these conclusions have any practical application in everyday life?
FURTHER RESEARCH: Very often your research opens up further questions related to your topic
which someone else might like to investigate.
ACKNOWLEDGEMENTS AND REFERENCES :
Include titles of books used, publishers and pages. Include names of people who advised and helped you.
Be honest in this regard.
ASSESSMENT:
PRESENTATION
Neatness10
Visual impact10
CONTENT
Factual accuracy10
Adequate coverage10
EXPERIMENTAL WORK/RESEARCH
Technique and controls10
Display of results10
Interpretation of results10
VERBAL COMMUNICATION
Factual accuracy10
Insight and understanding10
EVIDENCE OF PERSONAL GROWTH
Available for free at Connexions <http://cnx.org/content/col11410/1.3>
209
Enjoyment and learning experience5
PLANNING5
TOTAL100
DATES AND DEADLINES
PLANNING - Decide on topic. You must hand in a written report on what you are doing and how you
are planning to set about it. You will be penalised if your report is late. (This may be written)
DUE:
PROGRESS - You need to write about how far you've got with your
research, any complications/problems encountered, how you
propose to move on from there. . .
DUE:
FINAL PROJECT DUE:
3.1.4.4.4.5 Rich media

INTERACTIVE QUIZ http://www.oercommons.org/courses/ecology-quiz 34

This link is useful for consolidating the concepts of ecology.


http://www.oercommons.org/courses/ecology-and-the-ecosystem-quiz 35

This link is useful for consolidating the concepts of ecosystems.


http://www.oercommons.org/courses/ecological-footprint-calculator In this exercise, you will utilize
36

an online calculator to examine your ecological footprint, compare it to the average footprint in your country
and other countries, and critically examine ways to reduce it.
VIDEO: http://www.curriki.org/xwiki/bin/view/Coll_NROCscience/APBiologyIIChapter33Popul
ationsandEcosystems 37

Video on the study of the interaction between organisms and their environ-
ment. http://www.curriki.org/xwiki/bin/view/Coll_NROCscience/APBiologyIIChapter33Lesso
n64CommunityEcology?bc=;Coll_NROCscience.APBiologyIIChapter33PopulationsandEcos ystems 38

Video that examines the types of interactions that exist among organisms in a community, from
competition between consumers for specic resources to the relationship between predators and prey.
http://www.curriki.org/xwiki/bin/view/Coll_NROCscience/APBiologyIIChapter34Ecosy stems In this 39

Interactive Exercise, you will use a marine ecosystem to investigate the dynamic nature of ecosystems. You
will explore the various regions of the bay, and then investigate some of the natural and human-imposed
changes to which the bay responds.
3.1.5 3.1.5 - Energy ow40
3.1.5.1 Energy Flow and Nutrient Cycles

Organisms such as plants and animals need energy for growth, movement and reproduction. They get this in
the form of nutrients from the food they eat. The main source of energy for life on earth is the sun. The sun
provides energy to producers that use photosynthesis to grow and become food for consumers. Consumers
include herbivores, carnivores and omnivores. Decomposers break down discarded plant and animal (organic)
materials into simpler substances, which returns nutrients to the soil and atmosphere for new plants to use
to grow.
34 http://www.oercommons.org/courses/ecology-quiz
35 http://www.oercommons.org/courses/ecology-and-the-ecosystem-quiz
36 http://www.oercommons.org/courses/ecological-footprint-calculator
37 http://www.curriki.org/xwiki/bin/view/Coll_NROCscience/APBiologyIIChapter33PopulationsandEcosystems
38 http://www.curriki.org/xwiki/bin/view/Coll_NROCscience/APBiologyIIChapter33Lesson64CommunityEcology?bc=;Coll_NROCscience.AP
39 http://www.curriki.org/xwiki/bin/view/Coll_NROCscience/APBiologyIIChapter34Ecosystems
40 This content is available online at <http://cnx.org/content/m43176/1.1/>.

Available for free at Connexions <http://cnx.org/content/col11410/1.3>


210 CHAPTER 3. ENVIRONMENTAL STUDIES

3.1.5.1.1 Food chain

A food chain is a series of nutrients and energy moving through a chain of organisms. Below is an example of
a simple food chain in a grassland. The arrows show the movement of energy from one organism to another.

Figure 3.28

IMAGES sourced from: http://www.ickr.com/photos/blueridgekitties/4625665988/sizes/o/in/photostream/


http://www.ickr.com/photos/zest-pk/924783392/sizes/m/in/photostream/
http://www.ickr.com/photos/e3000/5922771249/sizes/l/in/photostream/
3.1.5.1.1.1 Activity 1:

Can you trace a food chain of the vegetables, fruit, cheese, eggs or meat that you had for breakfast or will
have for dinner?
3.1.5.1.1.2 Activity 2:

1. In the food chain shown above which of the three organisms is the
a. Herbivore
b. Carnivore
c. Producer
Available for free at Connexions <http://cnx.org/content/col11410/1.3>
211
2. Draw a food chain showing at least 4 organisms.
3. Producers use sunlight to manufacture their own food. Write a word equation as well as a balanced
equation to depict this process.
4. Draw in the decomposers in the above food chain. Ensure that the direction of the arrows is correct.
5. What animal will feed on the leopard?

3.1.5.1.2 Food web

A food web is made up of a number of food chains. It represents the dierent feeding relationships in an
ecosystem or a biome. It is usually more complicated than a food chain because organisms can get their
energy or food from more than one source. The presence of a number of food sources makes the system more
stable. If one organism is removed, the whole system will not collapse, unlike in a single food chain.

Figure 3.29

(image source: http://per8eocreview.wikispaces.com/ )


3.1.5.1.2.1 Activity 3: Human Food Web

1. Divide into teams of at least ve students each and stand in a circle.
2. One person in each group takes a ball of wool or string. This person is the sun and starts the food
web.
3. The rst person (sun) holds onto the end of the wool and throws the ball of string to another person
in the group.
4. The person who catches the ball has to name themselves something that uses energy from the sun
(primary producer).
Available for free at Connexions <http://cnx.org/content/col11410/1.3>
212 CHAPTER 3. ENVIRONMENTAL STUDIES

5. The person holding the ball (primary producer) has to hold the string with one hand and toss the ball
to another student in the circle with his/her other hand.
6. The person who catches the ball has to name something that eats or is eaten by the previous organism
named.
7. Carry on until everyone in the circle is holding the ball. You can throw the ball to someone who has
already named themselves, as long as they eat you or are eaten by you
8. Look at the web you have created and the ones the other groups have created.
9. Are some webs more complex than others? Why?
(source: http://www.oercommons.org/courses/got-energy-spinning-a-food-web/view)
3.1.5.1.3 Trophic levels and the Food Pyramid:

The trophic level of an organism is the position it holds in a food chain and depends on how much energy
it consumes or produces. The trophic level of each organism can be drawn as a pyramid starting with the
producers at the bottom and moving up through the food chain.
The organism at the bottom gives the most energy and needs the least and the organism at the top needs
the most energy and releases the least. Energy is lost from activities at every level - through heat, egestion,
urination and reproduction (pregnancy and egg-laying). This is why there is less and less energy as you
move up the pyramid.
Producers, eg. Plants are on the rst level, or bottom of the pyramid, because they produce their own
nutrients using energy from the sun and therefore have a lot of energy to pass on.
Primary Consumers, eg. Herbivores are on the second level because they feed o plants
Secondary Consumers, eg. Carnivores feed on herbivores so they get their energy from plants indirectly
and are on the third level.
Tertiary Consumers, eg. Carnivores feed on organisms below them in the pyramid
(image source: http://per8eocreview.wikispaces.com/)
Figure 3. Food pyramid
Trophic levels can be drawn as a pyramid of numbers, where each level shows the number of organisms
(look at gure 3). They can also contain the biomass of a population. The biomass is the mass of living
organisms in an ecosystem.
3.1.5.1.3.1 Activity 4:

Look at the food web and the diagram showing the dierent trophic levels.
1. Identify a food chain that has three trophic levels.
2. Identify a food chain that has four trophic levels.
3. Name 2:
a. Producers
b. Primary consumers
c. Secondary consumers
d. Tertiary consumers
4. There are very few tertiary consumers compared to the primary consumers. Why?
5. What will happen if the impala is removed from the food web?
(source: http://www.learner.org/courses/envsci/interactives/ecology/ )
VIDEO: http://www.youtube.com/watch?v=TE6wqG4nb3M
This a catchy song about food chains to help you remember.
VIDEO: http://www.youtube.com/watch?v=-YwW-iWxLr4&NR=1
Bill Nye the Science Guy talks about the Food Web

Available for free at Connexions <http://cnx.org/content/col11410/1.3>


213
3.1.6 3.1.6 Nutrient Cycles41
3.1.6.1 Nutrient cycles

Organisms rely on nutrients in order to survive. These include carbon, oxygen, nitrogen, water and
mineral salts.
These nutrients need to be cycled through the ecosystem so that they can be re- used. This is called
nutrient recycling . In biology, this is the movement of nutrients from the physical environment into living
organisms and back into the environment. The ow of energy you saw before from the sun to herbivores
and then to carnivores is part of this process. In any environment the ow of nutrients must be stable and
balanced so that organisms can survive. If the cycle stops at any point, nutrients will become locked in place
and cannot be used in the next step.
The water cycle, carbon cycle, oxygen cycle and nitrogen cycle are examples of nutrient
re-cycling.
Video: http://www.youtube.com/watch?v=AXWDbAYb-5c 42

Here is a simple video explaining nutrient cycling


3.1.6.1.1 Water

(ex http://cnx.org/content/m16470/latest/?collection=col10548/latest )
The earth is sometimes known as the "water planet" because over 70 percent of its surface is covered by
water. All living organisms need water for their survival.
In this cycle, water is transported between water reservoirs in the environment and living organisms.
This happens through these processes:
• Liquid water in oceans and lakes is converted to water vapour by evaporation . This is caused by
heating of the water by the sun. The water vapour rises into the atmosphere.
• Plants release water into the air as vapour during photosynthesis, this is called transpiration .
• When water vapour in the atmosphere cools, it can transform into tiny droplets of liquid water to form
clouds by the process of condensation
• When condensed water droplets grow so large that the air can no longer support them against the pull
of gravity, they fall to the earth through precipitation . If the water droplets fall as liquid, it is
called rain. If the temperature of the surrounding air is cold enough to freeze the water droplets, then
the water falls as snow, sleet or hail.
• Water that falls onto the earth runs into lakes, rivers or oceans.
41 This content is available online at <http://cnx.org/content/m43195/1.1/>.
42 http://www.youtube.com/watch?v=AXWDbAYb-5c

Available for free at Connexions <http://cnx.org/content/col11410/1.3>


214 CHAPTER 3. ENVIRONMENTAL STUDIES

43
Figure 3.30: (image source: http://per8eocreview.wikispaces.com/ )The Water Cycle.

Animation: http://www.epa.gov/ogwdw/kids/ash/ash_watercycle.html 44

This is an animation of the water cycle


3.1.6.1.2 Oxygen

Oxygen is one of the main gases found in the air, along with nitrogen.
Oxygen is re-cycled between the air and living organisms in the following ways:
• Organisms take in oxygen during respiration , which they use for cellular processes to break down
energy rich nutrients.
• When wood or fossil fuels burn, they consume oxygen and release carbon dioxide and water into the
atmosphere through combustion .
• Plants release oxygen into the air as a by-product of photosynthesis .

43 http://per8eocreview.wikispaces.com/
44 http://www.epa.gov/ogwdw/kids/ash/ash_watercycle.html

Available for free at Connexions <http://cnx.org/content/col11410/1.3>


215
Because animals trap oxygen during respiration, the release of oxygen by plants during photosynthesis is the
main way oxygen is released into the atmosphere.

45
Figure 3.31: (image source: http://gellertclass.wikispaces.com/Chapter+3 )The Oxygen Cycle.

Video: http://www.youtube.com/watch?v=mUVX5rg1E0I 46

This is a video explaining the oxygen cycle


3.1.6.1.3 Carbon

Carbon is the basic building block of all organic materials, and therefore, of living organisms. Most of the
carbon on earth can be found in the crust. Other reservoirs of carbon include the oceans and atmosphere.
Carbon moves from one reservoir to another by these processes:
• Combustion of wood and fossil fuels transfers carbon to the atmosphere as carbon dioxide.
45 http://gellertclass.wikispaces.com/Chapter+3
46 http://www.youtube.com/watch?v=mUVX5rg1E0I

Available for free at Connexions <http://cnx.org/content/col11410/1.3>


216 CHAPTER 3. ENVIRONMENTAL STUDIES

•Carbon dioxide is taken up by plants during photosynthesis and gets converted into energy rich
sources, such as glucose, that contain carbon.
• Animals eat plants for food, taking up the carbon. They release carbon into the atmosphere as carbon
dioxide during respiration .
• Organisms convert carbon into organic molecules like fats, carbohydrates and proteins when they eat
plants or animals.
• Carbon dioxide in the atmosphere can also precipitate as carbonate in ocean sediments. These
ocean sediments are melted by the movement of tectonic plates and then returned to the surface
during volcanic activity.
• Carbon dioxide gas is released into the atmosphere during volcanic eruptions.

Photosynthesis and respiration are the main carbon cycling processes involving living organisms.

Available for free at Connexions <http://cnx.org/content/col11410/1.3>


217

47
Figure 3.32: (image source: http://per8eocreview.wikispaces.com/ )The Carbon Cycle

GAME: http://www.windows2universe.org/earth/climate/carbon_cycle.html 48

This is a game you can play to learn more about the carbon cycle
3.1.6.1.4 Nitrogen

Nitrogen (N2) makes up most of the gas in the atmosphere (about 78%). Nitrogen is important to living
organisms and is used in the production of amino acids, proteins and nucleic acids (DNA, RNA). Only a
few single-cell organisms, like bacteria can use nitrogen from the atmosphere directly. For multi-cellular
47 http://per8eocreview.wikispaces.com/
48 http://www.windows2universe.org/earth/climate/carbon_cycle.html

Available for free at Connexions <http://cnx.org/content/col11410/1.3>


218 CHAPTER 3. ENVIRONMENTAL STUDIES

organisms, like plants and animals, nitrogen has to be changed into other forms, eg. nitrates or ammonia.
This process is known as nitrogen xation .
The nitrogen cycle involves these steps:
During decomposition , bacteria and fungi break down proteins and amino acids from plants and
animals into nitrogen in the form of ammonia (NH3) by the process of ammonication and convert the
ammonia to nitrate (NO3-) by nitrication .
Nitrogen can be changed to nitrates directly by lightning . The rapid growth of fungi and algae after
thunderstorms is because of this process, which increases the amount of nitrates that fall onto the earth in
rain water, acting as fertilizer.
Ammonia and nitrates are absorbed by plants through their roots.
Humans and animals get their nitrogen supplies by eating plants or plant-eating animals.
The nitrogen is returned to the cycle when bacteria decompose the waste or dead bodies of these
higher organisms, and in the process, convert organic nitrogen into ammonia.
In a process called denitrication , other bacteria convert ammonia and nitrate into nitrogen and
nitrous oxide (N O). Nitrogen is returned to the atmosphere to start the cycle over again.
2

Figure 3.33: The Nitrogen Cycle

SIMULATION: http://www.teachersdomain.org/asset/lsps07_int_nitrogen/
You can play with this simulation to learn more about the Nitrogen cycle.
Animation:
Available for free at Connexions <http://cnx.org/content/col11410/1.3>
219
Here are some animations of the nitrogen cycle:
http://www.mhhe.com/biosci/genbio/tlw3/eBridge/Chp29/animations/ch29/1_nitrogen_ cycle.swf 49

https://www.classzone.com/books/ml_science_share/vis_sim/em05_pg20_nitrogen/em05
_pg20_nitrogen.html 50

3.1.7 3.1.7 Ecotourism51


Ecotourism Tourism in natural environments to observe wildlife, often that are under protection or contain
endangered species. It also refers to the practise of travelling to areas in order to support conservation eorts
and uplift the lives of local people.
3.1.7.1 The attractions of touring South Africa

South Africa is a beautiful country that boasts great diversity in its ora and fauna. There are many
interesting cultural, historical and environmental place that people from South Africa and other countries
want to visit.
From what you learned from the dierent ecosystems, you can see that South Africa has a range of
systems from desert, wetland, mountains, sea and our own unique Fynbos biome.
South Africa encompasses about 1,200,000 km and has about 10% of all plant species on Earth. It is
2

the third most biodiverse country in the world, and together with seventeen other countries, is considered
mega diverse which means those countries contain 70% of the planet's biodiversity. South Africa's unique
geography allows the country to support such a diverse population of plants and animals. This makes South
Africa an interesting travel destination to many.
3.1.7.2 Benets to visitors, locals and the environment

Eco-tourism is a mutually benecial practice for visitors, locals and the environment.
Eco-tourism has the potential to alleviate poverty in South Africa through bringing money into the
economy and creating jobs for locals, while at the same time turning our great biodiversity and natural
resources into a national asset that will be nurtured, protected and grown. Tourism is the fastest growing
part of the South African economy. In fact, tourism generates more money in South Africa than gold mining.
3.1.7.3 Ethical Issues

While tourism has great economic potential and gives people access to unique places and cultures, it can
have a negative impact. Sensitive ecosystems such as wetlands and coasts need to be protected so that the
balance of organisms can be maintained. Too many visitors and visitors who are not informed about their
impact on the environment can have a harmful eect. In the same way tourists need to be sensitive to the
cultures and people that they visit.
To protect the plants and animals in the unique ecosystems of South Africa, many areas have been declared
National Parks and have strict rules about how to behave. You can visit the South African National Parks
website to learn more about them: www.sanparks.org 52

In the same way, places that are historically or culturally important have been declared national heritage
sites that are protected and maintained. South Africa is also proud to have eight UNESCO (The United
Nations Educational, Scientic and Cultural Organisation) sites:
CULTURAL
• Fossil Hominid Sites of Sterkfontein, Swartkrans, Kromdraai, and Environs (1999)
• Mapungubwe Cultural Landscape (2003)
• Robben Island (1999)
49 http://www.mhhe.com/biosci/genbio/tlw3/eBridge/Chp29/animations/ch29/1_nitrogen_cycle.swf
50 https://www.classzone.com/books/ml_science_share/vis_sim/em05_pg20_nitrogen/em05_pg20_nitrogen.html
51 This content is available online at <http://cnx.org/content/m43191/1.1/>.
52 http://www.sanparks.org/

Available for free at Connexions <http://cnx.org/content/col11410/1.3>


220 CHAPTER 3. ENVIRONMENTAL STUDIES

• Richtersveld Cultural and Botanical Landscape (2007)


MIXED
• UKhahlamba / Drakensberg Park (2000)
NATURAL
• Cape Floral Region Protected Areas (2004)
• Greater St. Lucia Wetland Park (1999)
• Vredefort Dome (2005)
You can nd out more about them here: http://www.sa-venues.com/unesco_world_heritage_sites.htm
53

3.1.7.4 How to be a responsible ecotourist

Many areas of South Africa are protected and to travel to these areas you need to respect the area and the
people that you are visiting. These are a few tips:
• Learn a little about the place you are visiting before you go to know the do's and don'ts. For example,
littering is not allowed in any National Park in South Africa.
• South Africa is rich in cultural diversity, which means that people from dierent areas have dierent
ways of doing things. Learn about the culture of local people so that you can make sure not to oend
anyone by your behaviour.
• When you are in a protected area, do not damage plants or animals or buildings. For example, writing
grati on historical buildings or sites. Remember the saying take only pictures, leave only footprints.
http://ethemes.missouri.edu/themes/1382 54

This an interesting website where you can learn about being a responsible tourist

53 http://www.sa-venues.com/unesco_world_heritage_sites.htm
54 http://ethemes.missouri.edu/themes/1382

Available for free at Connexions <http://cnx.org/content/col11410/1.3>


221
3.1.8 3.1.8 Summary55
3.1.8.1 Summary

Biosphere:
The biosphere is the region of the earth that encompasses all living organisms: plants, animals and
bacteria. The biosphere includes the outer region of the earth (the lithosphere) and the lower region of the
atmosphere (the troposphere) as well as the lakes, oceans, streams, ice and clouds comprising the earth's
water resources (hydrosphere).
Lithosphere:
The layer of the mantle above the atmosphere plus the entire crust make up a region called the lithosphere.
Hydrosphere:
The Hydrosphere contains all the water on Earth. As groundwater, the hydrosphere penetrates the soil
as far down as bedrock. It is found in aquifers as groundwater and also between soil particles. As surface
water, it is found in wetlands, marshes, estuaries, lakes, streams, rivers, lakes, seas, and oceans. In the
atmosphere, water is found as a gas throughout the dierent regions.
Atmosphere:
The gaseous layer that surrounds the earth. It extends outward to about 1,000 kilometers where it
transitions to interplanetary space.
Biomes Biomes are, the categories into which ecologists organise similar communities of plants, animals,
and the environmental conditions in which they live. The four major types of biomes are aquatic, grasslands,
forests, and desert. Aquatic biomes are probably the most important of all the biomes. Their medium, water,
is a major natural resource.
The distribution of plants and animals around the world is anything but random. Instead, it is a result of
the interplay of individual environmental tolerances of species and the environmental conditions, especially
variations in temperature and precipitation. These interactions result in
Enviroment
In this unit we explore the dierent abiotic and biotic factors that interact or the relationships within an
ecosystem. Also we have included a variety of possible investigations and activities to explore each factor
within any given ecosystem.
Energy ow
In this unit, you have learnt that organisms get their energy either directly or indirectly from the sun.
All organisms t together in a complicated food web depending on whether they supply energy or get
energy from each other. Food webs are made up of food chains that show the ow of energy from producers
(plants) to consumers (animals) to decomposers (bacteria).
The ow of nutrients between the atmosphere and organisms is called nutrient cycling.The four important
cycles for life on earth are the water, carbon, oxygen and nitrogen cycles.

55 This content is available online at <http://cnx.org/content/m43081/1.1/>.

Available for free at Connexions <http://cnx.org/content/col11410/1.3>


222 CHAPTER 3. ENVIRONMENTAL STUDIES

Available for free at Connexions <http://cnx.org/content/col11410/1.3>


Chapter 4

Diversity, change and continuity

4.1 Biodiversity and classication


4.1.1 4.1.1 Classication Schemes1
4.1.1.1 Biodiversity and Continuity

4.1.1.1.1 Classication Schemes

Image of classication:
http://www.tutorvista.com/content/biology/biology-iii/animal-kingdom/animal- classication.php 2

AP Biology: Microbe evolution and classication video


http://outreach.mcb.harvard.edu/materials.htm 3

Articial classication systems, such as the grouping of vehicles into those that provide transport on land
/ water / air etc., are based on arbitrary groupings and have little meaning. The biological classication
system, however, is based on research in biology, chemistry, genetics, etc. It is a scientic method of
classication used in biology to group similar organisms that share common features and is and is more
universally accepted.
It is always necessarily hierarchical where the important features inherited from a common ancestor
determine the group in which the organisms are placed. For example humans and whales both feed their
young on milk and it is a characteristic inherited from a common ancestor which places them under the same
class mammals even though their habitat is completely dierent.
Each organism is grouped into one of 5 large groups or kingdoms , which are subdivided into smaller
groups called phyla (singular: phylum) and then smaller and smaller groups with other names.
Kingdom
Phylum
Class
Order
Family
Genus
Species
4.1.1.1.2 History of Classication

Aristotle a 4th century (384 to322 BC) Greek philosopher divided organisms into two main groups namely
plants and animals. His system was used into the 1600's. People who wrote about animals and plants either
1 This content is available online at <http://cnx.org/content/m43187/1.1/>.
2 http://www.tutorvista.com/content/biology/biology-iii/animal-kingdom/animal-classication.php
3 http://outreach.mcb.harvard.edu/materials.htm

Available for free at Connexions <http://cnx.org/content/col11410/1.3>

223
224 CHAPTER 4. DIVERSITY, CHANGE AND CONTINUITY

used their common names in various languages or adopted more-or-less standardized descriptions. Caspar
Bauhin (15601624) took some important steps towards the binomial system by modifying many of the Latin
descriptions to two words.

Figure 4.1

Carolus Linnaeus (Carl Von Linne) was an 18th century (17071778) Swedish botanist and physi-
cian. He classied plants and animals according to similarities in form and divided living things into two
main kingdoms namely - plant and animal kingdoms. He named the plants and animals in latin or used
latinised names in his book Species Plantarum (1753) and Systema Naturae (1758).
You can watch a video about Carolus Linnaeus at Photo of Carolus Linnaeus
http://www.youtube.com/watch?v=Gb_IOSzLgk&#38;feature=related
Since Latin was once the universal language of science among western scholars in medieval Europe these
names were typically in Latin. Latin is used to name and classify living organisms, since it is a dead language
- it's no longer changing and is regarded as international.
His classication system is still used today; however, we use a ve kingdom system instead of two kingdom
system.
He designed a scientic system of naming organisms called binomial nomenclature ("bi-two, nomial-
names. He gave each organism a two part scientic name - genus (plural = genera) and species (plural
= species) names. The genus and species names would be similar to your rst name and surname. Genus
name is always written with a capital letter whereas species name is written with a small letter. Species
name belongs to that specic type of organism which are only able to interbreed and produce fertile ospring.
The scientic name must always be either underlined or written in italics.
For example the scientic name of the African elephant is...

Available for free at Connexions <http://cnx.org/content/col11410/1.3>


225

Figure 4.2: Loxodonta africana

Genus Species
http://www.ickr.com/photos/12333120@N00/3679975496/sizes/l/in/photostream/ 4

An organism will always have only one scientic name even though they might have more than one
common name. For example  Blue crane, indwe (for amaXhosa) and mogolori (for Batswana) are all
common names for South Africa's national bird. However, it has got only one scientic name which is
Anthropoides paradiseus.
The scientic name of our human race is Homo sapiens . We are the only surviving members of the
genus Homo  other more ancient / ancestral types have all become extinct, such as Homo ergaster and
Homo neanderthalensis.

4.1.1.1.2.1 Prokaryotes and eukaryotes

Prokaryotes are uni- or multicellular organisms made up of cells that do not have a nuclear envelope (pro-,
before, karyon, nucleus). The genetic material is not bound in a nucleus. They also lack cell organelles
such as an endoplasmic reticulum, a Golgi apparatus, lysosomes, and mitochondria. Prokaryotes are divided
into two main groups namely the Bacteria and the Archaea (ancient bacteria).
Eukaryotes are multicellular organisms made up of cells (eu-, true, karyon, nucleus) that possess a
membrane- bound nucleus (that holds genetic material) as well as membrane-bound cell organelles. Genetic
material in eukaryotes is contained within a nucleus. Eukaryotic organisms include organisms such as plants,
animals, fungi, and protists.
Table: Dierences between prokaryotes and eukaryotes.
Eukaryotes

Small cells Large cells


Unicellular or multicellular Often (but not always) multicellular
Genetic material is not contained within a nucleus Genetic material is contained in a membrane-bound
nucleus
4 http://www.ickr.com/photos/12333120@N00/3679975496/sizes/l/in/photostream/

Available for free at Connexions <http://cnx.org/content/col11410/1.3>


226 CHAPTER 4. DIVERSITY, CHANGE AND CONTINUITY

Lacks cell organelles such as endoplasmic reticulum, a Golgi apparatus, lysosomes, and mitochondria
Has cell organelles
Divided into bacteria and Archea Divided into protists, fungi, plants and animals.

Eukaryotes

Small cells Large cells


Unicellular or multicellular Often (but not always) multicellular
Genetic material is not contained within a nucleus Genetic material is contained in a membrane-bound
nucleus
Lacks cell organelles such as endoplasmic reticulum, a Golgi apparatus, lysosomes, and mitochondria
Has cell organelles
Divided into bacteria and Archea Divided into protists, fungi, plants and animals.

Eukaryotes

Small cells Large cells


Unicellular or multicellular Often (but not always) multicellular
Genetic material is not contained within a nucleus Genetic material is contained in a membrane-bound
nucleus
Lacks cell organelles such as endoplasmic reticulum, a Golgi apparatus, lysosomes, and mitochondria
Has cell organelles
Divided into bacteria and Archea Divided into protists, fungi, plants and animals.

Prokaryotes Eukaryotes

Small cells Large cells


Unicellular or multicellular Often (but not alw
Genetic material is not contained within a nucleus Genetic material is
Lacks cell organelles such as endoplasmic reticulum, a Golgi apparatus, lysosomes, and mitochondria Has cell organelles
Divided into bacteria and Archea Divided into protis
Table 4.1

Available for free at Connexions <http://cnx.org/content/col11410/1.3>


227
4.1.2 4.1.2 Five kingdom system5
4.1.2.1 Five kingdom system

This is the most common way of grouping living things based on simple distinctive characteristics. Clas-
sication systems are always changing as new information is made available. Modern technologies such as
electron microscopy make it possible to observe microscopic organisms in greater detail. The current system
was developed by Robert H. Whittaker in 1969 and was built on the work of previous biologists such as
Carolus Linnaeus. The highest grouping is called a kingdom.
Five kingdoms: http://www.tutorvista.com/content/biology/biology-iii/modern-classication
/ve-kingdom-classication.php 6

Bug scope: Images of microscopic organisms http://bugscope.beckman.uiuc.edu/ 7

Neok12: Animals and wildlife videos http://www.neok12.com/Animals-Wildlife.htm 8

Encyclopedia of life: Images and explanations of terms http://eol.org/index 9

Living things can be classied into ve major kingdoms:


5 This content is available online at <http://cnx.org/content/m43221/1.1/>.
6 http://www.tutorvista.com/content/biology/biology-iii/modern-classication/ve-kingdom-classication.php
7 http://bugscope.beckman.uiuc.edu/
8 http://www.neok12.com/Animals-Wildlife.htm
9 http://eol.org/index

Available for free at Connexions <http://cnx.org/content/col11410/1.3>


228 CHAPTER 4. DIVERSITY, CHANGE AND CONTINUITY

Figure 4.3

Kingdom Monera (Bacteria)


Kingdom Protista
Kingdom Fungi
Kingdom Plantae
Kingdom Animalia
http://www.youtube.com/watch?v=5uJ8QeFRvJA&feature=related 10
A video showing a brief sum-
mary of the ve kingdoms
10 http://www.youtube.com/watch?v=5uJ8QeFRvJA&feature=related

Available for free at Connexions <http://cnx.org/content/col11410/1.3>


229
4.1.2.1.1 Distinctive Features of the ve kingdoms

4.1.2.1.1.1 Kingdom Monera

• Prokaryotic, unicellular.
• No nuclear membrane or membrane bound organelles such as
chloroplasts, Golgi complex, mitochondria and endoplasmic reticulum.
• Have a cell wall made without cellulose.
• Reproduction is mainly asexual by binary ssion.
• Important examples: Archaea, cyanobacteria (blue-green algae), bacteria

Interesting fact: Bacteria are found everywhere and are the most numerous organisms on Earth. In a single
gram of soil, there are about 40 million bacterial cells. The human body also contains 10 times as many
bacterial cells as human cells!!
4.1.2.1.1.2 Kingdom Protista

• Eukaryotic, can be unicellular or simple multicellular.


• Reproduction can be asexual or sexual.
• Important examples: Plasmodium (causes malaria), amoeba, euglena

4.1.2.1.1.3 Kingdom Fungi

• Eukaryotic, multicellular (some unicellular like yeasts).


• Have a cell wall made of chitin.
• Non-motile with long extensions called hyphae.
• Nutrition is heterotrophic: important as decomposers (saprophytes), can be parasitic.
• Store food as glycogen
• Reproduction is by spore formation (both asexual and sexual).
• Important examples: Mushrooms, Penicillium (a fungus which was used to make penicillin), bread
mould
http://blog.ted.com/2008/05/06/paul_stamets/ A TED video on the many uses of Fungi
11

4.1.2.1.1.4 Kingdom Plantae

• Eukaryotic, multicellular.
• Distinct cell wall made of cellulose.
• Have plastids and photosynthetic pigments such as chlorophyll.
• Non-motile.
• Nutrition is autotrophic (make their own food by photosynthesis).
11 http://blog.ted.com/2008/05/06/paul_stamets/

Available for free at Connexions <http://cnx.org/content/col11410/1.3>


230 CHAPTER 4. DIVERSITY, CHANGE AND CONTINUITY

• Sexual reproduction.
• Important examples: Green algae, mosses, ferns, conifers, owering plants.

4.1.2.1.1.5 Kingdom Animalia

• Eukaryotic and multicellular but have no cell wall or photosynthetic pigments


• Mostly motile
• Heterotrophic nutrition.
• Sexual or vegetative (asexual) reproduction
• Important examples: Porifera (sponges), Mammalia, Insects
Additional resource:
• Tree of life project: collaborative eort of biologists and nature enthusiasts from around the world
providing information about biodiversity, the characteristics of dierent groups of organisms, and their
evolutionary history ( phylogeny ). Link: http://tolweb.org/tree/phylogeny.html
12 13

• ARKive project: For pictures and information on a wide range of life forms
• http://bugscope.beckman.uiuc.edu/ : For high magnication pictures of insects using a scanning
14

electron microscope.
• http://www.neok12.com/Microorganisms.htm : For interactive videos on microorganisms.
15

Projects and assignments:


1. Research one benecial and one harmful application of one member from each kingdom, with examples
from their use in South Africa. Students can be grouped into smaller groups and each one is given one
kingdom to research. (Use www.arkive.org as a research tool for your favourite animal or plant or
16

http://bugscope.becnkman.uiuc.edu/ for nice pictures of insects). Results can be presented in the form
17

of a poster.
2. Go to your nearest supermarket or garden and nd one representative organism for each kingdom.
Present this information by drawing diagrams.

12 http://tolweb.org/tree/learn/concepts/whatisphylogeny.html
13 http://tolweb.org/tree/phylogeny.html
14 http://bugscope.beckman.uiuc.edu/
15 http://www.neok12.com/Microorganisms.htm
16 http://www.arkive.org/
17 http://bugscope.becnkman.uiuc.edu/

Available for free at Connexions <http://cnx.org/content/col11410/1.3>


231
4.1.3 4.1.3 Summary18
4.1.3.1 Summary

In Section 4.1 Biodiversity and Classication you learnt:


• Denition of biological classication and hierarchical grouping of living organisms based on similarities
and dierences.
• History of classication and scientic naming of organisms using the binomial nomenclature. All
organisms have only ONE scientic name but many common names!
• Division of organisms into prokaryotes (simple, Unicellular) and Eukaryotes (mostly multicellular) and
the major dierences between the two.
• Classication of living organisms into 5 major kingdoms: - Monera, Protista, Fungi, Plantae and
Animalia and their unique characteristics.

4.2 History of life on Earth


4.2.1 4.2.1 Life's History19
4.2.1.1 Introduction

In this section you will be introduced to some scientic theories about life's history. One of the popular
theories of life's history is the theory of Evolution. Another is the theory of Intelligent Design. In order to
be able to evaluate information critically, it is important to rst understand how people form knowledge,
and to be able to dierentiate between data and conclusions.
4.2.1.1.1 How do we know?

How do we know what happened in life's history? We cannot do experiments on the origin of life. There
are also no historical records about the origin of life. We have to rely on data we nd. From this we draw
conclusions about what might have happened in life's history.
http://www.youtube.com/watch?v=pXs-693ERSc
How do we know?

This media object is a Flash object. Please view or download it at


<http://www.youtube.com/v/pXs-693ERSc?version=3&hl=en_US>
Figure 4.4: This is a clip movie about how people get knowledge in general, and how people can get
knowledge about origins, in particular.

4.2.1.1.2 Data and Conclusions

Data means information people collect using their senses : sight, touch, hearing, feeling, smell. Usually
when a scientist collects data, other scientists will agree with him/her about this data. Sometimes other
scientists might question whether the data was correctly recorded, or whether the data is a forgery, but
usually scientists trust that the data was collected correctly.
18 This content is available online at <http://cnx.org/content/m43088/1.1/>.
19 This content is available online at <http://cnx.org/content/m43241/1.1/>.

Available for free at Connexions <http://cnx.org/content/col11410/1.3>


232 CHAPTER 4. DIVERSITY, CHANGE AND CONTINUITY

Conclusions are patterns people think up to help to make sense of data. When a scientist draws a
conclusion from some data, he/she makes various assumptions. Assumptions are thoughts which people take
to be true, without proof. Assumptions should be justied so that people can evaluate their validity
(how likely they are to be true). It is common for scientists to disagree on the validity of assumptions
and conclusions, even when they do agree on the data from which the conclusions are made. This is because
dierent conclusions can often be drawn from the same data.
http://www.youtube.com/watch?v=AVCPfrp-VDo
Data Conclusion

This media object is a Flash object. Please view or download it at


<http://www.youtube.com/v/AVCPfrp-VDo?version=3&hl=en_US>
Figure 4.5

4.2.1.2 Life's History

APBiology: Molecular Evolution and the early earth: http://outreach.mcb.harvard.edu/materials.htm 20

The extremely long period of time over which life has developed on earth can be represented in various
ways. Examine the diagrams below:
20 http://outreach.mcb.harvard.edu/materials.htm

Available for free at Connexions <http://cnx.org/content/col11410/1.3>


233

21
Figure 4.6: Taken from http://combattemple.com/clamping-102420-geologist-clyde-smith/

The various time periods are related to dramatic climate changes that the earth has experienced over
time. This is partly due to what is called continental drift :
4.2.1.2.1 Continental drift:

This theory proposed that all land was at one stage joined to form the supercontinent Pangaea ,
which split into Laurasia in the north and Gondwana (or Gondwanaland) in the south. See
http://www.exploratorium.edu/origins/antarctica/ideas/gondwana2.html 22

There is much evidence that continental drift occurred and is still continuing today:
There is biogeographic evidence of related species in widely isolated areas, such as the very similar
ightless birds like the rhea in South America, the ostrich in Africa, the moa in New Zealand, the emu and
21 http://combattemple.com/clamping-102420-geologist-clyde-smith/
22 http://www.exploratorium.edu/origins/antarctica/ideas/gondwana2.html

Available for free at Connexions <http://cnx.org/content/col11410/1.3>


234 CHAPTER 4. DIVERSITY, CHANGE AND CONTINUITY

cassowary in Australia. They are thought to have developed from a common ancestor on Pangaea. As the
climate gradually changed, organisms slowly adapted and underwent speciation in response to changes in
the temperature and vegetation around them.
OTHER EVIDENCE for continental drift include the following, showing that the climate in some areas
is now very dierent from what it once was:
• The discovery of fossilized tropical plants under Greenland's ice caps
• Glacial landscapes in central Africa and Central America
• Whale fossils in the Sahara desert
• The discovery of subtropical plant fossils in Antarctica, indicating that it once had a much warmer
climate and lush vegetation.
• South African examples of continental drift include the discovery of the fossils of marine organisms in
places that are VERY far from the sea, such as bivalves and ammonites in the Makhatini ats in
northern KZN, and marine trilobite fossils in the Karoo.

Figure 4.7: Ammonite fossil

Figure 4.8: Trilobite fossil

Available for free at Connexions <http://cnx.org/content/col11410/1.3>


235
4.2.1.2.2 Plate tectonics

This theory provides a mechanism for continental drift. The continents we know today rest on large,
interlocking plates of land called tectonic plates, which oat on a hot, molten layer that shifts them. Slow
movements of these plates move continents further apart, but more rapid movements of the plates are evident
when earthquakes occur. The continents are still moving apart at the rate of just a few cm per year.

Available for free at Connexions <http://cnx.org/content/col11410/1.3>


236 CHAPTER 4. DIVERSITY, CHANGE AND CONTINUITY

4.2.2 4.2.2 Geological timescale23


4.2.2.1 Geological Timescale

Figure 4.9

Available for free at Connexions <http://cnx.org/content/col11410/1.3>


23 This content is available online at <http://cnx.org/content/m43214/1.2/>.
237
Note that the time scale above shows the Carboniferous period as represented by the Pennsylvanian and
Mississippian separately. This is not always done.
Geological time: interactive site: http://learn.genetics.utah.edu/content/variation/time/ 24

Geological time: http://www.pbs.org/wgbh/evolution/change/deeptime/index.html 25

4.2.2.1.1 The three eras of the geological time scale

The earth's history can be traced over MANY millions of years, so scientists have developed a geological time
scale to help visualize these periods. This vast amount of time is divided into eons, eras and periods
for easier reference. You MUST know the names of the three ERAS:
So, for example, the most recent eon is divided into 3 eras, called the
Paleozoic (meaning ancient life)
Mesozoic (meaning middle life)
Caenozoic (meaning recent life)
You don't have to memorize the periods, only the eras . It is important that you become familiar with
these names, so that you can use such information in a test or exam. The end of each era is marked by a
series of catastrophic extinctions, which wiped out many of the previously successful species. Examine the
diagram below, showing eras & periods.

Figure 4.10

24 http://learn.genetics.utah.edu/content/variation/time/
25 http://www.pbs.org/wgbh/evolution/change/deeptime/index.html

Available for free at Connexions <http://cnx.org/content/col11410/1.3>


238 CHAPTER 4. DIVERSITY, CHANGE AND CONTINUITY

(Ma = million years)


Examine the pie chart and the table, which has the main events of each period
Cambrian period
Explosion of multi-cellular life, many trilobites in seas, modern groups develop
Ordovician and Silurian periods
Invertebrates with shells, rst sh, rst plants with vascular tissue
Devonian period
Arthropods on land, rst trees, many primitive sh, rst amphibians and insects
Carboniferous period
Coal formed, Gondwana is under ice sheets, 1st reptiles develop, many diverse insects
Permian period
Glossopteris trees in Gondwana, many marine Molluscs, mammal-like reptiles
MESOZOIC
Triassic period
First dinosaurs, rst small mammals develop
Jurassic period
Dinosaurs develop many forms, 1st birds develop, conifers form, ammonites in seas
Cretaceous period
Flowering plants and insects evolve, more dinosaurs develop, placental mammals
CAENOZOIC
Many dierent forms of mammals and birds develop, the earth cools down after widespread heating,
modern animals develop, hominids develop

Cambrian period
Explosion of multi-cellular life, many trilobites in seas, modern groups develop
Ordovician and Silurian periods
Invertebrates with shells, rst sh, rst plants with vascular tissue
Devonian period
Arthropods on land, rst trees, many primitive sh, rst amphibians and insects
Carboniferous period
Coal formed, Gondwana is under ice sheets, 1st reptiles develop, many diverse insects
Permian period
Glossopteris trees in Gondwana, many marine Molluscs, mammal-like reptiles
MESOZOIC
Triassic period
First dinosaurs, rst small mammals develop
Jurassic period

Available for free at Connexions <http://cnx.org/content/col11410/1.3>


239
Dinosaurs develop many forms, 1st birds develop, conifers form, ammonites in seas
Cretaceous period
Flowering plants and insects evolve, more dinosaurs develop, placental mammals
CAENOZOIC
Many dierent forms of mammals and birds develop, the earth cools down after widespread heating,
modern animals develop, hominids develop

Cambrian period
Explosion of multi-cellular life, many trilobites in seas, modern groups develop
Ordovician and Silurian periods
Invertebrates with shells, rst sh, rst plants with vascular tissue
Devonian period
Arthropods on land, rst trees, many primitive sh, rst amphibians and insects
Carboniferous period
Coal formed, Gondwana is under ice sheets, 1st reptiles develop, many diverse insects
Permian period
Glossopteris trees in Gondwana, many marine Molluscs, mammal-like reptiles
MESOZOIC
Triassic period
First dinosaurs, rst small mammals develop
Jurassic period
Dinosaurs develop many forms, 1st birds develop, conifers form, ammonites in seas
Cretaceous period
Flowering plants and insects evolve, more dinosaurs develop, placental mammals
CAENOZOIC
Many dierent forms of mammals and birds develop, the earth cools down after widespread heating,
modern animals develop, hominids develop

Available for free at Connexions <http://cnx.org/content/col11410/1.3>


240 CHAPTER 4. DIVERSITY, CHANGE AND CONTINUITY

PALEOZOIC Cambrian period Explosion of multi-cellular life, many trilobites in seas, modern groups develop
Ordovician and Silurian periods Invertebrates with shells, rst sh, rst plants with vascular tis

Devonian period Arthropods on land, rst trees, many primitive sh, rst amphibians and insects

Carboniferous period Coal formed, Gondwana is under ice sheets, 1st reptiles develop, many dive

Permian period Glossopteris trees in Gondwana, many marine Molluscs, mammal-like reptiles

MESOZOIC Triassic period First dinosaurs, rst small mammals develop

Jurassic period Dinosaurs develop many forms, 1st birds develop, conifers form, ammonites in sea

Cretaceous period Flowering plants and insects evolve, more dinosaurs develop, placental mamma

CAENOZOIC Many dierent forms of mammals and birds develop, the earth cools down after widespread heating, m
Table 4.2

4.2.2.1.2 The following are the major events in each era of the time scale:

• During the Pre-Cambrian time, life in general consisted of bacteria, simple algae and simple unicellular
organisms. The best examples of such early life are the stromatolites, large mounds of cyanobacteria
in sediment along the continental shelf that oxygenated the early atmosphere and allowed other aerobic
life forms to exist.

Figure 4.11: Stromatolites

• Paleozoic : Started with an explosion of multicellular life, called the Cambrian Explosion. Marine
trilobites were common. The rst invertebrates and sh are found, later the rst amphibians and
insects. Diverse land plants develop and coal swamps form in certain areas. The rst reptiles form.
This era ended with the massive Permian Extinction, wiping out many successful species
Available for free at Connexions <http://cnx.org/content/col11410/1.3>
241

Figure 4.12: Several trilobite fossils

• Mesozoic : The Mesozoic era starts with the Triassic period, which saw the rise of the dinosaurs
as the world's dominant organisms. The dominant plants were Gymnosperms (cone bearers, like
cycads and pine trees). Later, the rst mammals develop and owering plants or Angiosperms are
formed. Birds develop, as well as the rst placental mammals. This era ends with the Cretaceous
Extinction (the so-called KT-boundary), that wiped out the dinosaurs 65 million years ago.
Dinosaur

Available for free at Connexions <http://cnx.org/content/col11410/1.3>


242 CHAPTER 4. DIVERSITY, CHANGE AND CONTINUITY

Figure 4.13: cycad

• Coenozoic : Mammals developed further, including primates. Development of birds and owering
plantsis evident. Global cooling occurs (i.e. the most recent ice ages, which caused a drop in sea
levels. This allowed development of land bridges between North America and Asia, also between India
and Sri Lanka and between Australia and the islands to its north. These land bridges assisted greatly
in the migration of species to new land masses).
Hominid evolution started during the Coenozoic. This will be discussed more fully in Grade 12.

Available for free at Connexions <http://cnx.org/content/col11410/1.3>


243

Figure 4.14

Figure 4.15

Two types of owering plants (Angiospermae)

Available for free at Connexions <http://cnx.org/content/col11410/1.3>


244 CHAPTER 4. DIVERSITY, CHANGE AND CONTINUITY

Figure 4.16: Models of Australopithecus, an early hominid

Figure 4.17: Baby mammoth

Available for free at Connexions <http://cnx.org/content/col11410/1.3>


245
4.2.2.1.2.1 Critical evaluation

The geological column summarises a commonly accepted evolutionary theory about life's history. As with
most scientic theories, not all scientists agree about the theory. Criticisms include the following:
• The sequence (order) of fossils given in the geological column is not found anywhere on earth.
Instead, there are many places where `older' layers are found on top of `younger' layers.
• There are many gaps in every sequence. To explain this, palaeontologists suggest that about 2/3 of
all the kinds of organisms that ever lived could not have left any fossils. (Palaeontologists are scientists
who study fossils and try to use then to reconstruct the past.) It is reasonable to expect that many
soft-bodied creatures would not have been fossilised. However, it is strange that there are gaps in the
fossil records even for creatures with hard shells and strong bones.
• The dating methods used to date rocks might not give valid dates. This is discussed in the section
about fossils and dating.

4.2.2.1.2.2 THE MISSING LINK BETWEEN DINOSAURS AND BIRDS

It's been believed for many years that modern birds developed from reptilian ancestors, as dinosaur fossils
often showed AVIAN or `bird-like' features. There was thus much excitement when the rst fossil of a
missing link between birds and dinosaurs was found, the so-called Archaeopteryx. Around 10 such fossils
have been found to date. Read the information below:

Figure 4.18

Available for free at Connexions <http://cnx.org/content/col11410/1.3>


246 CHAPTER 4. DIVERSITY, CHANGE AND CONTINUITY

Figure 4.19

Left is a specimen of Archaeopteryx , and right is an artist's impression of the skeleton in an upright
position. Note the very ne feather impressions, including the ight feathers of the wings. Despite its obvious
avian nature, Archaeopteryx has a hand virtually identical to other dinosaurs such as Velociraptor .

Available for free at Connexions <http://cnx.org/content/col11410/1.3>


247

Figure 4.20: Artist's impression of the animal

Available for free at Connexions <http://cnx.org/content/col11410/1.3>


248 CHAPTER 4. DIVERSITY, CHANGE AND CONTINUITY

Figure 4.21: A mounted archeopteryx model. Note the teeth.

Exercise 4.2.2.1 (Solution on p. 281.)


Use the pictures below and on the previous page to compare the skeletons of a
dinosaur, Archaeopteryx and a modern bird. In your answer, give dierences and
similarities between Archaeopteryx and dinosaurs, and between Archaeopteryx and birds.

Available for free at Connexions <http://cnx.org/content/col11410/1.3>


249

Available for free at Connexions <http://cnx.org/content/col11410/1.3>


250 CHAPTER 4. DIVERSITY, CHANGE AND CONTINUITY

4.2.2.1.2.2.1 Activity: Critical Analysis of Archaeopteryx


Some scientists consider Archaeopteryx to be a transitional fossil between dinosaurs and birds. Other
scientists disagree. Below are two arguments given by scientists to support their interpretation that Ar-
chaeopteryx is not a transitional fossil between dinosaurs and birds. For each of these arguments, state
whether the scientists are questioning the DATA, or the CONCLUSIONS about the Archaeopteryx fossil.
Exercise 4.2.2.2 (Solution on p. 284.)

a. Watkins, Hoyle, Wickramasinghe, Watkins, Rabilizirov and Spetner (1985) published an arti-
cle where they claimed that the feather impressions of Archaeopteryx were forged. They pro-
vide evidence suggesting that chicken feathers were pressed into a cement mix, and these were
applied over dinosaur fossils: Watkins, R.S., Fred Hoyle, N.C. Wickramasinghe, J. Watkins,
R. Rabilizirov, and L.M. Spetner (1985),  Archaeopteryx : A Photographic Study, British
Journal of Photography, 132:264-266, March 8. These scientists are questioning the DATA /
CONCLUSIONS made by other scientists about Archaeopteryx.
b. Some scientists argue that even if the feathers on Archaeopteryx were not forged, there is no
evidence that the fossil is transitional between reptiles and birds, since the fossil is compatible
with the fossil of a bird. These scientists are questioning the DATA / CONCLUSIONS made
by other scientists about Archaeopteryx.
c. Some scientists argue that birds have been found in younger rocks than the rocks in which
Archaeopteryx was found, and so Archaeopteryx cannot be the ancestor to birds. These
scientists are questioning the DATA / CONCLUSIONS made by other scientists about Ar-
chaeopteryx.

4.2.2.1.2.3 The missing link between sh and amphibians:

In 1938, an East London sherman caught a deep-sea sh he had never seen, so it was taken to the East
London museum, where it was identied as a coelacanth by Professor JLB Smith and his assistant, Marjorie
Courtenay-Latimer. This caused an international uproar, because the sh was previously known only from
fossils and had been believed to be extinct. People were amazed that fossil coelacanth found in rock strata
70 350 million years old could look exactly like the large blue sh before their eyes! This started a search
for more specimens and 14 years later, another one was caught o the Comoro Islands in the Indian Ocean,
north of Madagascar and more have been found since, including in deep crevices at St. Lucia in northern
KZN.

Available for free at Connexions <http://cnx.org/content/col11410/1.3>


251

Figure 4.22: Coelacanth display

Figure 4.23: Madge Courtenay-Latimer and the coelacanth

Available for free at Connexions <http://cnx.org/content/col11410/1.3>


252 CHAPTER 4. DIVERSITY, CHANGE AND CONTINUITY

Scientists agree that the coelacanth is probably the oldest living vertebrate and they are thought to
be closely related to the freshwater sh that is seen as the ancestor of all land animals. The coelacanth's
internal organs are in many ways similar to those of amphibians, which are the earliest life forms to have
colonized land. Unlike amphibians, its spine is supported by a permanent notochord, a cartilage and oil
tube  it never develops a bony vertebral column like other sh do. The body is lobe-nned, not ray-nned
like modern sh, although it is not known to walk on the ocean oor. Many scientists had thought that
the coelacanth's ns looked like legs (it was referred to as `old four legs') and that it was in the process of
developing limbs for use on land, but here was a specimen, millions of years later, that still had the same
n structure. Its nostrils are also not blind-ending as in sh; they open into the mouth and can be used for
breathing.
Assignment:
The modern coelacanth-nding throws doubt on an assumption palaeontologists use when reasoning about
what the fossil record says about extinctions.
a. What is the assumption the modern coelacanth-nding throws doubt on?
b. How does the modern coelacanth-nding throw doubt on this assumption ?
An extinction event can be recognised by fossils of that type not being found in higher (i.e. younger) rock.
Although no coelacanth fossils are found in rock younger than that assumed to be 100 million years old,
coelacanths are not extinct: they are alive today (extant).
4.2.3 4.2.3 Cambrian Explosion26
4.2.3.1 What does this mean?

Cambrian explosion refers to the tremendous increase in the number and type of multicellular organisms at
the start of the Cambrian period of the Paleozoic era. Ancestral forms of all organisms emerged then, which
have gradually changed to become the life forms we see today.
Cambrian explosion: explanation http://www.fossilmuseum.net/Paleobiology/CambrianExplosion.htm
27

4.2.3.2 Signicant changes have taken place in all groups of plants and animals.

4.2.3.2.1 Plants:

From being purely aquatic, plants gradually colonized land and had to cope with life in a completely dierent
medium. They had to develop several structural changes, such as:
• They developed true roots, stems and leaves with dierent tissues in these organs. Some plants devel-
oped owers and fruit.
• They developed supporting tissue like xylem and thick secondary walls with lignin.
• They developed a means of protecting themselves against dehydration, such as the development of a
cuticle and sunken stomata.
• They became less dependent on water for fertilization, using wind and or animals to transfer pollen
grains with sperm nuclei.

4.2.3.2.2 Animals:

Animals also became more complex than their primitive ancestors. Some colonized land and had to make
further changes.
• They developed a head with a brain and limbs on both sides of the body.
26 This content is available online at <http://cnx.org/content/m43205/1.1/>.
27 http://www.fossilmuseum.net/Paleobiology/CambrianExplosion.htm

Available for free at Connexions <http://cnx.org/content/col11410/1.3>


253
• The digestive system became more complex, developing a gut with two openings and various glands
associated with digestion.
• They had to protect themselves against dehydration, so many developed internal breathing systems,
protecting the delicate lungs.
• Birds and mammals became endothermic, i.e. able to keep their body temperatures constant despite
changes in environmental temperature.
See: http:www.khanacademy.org/video/introducti-to-evolution-and-natural- election?playlist=Biology
4.2.4 4.2.4 Mass extinctions28
4.2.4.1 What does this mean?

When all the members of a species die out the species has become extinct. A species is deduced to be extinct
if it stops appearing in apparently later (more recent) rock. There seem to have been times in the earth's
history when many species have become extinct at about the same time. Such an event is called a mass
extinction.
4.2.4.2 Five major mass extinctions

Evolutionists think that there were ve large mass extinctions, with the two largest being:
• The rst major mass extinction is said to have occurred at the end of the Permian period. This is
thought to have been about 225 million years ago. At that time 90% of marine life seems to have
become extinct.
• The second is thought to have occurred about 65 million years ago, at the end of the Cretaceous Period.
This led to the disappearance of the dinosaurs.
Each time a mass extinction seems to have happened, many new kinds of fossils appear in later (more recent)
rocks. These are considered new because their fossils were absent from older-dated rocks. Transitional
evolutionary ancestors of these new organisms are also not found in older rock. This sudden appearance of
new species is explained as follows. After a mass extinction a large part of the environment is cleared of the
life forms which had previously dominated (controlled) the earth. This makes way for new forms to evolve
and ll the many new vacant ecological niches (open spaces in the ecosystem).
4.2.4.2.1 Possible causes of mass extinctions

Mass extinctions can be caused by various factors. These may be extra-terrestrial, due to climate change,
caused by humans, genetic, or due to continental drift:
• Extra-terrestrial events are caused by something outside of the earth, e.g. by a meteorite. At least two
mass extinctions are thought to have been caused by huge meteorites striking the earth. A meteorite is
s chunk of rock from outer space, which enters the earth's atmosphere. These meteorites are thought
to have been about 10 km in diameter. Some scientists believe that such impacts happened many
millions of years ago. Other data suggests that a meteor of diameter 200km hit the earth only a few
thousand years ago. This does not t in with the geological time scale and is usually ignored. (This
is based on observations of the changing tilt of the earth's axis, made over a period covering most of
recorded history.
• Climate change can cause extinction.
• Link to simulation - glaciers http://phet.colorado.edu/en/simulation/glaciers 29

28 This content is available online at <http://cnx.org/content/m43235/1.1/>.


29 http://phet.colorado.edu/en/simulation/glaciers

Available for free at Connexions <http://cnx.org/content/col11410/1.3>


254 CHAPTER 4. DIVERSITY, CHANGE AND CONTINUITY

• Human activity can cause extinction. Humans are destroying the habitat of many types of plants
and animals. This is done by chopping down forests and draining swamps to grow crops and to build
houses.
• Continental drift might have caused extinction. According to the theory of continental drift, all land
used to be joined in a supercontinent called Pangaea. This later split into Laurasia in the north and
Gondwanaland in the south. There is much evidence for this. As the continents moved apart, some
populations were split and moved apart. This could have caused extinction as habitats and climates
changed as a result of the continental drift.
See http://www.exploratorium.edu/origins/antarctica/ideas/gondwana2.html 30

What killed the dinosaurs: http://www.pbs.org/wgbh/evolution/extinction/dinosaurs/ 31

Biodiversity: the 6th great wave: http://news.bbc.co.uk/2/hi/science/nature/3667300.stm 32

Vertebrate extinction crisis (videos): http://www.arkive.org/newsletter/?u=722a3ad7755a24108f2cd402e&id=db50e9fd5d


IUCN_Press_Release10_27_2010&utm_medium=email 33

4.2.4.2.1.1 Activity: Mass extinctions

1. How do extinction events seem to help the process of evolution?


2. Name any four possible causes of mass extinctions.
3. Complete this table:

Geological time period when ex- Years before present when evolu- Main feature of extinction event
tinction is thought to have oc- tionists think this happened
curred.
End of the Permian period.
Dinosaurs became extinct.
Table 4.3

Activity: Answers
1. How do extinction events seem to help the process of evolution? They possibly cleared the earth of
previously dominant species, allowing new species to evolve into the ecological niches this opened up.
2. Name any four possible causes of mass extinctions. Meteorites, volcanoes, climactic changes (e.g. ice age
/ global warming), man-caused (e.g. over-hunting), due to genetic deterioration
3. Complete this table:

Geological time period when ex- Years before present when evolu- Main feature of extinction event
tinction is thought to have oc- tionists think this happened
curred.
End of the Permian period. 225 million years ago 90% of marine life became extinct
End of Cretaceous era 65 million years ago Dinosaurs became extinct.
Table 4.4

30 http://www.exploratorium.edu/origins/antarctica/ideas/gondwana2.html
31 http://www.pbs.org/wgbh/evolution/extinction/dinosaurs/
32 http://news.bbc.co.uk/2/hi/science/nature/3667300.stm
33 http://www.arkive.org/newsletter/?u=722a3ad7755a24108f2cd402e&id=db50e9fd5d&e=&utm_source=ARKive&utm_campaign=db50e9fd5d
IUCN_Press_Release10_27_2010&utm_medium=email

Available for free at Connexions <http://cnx.org/content/col11410/1.3>


255
4.2.5 4.2.5 Fossils34
4.2.5.1 Introduction

Fossils are traces of organisms which used to be alive. Fossils can tell us something about the creatures they
came from. Hard tissue, such as bone or shell, is fossilized better than softer body parts. This is to be
expected, since soft tissue is eaten or it decays soon after death, leaving the bones and shells to petrify (turn
to stone).However, examples of perfectly fossilized soft creatures, like jellysh, do exist. In this section, you
learn about theories of how fossils form, types of fossils, how fossils are dated, and fossil tourism. People
who study fossils are called paleontologists.
Neok12: http://www.neok12.com/Fossils.html 35

The way fossils form: Interactive site: http://www.ashyourbrain.com/pieces/fossils/index.php 36

4.2.5.2 Fossil formation

Dierent kinds of fossils are formed in dierent ways. Many form by petrifaction, i.e. turning into stone.
The following may petrify:

• Parts of the body


• Sediments surrounding the body. Sediments are layers of material (e.g. sand) deposited (dropped) by
wind / water / ice.
• Cavities (spaces) left by the body.
Scientists have observed petrifaction occurring both in nature and in laboratory experiments. Min-
erals(inorganic compounds) enter the body, hardening it and turning it to stone. Fossilization has been
observed to happen very quickly when high concentrations of suitable minerals were present, and when the
conditions were favourable for certain bacteria which aid the petrifaction process.
4.2.5.3 Fossil types

There are several kinds of fossils. These include footprints, dung, moulds, casts, permineralised, and trace
fossils.
Footprints, made when creatures walked in soft sand or mud, can be preserved (kept safe) when the sand
or mud dries, hardens and then petries. From the footprints we can deduce the shape and size of the feet
and how the creature which made them walked. A fossilized dinosaur footprint is shown in Figure 1.
34 This content is available online at <http://cnx.org/content/m43253/1.1/>.
35 http://www.neok12.com/Fossils.html
36 http://www.ashyourbrain.com/pieces/fossils/index.php

Available for free at Connexions <http://cnx.org/content/col11410/1.3>


256 CHAPTER 4. DIVERSITY, CHANGE AND CONTINUITY

Figure 4.24: A fossilized dinosaur footprint

Dung can also become petried. Petried dung is called a coprolite. From this we can deduce what kind
of food a creature ate.
Mould fossils form around a cavity which the organism used to ll. A dead creature can be buried in
sediments. These may petrify, forming a mould around the body. The soft body of the creature decays
(rots), leaving the mould empty. The shape of the body can be seen from the mould.
Cast fossils look the same as the original body, except that they are made of minerals. Minerals (e.g.
calcite) ll the cavity inside a mould, making a cast fossil. A cast fossil of a shelled sea animal is shown in
Figure 2.

Figure 4.25: A cast fossil of a shelled sea animal

Permineralized fossils form when the body of an organism becomes petried. Water moves through the
tissues of the dead creature and leaves minerals behind. These minerals replace the living tissue. These
fossils can give a very good idea of the original structure of a dead creature. A permineralised fossil of a tree
stump is shown in Figure3.

Available for free at Connexions <http://cnx.org/content/col11410/1.3>


257

Figure 4.26: Petried wood. This tree stump turned into stone as minerals replaced the living tissue,
and hardened

Film fossils form where leaves have left black or brown marks, made of carbon, on rock surfaces. A lm
fossil of a leaf is shown in Figure 4.

Figure 4.27: A lm fossil of a leaf

Available for free at Connexions <http://cnx.org/content/col11410/1.3>


258 CHAPTER 4. DIVERSITY, CHANGE AND CONTINUITY

4.2.5.4 Fossil dating

4.2.5.4.1 Radiometric dating

Radiometric dating is sometimes used when scientists want to date fossils. Some elements are unstable and
break down into lighter elements over time. Such substances are said to be radioactive. The original element
is called the parent and the element it changes into is called the daughter. For example, uranium-238 decays
into lead-206. What is the data which scientists use in this method, and the assumptions they might make
when drawing conclusions about fossil dates?:
Data: A rock has a certain ratio of the parent to daughter elements, e.g. a certain ratio of uranium
to lead.
Conclusions: In radiometric dating, the ratio of parent to daughter elements is compared. From this the
age of the rock is concluded.
Assumptions: A scientist has to make many assumptions in order to conclude the age of a rock from
ratios between elements. These assumptions include:
The rock must have had only parent, and no daughter, elements when the rock formed.
The rate at which the parent element changed into the daughter element is known from the rate this occurs
today.
For example, if a rock is found containing both uranium-238 and lead-206, it is assumed that all this lead
came from the uranium by radioactive decay and that the rate at which that happened is the same as the
rate observed today.
4.2.5.4.2 Carbon dating

Carbon dating is the only type of radiometric dating which can be done directly on fossils. However, carbon
dating gives young ages, and so is generally not accepted, except for dating organisms which have been dead
for less than about 30 000 years.
How does carbon dating work? All organisms incorporate carbon atoms into their bodies when they are
alive. This stops at death. There are two isotopes (forms) of carbon. The most common isotope is C12.
The other isotope, C14, is radioactive. The proportion C12:C14 in the atmosphere is known. Both carbon
isotopes combine with oxygen to form CO2 and both are incorporated into plants during photosynthesis.
Therefore both carbon isotopes form part of animals' food. The organism dies. The C12 in the dead body
does not break down radioactively, but the C14 in the body does break down radioactively. We can measure
the rate at which C14 breaks down radioactively over time today. Scientists compare the ratio of C12:C14in
the body to the ratio of C12:C14in the atmosphere. This is the data used. The scientists may assume that
the C12:C14 ratio at the time of death was the same as the atmosphere's C12:C14ratio, and that the rate
at which C14broke down in the past was the same as it is today. From this they may conclude the age of
the rock.
4.2.5.4.3 Dating fossils by dating rocks

For fossils which are believed to be older than 30 000 years, carbon dating cannot be used, so other radiometric
dating methods are used. However, these methods can only be done on igneous rocks (rocks formed from
volcanoes).Igneous rock cannot contain fossils. Fossils are only found in sedimentary rock. In order to
conclude the date of a fossil the following process is followed:
The scientist nds some igneous rock which he assumes to be the same age as the sedimentary rock in
which the fossil was found.
The scientist measures the ratio of parent to daughter elements in this igneous rock.
The scientist concludes the age of the igneous rock, as previously explained.
The scientist concludes the age of the fossil from the age of the igneous rock.
Link to movie: Fossil dating 6'14

Available for free at Connexions <http://cnx.org/content/col11410/1.3>


259
<iframe width="425" height="349" src="http://www.youtube.com/embed/K5GWSbBkjuk?hl=en&fs=1"
frameborder="0" allowfullscreen></iframe>
Caption: This is a 6'14 video in which radiometric dating is explained.
4.2.5.5 Fossil tourism

South Africa has a number of fossil-rich areas which attract tourists. The Cradle of Mankind (Maropeng,
Sterkfontein, Kromdraai and surrounding areas) and Langebaan are examples of places in South Africa which
attract fossil tourism.
Maropeng is a Tswana word for the origin of mankind.
4.2.5.5.1 Living fossils

Many creatures which disappeared from the fossil record millions of geological years ago have been found to
be alive still and almost identical to their fossilized counterparts.
There are many examples of living fossils. The Tuatara, which looks like a lizard with a beak-shaped
head, disappears from the record 135 million geological years ago, but is still alive. Lingula, a worm-like
creature which disappears from the record about 400 million years ago, not very far from the beginning of
metazoan evolution, is still alive and shows no sign of signicant evolution from its fossilized ancestors. This
suggests that we may be interpreting the fossil record incorrectly, and our conclusions based on it may be
wrong.
4.2.5.5.1.1 Activity: Fossils

1.Name / give the term for:

• Turning to stone
• A fossil in which the surroundings became stone, leaving a cavity where the body had been
• The time period and era, according to the geological column, when dinosaurs rst appeared
• The time period and era, according to the geological column, when dinosaurs became extinct
• A person who studies fossils
• A person who studies rocks
• Layers of deposited material
• No members of that organism exist anymore
• The time era, according to the geological column, when mankind evolved
2.For each of the following sentences, say whether the information is: (Circle i / ii / iii)
• Accepted by denition if people have dened this to be so, or
• Known by observation if people have observed this to be so, or
• Deduced if people have come to this knowledge by reasoning.

• During petrifaction, minerals enter, harden, and turn a body to stone.[ i / ii / iii]
• The Triassic Period is more recent than the Permian Period. [ i / ii / iii]
• The oldest fossils are 650 million years old. [ i / ii / iii]
• A lm fossil is a carbon mark left on rock e.g. by leaves. [ i / ii / iii]
• The deepest rocks with reptile fossils are classed as Carboniforous. [ i / ii / iii]
• Reptiles rst evolved 350 million years ago, at the start of the Carboniforous period. [ i / ii / iii]
• Only 2/3 of all types of organisms which have ever lived have left any fossils.[ i / ii / iii]
3.The modern coelacanth-nding throws doubt on an assumption palaeontologists use when reasoning
about what the fossil record says about extinctions.

Available for free at Connexions <http://cnx.org/content/col11410/1.3>


260 CHAPTER 4. DIVERSITY, CHANGE AND CONTINUITY

• What is the assumption the modern coelacanth-nding throws doubt on?


• How does the modern coelacanth-nding throw doubt on this assumption?
4.Look at these premises:
• A cadborosaurus was found in the stomach of a whale in 1993 .
• A pterodactyl was killed at Culment, France, in 1856 .
• Blood vessels, red blood cells, osteocytes and proteins have been found in unfossilized dinosaur bones .
• Undamaged DNA pieces have been found in insects fossilised in amber (a sticky plant resin).
• Unfossilised bones, blood vessels, blood cells and DNA break down quickly .
• The extinction of the dinosaurs is assumed at more than 60 million years ago.

Give a conclusion which follows from these premises


5.A rock is found to have a K:Ar ratio of 1:10. Its age is calculated as 4,37 x109 years.
a. K (potassium) is the parent element. Ar (argon) is the _________________ element.
b. What data is used to do this calculation?
c. What conclusion/inference is made from this data?
d. Name two assumptions used when making this conclusion from this data:
e. Give a counterargument, showing how this same data could t with a young age.
Activity: Fossils: Suggested Answers
1.Name / give the term for:
a. Turning to stone Petrifaction
b. A fossil in which the surroundings became stone, leaving a cavity where the body had been Mould
c. The time period and era, according to the geological column, when dinosaurs rst appeared Triassic
period; Mesozoic era
d. The time period and era, according to the geological column, when dinosaurs became extinct Cretaceous
period; Mesozoic era
e. A person who studies fossils Palaeontologist
f. A person who studies rocks Geologist
g. Layers of deposited material Sediments / Strata
h. No members of that organism exist anymore Extinct
i. The time era, according to the geological column, when mankind evolved Cenozoic
2.For each of the following sentences, say whether the information is: (Circle i / ii / iii)
i. Accepted by denition if people have dened this to be so, or
ii. Known by observation if people have observed this to be so, or
iii. Deduced if people have come to this knowledge by reasoning.
a. During petrifaction, minerals enter, harden, and turn a body to stone.[ i / ii / iii]Observed and Dened
b. The Triassic Period is more recent than the Permian Period. [ i / ii / iii]
c. Dened
d. The oldest fossils are 650 million years old. [ i / ii / iii]Deduced
e. A lm fossil is a carbon mark left on rock e.g. by leaves. [ i / ii / iii]Dened
f. The deepest rocks with reptile fossils are classed as Carboniforous. [ i / ii / iii]Dened
g. Reptiles rst evolved 350 million years ago, at the start of the Carboniforous period. [ i / ii / iii]Deduced
h. Only 2/3 of all types of organisms which have ever lived have left any fossils.
i / ii / iii Deduced
3.The modern coelacanth-nding throws doubt on an assumption palaeontologists use when reasoning about
what the fossil record says about extinctions.
Available for free at Connexions <http://cnx.org/content/col11410/1.3>
261
a. What is the assumption the modern coelacanth-nding throws doubt on? An extinction event can be
recognised by fossils of that type not being found in higher (i.e. younger) rock.
b. How does the modern coelacanth-nding throw doubt on this assumption? Although no coelacanth
fossils are found in rock younger than that assumed to be 100 million years old, coelacanths are not
extinct: they are alive today (extant).
4.Look at these premises:
• A cadborosaurus was found in the stomach of a whale in 1993.
• A pterodactyl was killed at Culment, France, in 1856.
• Blood vessels, red blood cells, osteocytes and proteins have been found in unfossilized dinosaur bones.
• Undamaged DNA pieces have been found in insects fossilised in amber (a sticky plant resin).
• Unfossilised bones, blood vessels, blood cells and DNA are known to break down quickly.
• The extinction of the dinosaurs is assumed at more than 60 million years ago.

Give a conclusion which follows from these premises: Either the assumption that dinosaurs became extinct
60 million years ago is faulty or cadborosauruses and pterodactyls are not dinosaurs.
5.A rock is found to have a K:Ar ratio of 1:10. Its age is calculated as 4,37 x109 years.
a) K (potassium) is the parent element. Ar (argon) is the daughter element.
b) What data is used to do this calculation? The K:Ar ratio .
c) What conclusion/inference is made from this data? The rock is 4,37 x109 years old .
d) Name two assumptions used when making this conclusion from this data:
All the Ar in the rock came from radioactive decay of the K. This radioactive decay happened at the same
rate as it had in the past.
e) Give a counterargument, showing how this same data could t with a young age.
The rock might have started with a lot of Ar in it. The conditions in the past might have caused K to decay
into Ar at a higher rate than today. Further, Argon is known to move easily through rocks, so the Ar might
not all have come from radioactivity.
4.2.6 4.2.6 Key Events in Life's History37
4.2.6.1 Key Events in Life's History

South Africa is home to many dierent kinds of fossils, including living fossils. Many of these fossils are key
events in life's history.
37 This content is available online at <http://cnx.org/content/m43229/1.1/>.

Available for free at Connexions <http://cnx.org/content/col11410/1.3>


262 CHAPTER 4. DIVERSITY, CHANGE AND CONTINUITY

4.2.6.1.1 Fossilised Bacteria In Baberton Area

Figure 4.28: the microbes broke down volcanic glass to extract nutrients

Figure 4.29: the rocks were altered after they were formed but the burrows were preserved

Available for free at Connexions <http://cnx.org/content/col11410/1.3>


263

Figure 4.30: Stromatolite formation near Barberton area

The earliest signs of life are about 3500 million years old. Some of the most ancient fossils that are known
to exist have been found in the rocks from the Barberton area of Mpumalanga. They are more than 3000
million years old. These tiny fossils look like modern blue-green bacteria. Later other types of blue- green
algae grew that formed a mat in shallow water. These are called stromatolites and can be found in the
ancient rocks near Barberton.
Evolution of the cell:
http://learn.genetics.utah.edu/content/begin/cells/organelles/ 38

4.2.6.1.1.1 Interesting Information

Conditions on Earth 4 billion years ago were very dierent than they are today. The atmosphere lacked
oxygen, and an ozone layer did not yet protect Earth from harmful radiation. Heavy rains, lightning and
volcanic activity were common. Yet the earliest cells originated in this extreme environment. Today, a group
of single-celled organisms called archaeabacteria, or archaea, still thrive in extreme habitats.
Astrobiologists are now using archaea to study the origins of life on Earth and other planets. Because
archaea inhabit places previously considered incompatible with life, they may provide clues that will improve
our ability to detect extraterrestrial life. Interestingly, current research suggests archaea may be capable of
space travel by meteorite. Such an event could have seeded life on Earth or elsewhere.

Figure 4.31: Lake that supports Archaea bacteria

38 http://learn.genetics.utah.edu/content/begin/cells/organelles/

Available for free at Connexions <http://cnx.org/content/col11410/1.3>


264 CHAPTER 4. DIVERSITY, CHANGE AND CONTINUITY

4.2.6.1.2 Soft Bodied Animals In Namibia And The Northern Cape

Figure 4.32

Figure 4.33

These soft-bodied animal-like creatures were unrelated to anything we see today and died out about 450
million years ago. Fossils of these ancient forms of life have been found in Namibia and the Northern Cape,
and also in Europe and North America. For animals without skeletons, like worms or jellysh, fossilization
is a very rare event. Paleontologists seldom nd a well- preserved fossil of a soft-bodied animal. For a
soft-bodied animal to be fossilized, its body must be protected from decomposition. The body is usually
exposed to air and water with a lot of oxygen, so it decomposes quickly. The animal is likely to be fossilized
only if it is buried soon after it dies (or when it is buried alive!).
Available for free at Connexions <http://cnx.org/content/col11410/1.3>
265
Soft bodied fossils in Namibia: (notes) http://fairley.ca/sclips_les/Eblossom.htm 39

Soft bodied animals and fossil formation: http://www.k5geosource.org/1content/1sc/fossils/pg6.html 40

4.2.6.1.3 Early Land Plants In Grahamstown Area

Figure 4.34

Figure 4.35

39 http://fairley.ca/sclips_les/Eblossom.htm
40 http://www.k5geosource.org/1content/1sc/fossils/pg6.html

Available for free at Connexions <http://cnx.org/content/col11410/1.3>


266 CHAPTER 4. DIVERSITY, CHANGE AND CONTINUITY

Figure 4.36

Figure 4.37

By the end of the Devonian period of the Palaeozoic era there were more club mosses, ferns and horsetails,
which are all plants that had primitive conducting tissue. Some of their relatives are alive today. There are
well preserved fossils of these club mosses and the simple relatives of conifers like cycads in the late Devonian
rocks near Grahamstown in the Eastern Cape.
Cycads  early land plants near Grahamstown:
http://www.arkive.org/eastern-cape-blue-cycad/encephalartos-horridus/ 41

41 http://www.arkive.org/eastern-cape-blue-cycad/encephalartos-horridus/

Available for free at Connexions <http://cnx.org/content/col11410/1.3>


267
4.2.6.1.4 Forests Of Primitive Plants Such As Glossopteris Near Mooi River And Estcourt

Figure 4.38: Distribution of Glossopteris in Pangaea

Available for free at Connexions <http://cnx.org/content/col11410/1.3>


268 CHAPTER 4. DIVERSITY, CHANGE AND CONTINUITY

Figure 4.39: Glossopteris

Near the end of the Palaeozoic Era, in the Permian period, Southern Africa had moved away from the South
Pole. When the climate became warmer, the land became covered in new types of cone-bearing plants.
The best known plant in the Southern hemisphere from that time is a tree called Glossopteris. Fossils of
Glossopteris tree trunks are common near Estcourt and Mooi River in Kwa-Zulu Natal. Plants like these
formed the huge coal deposits that are mined in South Africa today. The rst true seed-bearing plants,
which were simple conifers, appeared at the very end of the Permian Period.
Glossopteris and coal deposits:
http://www.scienceinafrica.co.za/2006/february/coal.htm 42

Map of coal deposits in SA:


http://www.mapsofworld.com/business/industries/coal-energy/south-africa-coal- deposits.html 43

42 http://www.scienceinafrica.co.za/2006/february/coal.htm
43 http://www.mapsofworld.com/business/industries/coal-energy/south-africa-coal-deposits.html

Available for free at Connexions <http://cnx.org/content/col11410/1.3>


269
4.2.6.1.5 The Coelacanth As A Living Fossil Of The Group That Is Ancestral To Amphibians

Figure 4.40: The coelacanth is a living fossil; it is similar to 350 million year old fossils but is still
alive today.http://www.ickr.com/photos/26573031@N05/2903081204/

The coelacanth is a lobe-nned sh from the Permian Period that was once thought to be extinct and was
only known as a fossil. In 1938, a South African shing boat netted a large sh with deep blue scales and
ns on short legs. Professor J.L.B. Smith recognised it as a fossil sh from rocks up to 350 million years
old. It was thought to have been extinct for at least 65 million years. A second coelacanth was found in
1952 in the Comores Islands. Many have been found since then. Another population has also been found
near Sodwana Bay on the Kwazulu-Natal north coast.
See article at
http://animals.nationalgeographic.com/animals/sh/coelacanth.html 44

4.2.6.1.6 Mammal-Like Reptiles In The Karoo

Figure 4.41: Therapsid fossil in the Karoo (ickr)

44 http://animals.nationalgeographic.com/animals/sh/coelacanth.html

Available for free at Connexions <http://cnx.org/content/col11410/1.3>


270 CHAPTER 4. DIVERSITY, CHANGE AND CONTINUITY

Figure 4.42: Lystrosaurus (Wikipedia)

Reptiles diversied and spread as the most successful and largest land vertebrates in the Permian Period.
One important group was reptiles with some mammal features that are called Therapsids. Thrinaxodon and
Lystrosaurus were small therapsids that grew to about 50cm long. Fossils of these mammal-like reptiles have
been found in the Karoo. These animals are seen as transitional between reptiles and mammals. They have
several developmental advances such as extra ear bones and a bony palate, which are not normally found in
reptiles.
4.2.6.1.7 Dinosaurs

Figure 4.43: Euskelosaurus browni

The special type of reptile called a dinosaur appeared and spread during the Triassic Period of the Mesozoic
Era. A South African example of a dinosaur is Euskelosaurus browni. It was one of the rst dinosaurs
discovered in Africa. It lived in the area now called the Eastern Cape and fossils have been found near
Ladybrand.
One of the best places in the whole world to look for dinosaur fossils is the sedimentary rocks of the
Drakensberg Mountains and Maluti Mountains of southern Africa.
Fossils of ferns and cone-bearing plants have also been found in the same areas as the dinosaurs.
Available for free at Connexions <http://cnx.org/content/col11410/1.3>
271
4.2.6.1.8 First Mammals

Figure 4.44: Megazostrodon  rst mammal during the Jurassic period. Mammals
could not dominate during the reign of the dinosaurs, as the dinosaurs were huge.(Flickr)
45
http://www.ickr.com/photos/nordelch/2052121799/

The rst true mammals appeared during the Jurassic Period of the Mesozoic Era. They were small, about
the size of a mouse and lived side by side with the dinosaurs. South Africa is the only place in the world
where there are fossils that show changes from the earliest Therapsid (mammal-like) reptiles from the Karoo
rocks of the Permian Period to the rst true mammals from the Drakensberg rocks in the Eastern Cape and
Lesotho.
Early mammal evolution: Interactive site
http://www.mnh.si.edu/mammals/pages/how/index.htm 46

4.2.6.1.9 Prehumans

Discovery of early Hominids in SA:


http://anthro.palomar.edu/hominid/australo_1.htm 47

The Hominids are the human-like organisms that diversied from the apes. The apes lived in tropical
forests, while the hominids lived in woodlands and savannah. About 4mya, several dierent species of
Hominids with large brains appeared in Africa. These prehumans are in the genus Australopithecus. The
modern human Homo sapiens appeared in Africa about 200 000 years ago and spread all around the Earth.
Many people believe that the eastern side of Africa and South Africa are the original home of humans.
The Cradle of Humankind at Sterkfontein and Kromdraai has yielded some of the most exciting and oldest
hominid fossils in the world.
45 http://www.ickr.com/photos/nordelch/2052121799/
46 http://www.mnh.si.edu/mammals/pages/how/index.htm
47 http://anthro.palomar.edu/hominid/australo_1.htm

Available for free at Connexions <http://cnx.org/content/col11410/1.3>


272 CHAPTER 4. DIVERSITY, CHANGE AND CONTINUITY

Figure 4.45

Figure 4.46: Ron Clarke with Little Foot and Mrs Ples. Both were discovered at Sterkfontein.

4.2.6.1.10 Activity: Map Key Events In Life's History In Southern Africa

The text on the following pages describes some of the key events in Life's History for which there is evidence
in Southern Africa.

Available for free at Connexions <http://cnx.org/content/col11410/1.3>


273
• When you read each description in the text that follows, identify the places where evidence has been
found and mark them on the map.
• For each place you have marked, [provide a brief description of the event that happened]

Figure 4.47

Available for free at Connexions <http://cnx.org/content/col11410/1.3>


274 CHAPTER 4. DIVERSITY, CHANGE AND CONTINUITY

4.2.7 4.2.7 Summary48


4.2.7.1 Summary

a. Life's History:

a. The extremely long time periods in the earth's history are divided into eons, eras and periods for
easier study.
b. Continental drift occurred by means of plate tectonic movements. The supercontinent Pangaea
split into Laurasia in the north and Gondwana in the south, ultimately forming today's continents.
b. The Geological Time Scale:

• Before the Paleozoic era, life consisted of bacterial mounds called stromatolites, which oxygenated the
atmosphere. The Cambrian explosion was a sudden increase in complex multicellular life early n the
Paleozoic era. Organisms underwent further changes later.
• The Paleozoic era ended with a massive extinction, killing many species. The Mesozoic era started,
when the dominant life forms were conifers and dinosaurs.
• 65 mya, the Mesozoic ended and the Caenozoic started, with mammals and owering plants becoming
dominant. Human evolution also occurred in the latter stages of this era.
a. Missing Links:

• Archaeopteryx is the link between dinosaurs and birds  it has teeth, feathers, a bony long tail and
claws on its forelimbs.
• The coelacanth is the link between amphibians and sh. It has lobed ns, primitive internal organs
and nostrils that go right through into the mouth.
a. Fossils:

• Fossils are formed in sedimentary rocks only over long periods.


• They can be casts, moulds, mineralized body parts like bones / shells, imprints, lm fossils or even
dung samples.
• Fossils are dated using carbon 14 dating (fairly recent organic samples only) or radiometric dating for
older fossils.
• Fossil sites bring in many tourists and can be important for the economy.
• Some ancient organisms are seen as living fossils  they were believed extinct, but are still present,
e.g. the coelacanth and tuatara.
a. Key events in the history of life:

• Stromatolite fossils were found in Barberton and fossils of soft-bodied animals in Namibia.
• Ancient plant fossils like Glossopteris are present in central KZN and near Grahamstown.
• The coelacanth was `rediscovered' by the East London museum in 1938.
• Mammal-like reptile fossils were found in the Karoo  they have advanced palates and complex ear
bones, unlike true reptiles. They are called therapsids.
• Dinosaur fossils can be seen in the Drakensberg and Maluti mountains. This are also has fossils of the
earliest small mammals.
• Pre-human fossils like Australopithecus have been found in the Sterkfontein area and nearby places.
This is unique  nowhere else in the world do these fossils occur. The majority of these nds were
made by paleontologists from Wits University.
48 This content is available online at <http://cnx.org/content/m43085/1.1/>.

Available for free at Connexions <http://cnx.org/content/col11410/1.3>


275
4.2.8 4.2.8 - exercises49
4.2.8.1 Exercises

4.2.8.1.1 Question 1

1. In each of the following cases, write down the LETTER of the most correct alternative:
a. Which of the following fossils have been found in Namibia ?
1. Mammal-like reptiles
2. Glossopteris leaves
3. Soft-bodied animals
4. Early mammals
b. What was the importance of stromatolites ?
1. They released oxygen into the air.
2. They absorbed oxygen and prevented animals being poisoned.
3. They are the closest ancestors of modern plants.
4. They were algae trapped in sediment.
c. A problem in determining the accuracy of radiometric dating is that . . .
1. scientists are not sure that radioactive decay actually occurs.
2. the decay rate of the minerals can change without warning.
3. the rocks that contain the fossils can't be dated.
4. only organic samples can be dated radiometrically.
d. Continental drift accounts for the fact that people have found . . .
1. fern-like fossils in Africa.
2. the fossils of subtropical plants in Antarctica.
3. tectonic plates under the continents.
4. trilobite fossils in almost all continents.
e. Which statement below DOES NOT apply to the Paleozoic era?
1. Trilobites and algae were the dominant life forms.
2. Coal swamps were formed in many areas.
3. It started with the Cambrian explosion.
4. Birds developed during this era. [10]

4.2.8.1.2 Question 2

Match the terms in the rst column with the phrases / descriptions in the second column. Write down only
the correct LETTER next to the question number: [5]
Questions Column 1 Column 2
2.1 Mesozoic era A ancestor of frogs and sh
2.2 Coelacanth B are ancestral insect forms
2.3 Therapsids C are always solid fossils
2.4 Cast fossils D very supercial fossil imprints
2.5 Trace/ lm fossils E early mammals developed
F are always hollow fossils
G ancestral to birds and dinosaurs
H have advanced palates and ear bones
49 This content is available online at <http://cnx.org/content/m43184/1.1/>.

Available for free at Connexions <http://cnx.org/content/col11410/1.3>


276 CHAPTER 4. DIVERSITY, CHANGE AND CONTINUITY

Table 4.5

4.2.8.1.3 Question 3

Give the correct biological TERM for each of the following. Write down only the question number and the
answer.
1. The era during which ferns and cycads were the dominant plants
2. The ancient supercontinent
3. A scientists who studies fossils
4. The type of rock that may contain fossils
5. An ancient sh type found in very deep water at Sodwana Bay [5]

4.2.8.1.4 Question 4

Examine the two images below:

Figure 4.48

Available for free at Connexions <http://cnx.org/content/col11410/1.3>


277

Figure 4.49

1. Suggest THREE things that fossil footprints like those in picture A can tell scientists about the animal
that made them.(3)
2. Identify the fossil in picture B. (1)
3. Explain briey how this fossil was formed.(8)
4. What can we conclude about the area where this fossil is located millions of years ago? Give a reason
for your answer.(2)
5. Why is it that fossils are not found in igneous rock?(2) [16]

4.2.8.1.5 Question 5

Examine the fossil and graph below and answer the questions:

Available for free at Connexions <http://cnx.org/content/col11410/1.3>


278 CHAPTER 4. DIVERSITY, CHANGE AND CONTINUITY

Figure 4.50

Figure 4.51

1. To which class of the Chordata does the fossil shown here belong? Choose from: Pisces, Amphibia,
Reptilia, Aves or Mammalia.(2)
2. What does the term the half life of a radioactive element mean? (2)
3. When the fossil was subjected to radiometric dating, it was found to contain 90% Uranium. If the half
life of Uranium 235 is 704 million years, how old is this fossil? Show all your working.(4)
4. If another fossil was found and dated to 1408 million years, how much Uranium did it contain? Explain
how you obtained your answer. (5) [13]
Available for free at Connexions <http://cnx.org/content/col11410/1.3>
279
4.2.8.1.6 Question 6

Read the information below and answer the questions: Therapsid fossils from the late Permian have been
found in South Africa. These animals had teeth dierentiated into incisors, canines and molars, unlike
reptilian teeth, which are all the same. Their legs were held more vertically, i.e. under the body, compared
to what is usual in reptiles. It seems that they survived into the Mesozoic, but died out when the dinosaurs
became to dominant life form.
1. Where in South Africa have therapsid fossils been found?(1)
2. Tabulate TWO ways n which therapsids are dierent from reptiles. (5) [6]
TOTAL 55

4.2.8.1.7 Suggested answers

4.2.8.1.7.1 Question 1

1. C
2. A
3. C
4. B
5. D[10]

4.2.8.1.7.2 Question 2

1. E
2. A
3. H
4. C
5. D [5]

4.2.8.1.7.3 Question 3

1. Mesozoic
2. Pangaea
3. Paleontologist
4. Sedimentary
5. Coelacanth[5]

4.2.8.1.7.4 Question 4

1. Mark 1st 3 only: Animal's species, approximate length of leg, stride length approximate body weight,
speed it moved at(3)
2. part 2
a. Ammonite
b. Ammonite died, sank into mud. Mud was covered by more sand and excluded oxygen. Body
decayed, shell was slowly mineralized over time. Mud was compressed and became harder, turning
to stone. Later uncovered by erosion. (max 8)
c. It was under the sea. Ammonites were marine creatures(2)
3. Igneous rock is volcanic - lava is too hot to preserve fossils OR heat from lava destroys remains of
animals(2) [ 16]

Available for free at Connexions <http://cnx.org/content/col11410/1.3>


280 CHAPTER 4. DIVERSITY, CHANGE AND CONTINUITY

4.2.8.1.7.5 Question 5

1. Amphibia(2)
2. The amount of time needed to reduce the amount of (parent) element to half of what it was (OR the
time needed for half of the element to decay / break down)(2)
3. 0,2 x 704 million = 140,8 million years(4)
4. 1408 million years ÷704 million years = 2 half lives, so the fossil had 25% Uranium 235.(5) [ 13]

4.2.8.1.7.6 Question 6

1. Karoo(1)
2. Table comparing therapsids and reptiles : (5) [6]

Therapsid Reptile

Specialized teeth (or explain) Legs held under body All teeth look the sameLegs out to the sides
Table 4.6

TOTAL 55

Available for free at Connexions <http://cnx.org/content/col11410/1.3>


281
Solutions to Exercises in Chapter 4
Solution to Exercise 4.2.2.1 (p. 248)
COMPARING THE SKELETONS OF DINOSAURS, Archaeopteryx & BIRDS:

SIMILARITIES:
1 Jaws have teeth
2 Hand / arm has claws
3 Long bony tail present
4 Presence of gastralia or dermal ribs (not attached to spine)
Archaeopteryx vs Dinosaur:
DIFFERENCES:
1 Long forelimbs, like wings Short forelimbs
2 Feathers present No feathers
3 Hand has three claws Hand has ve claws
4 Furcula / wish bone present No furcula present
Archaeopteryx vs Modern bird
SIMILARITIES:
1 Feathers are present
2 Forelimbs are long and wing-like
3 Furcula / wish bone present (fused clavicles)
4 Bones of the lower forelimb are separate
Archaeopteryx vs Modern bird
DIFFERENCES:
1 Teeth in jaws No teeth in the beak
2 Claws on forelimbs Forelimbs without claws
3 Long bony tail Short tail bones / pygostral present
4 No breast bone Breast bone with a keel

SIMILARITIES:
1 Jaws have teeth

Available for free at Connexions <http://cnx.org/content/col11410/1.3>


282 CHAPTER 4. DIVERSITY, CHANGE AND CONTINUITY

2 Hand / arm has claws


3 Long bony tail present
4 Presence of gastralia or dermal ribs (not attached to spine)
Archaeopteryx vs Dinosaur:
DIFFERENCES:
1 Long forelimbs, like wings Short forelimbs
2 Feathers present No feathers
3 Hand has three claws Hand has ve claws
4 Furcula / wish bone present No furcula present
Archaeopteryx vs Modern bird
SIMILARITIES:
1 Feathers are present
2 Forelimbs are long and wing-like
3 Furcula / wish bone present (fused clavicles)
4 Bones of the lower forelimb are separate
Archaeopteryx vs Modern bird
DIFFERENCES:
1 Teeth in jaws No teeth in the beak
2 Claws on forelimbs Forelimbs without claws
3 Long bony tail Short tail bones / pygostral present
4 No breast bone Breast bone with a keel

SIMILARITIES:
1 Jaws have teeth
2 Hand / arm has claws
3 Long bony tail present
4 Presence of gastralia or dermal ribs (not attached to spine)
Archaeopteryx vs Dinosaur:
DIFFERENCES:

Available for free at Connexions <http://cnx.org/content/col11410/1.3>


283
1 Long forelimbs, like wings Short forelimbs
2 Feathers present No feathers
3 Hand has three claws Hand has ve claws
4 Furcula / wish bone present No furcula present
Archaeopteryx vs Modern bird
SIMILARITIES:
1 Feathers are present
2 Forelimbs are long and wing-like
3 Furcula / wish bone present (fused clavicles)
4 Bones of the lower forelimb are separate
Archaeopteryx vs Modern bird
DIFFERENCES:
1 Teeth in jaws No teeth in the beak
2 Claws on forelimbs Forelimbs without claws
3 Long bony tail Short tail bones / pygostral present
4 No breast bone Breast bone with a keel

Available for free at Connexions <http://cnx.org/content/col11410/1.3>


284 CHAPTER 4. DIVERSITY, CHANGE AND CONTINUITY

Archaeopteryx vs Dinosaur: SIMILARITIES:


1 Jaws have teeth
2 Hand / arm has claws
3 Long bony tail present
4 Presence of gastralia or dermal ribs (not attached to spine)
Archaeopteryx vs Dinosaur: DIFFERENCES:
1 Long forelimbs, like wings Short forelimbs
2 Feathers present No feathers
3 Hand has three claws Hand has ve claws
4 Furcula / wish bone present No furcula present
Archaeopteryx vs Modern bird SIMILARITIES:
1 Feathers are present
2 Forelimbs are long and wing-like
3 Furcula / wish bone present (fused clavicles)
4 Bones of the lower forelimb are separate
Archaeopteryx vs Modern bird DIFFERENCES:
1 Teeth in jaws No teeth in the beak
2 Claws on forelimbs Forelimbs without claws
3 Long bony tail Short tail bones / pygostral present
4 No breast bone Breast bone with a keel
Table 4.7

Solution to Exercise 4.2.2.2 (p. 250)

a. Data
b. Conclusions
c. Conclusions

Available for free at Connexions <http://cnx.org/content/col11410/1.3>


Chapter 5
1
Glossary

Abiotic Describes the non-living elements of the ecosystem


Abort To purposefully end a pregnancy before full term
Absorb To take something in. For example if a cell takes in water it absorbs water.
Absorption (of nutrition) Taken into the cells of the body
Accessory Something extra which is added to make the thing complete
Accommodation Altering the shape of the lens of the eye to focus on close objects
Acetylcholine A chemical that carries a nerve impulse from one neuron to the next across the synapse
Acid A chemical substance that releases hydrogen ions (H+) in water and has a pH less than 7
Acquired A feature which occurs because of what has happened during the life of the
organism eg. an acquired characteristic could be a scar on the skin
Acromegaly Larger hands, feet and bones caused by too much growth hormone as an adult
Activation energy The energy needed for a reaction to take place
Active site Place on an enzyme molecule where a reaction happens; it ts the substrate
exactly
Active transport Movement of substances using respiration energy, needed for movement against a
concentration gradient
Adaptation A change to suit the conditions
ADH AntiDiuretic Hormone from the posterior pituitary; makes the collecting duct of the kidney more
permeable to water, reducing the volume of urine
Adhesion When a substance sticks on to a dierent kind of substance (compare Cohesion).
ADP Adenosine Diphosphate, a chemical that can take up an extra phosphate when supplied with
chemical energy
Adrenal gland An endocrine gland found near the kidney with an outer cortex (that makes
cortisone and aldosterone) and inner medulla (that makes adrenalin)
Adrenalin The hormone from the adrenal medulla that prepares the body for stress
Aerobic Uses oxygen
Aerent Carries something to a structure (compare this with "eerent")
Aldosterone A hormone that regulates water retention in the kidneys by controlling the
distribution of sodium ions
Alien Foreign to that place: in life sciences a species introduced by people
Alimentary canal The tube that leads from the mouth in which food is broken down and absorbed
for the body
Alkali An hydroxide chemical which dissolves in water to give hydroxyl ions (OH ) and a pH greater
-

than 7
Allergen Something that causes an allergic reaction in the body
1 This content is available online at <http://cnx.org/content/m43254/1.1/>.

Available for free at Connexions <http://cnx.org/content/col11410/1.3>

285
286 CHAPTER 5. GLOSSARY

Allergy A more than normal sensitivity to a common substance


Alveolus Small, thin walled bag in the lungs containing air; it is where gaseous exchange happens
Amino acid An organic molecule with an acid (-COOH) group and an amino (-NH ) group. It is
2
the building block of proteins
Amino group The -NH 2 arrangement of atoms which is found in all amino acids and therefore all
proteins
Amniocentesis Procedure to sample the cells of a foetus by removing some uid surrounding it in the
womb
Amnion The innermost extra-embryonic membrane; it makes a uid lled sac
Amniotic uid The protective liquid around the embryo and foetus; it is enclosed by the amnion
Amplify To increase the strength or loudness
Ampulla A bulge at the end of each semicircular canal where acceleration receptors are found
Anaemia A deciency disease caused by lack of iron which reduces the number of red blood cells,
aecting the ow of oxygen to the tissues
Anaerobic Does not use oxygen
Analagous Doing the same job as the other structure; compare with `homologous'
Androgen Male sex hormone produced by the testes; they develop and maintain the male sexual
characteristics eg. testosterone
Aneuploidy A mutation in which the number of chromosomes changes eg.47 chromosomes in Down's
syndrome
Anorexia A disease in which a person refuses to eat and cannot see that s/he is becoming ill
Anterior To the front of the organism
Anther The box on top of a stamen where pollen is made
Antibody A protein in the blood which ghts disease; made in response to the presence of foreign
substances
Antibiotic A chemical substance made by a micro-organism that stops the growth of other micro-
organisms
Antigen A substance which is foreign to the body and causes antibodies to be made
Antiseptic A chemical that kills germs such as bacteria and fungi
Aorta The largest artery in the body, leading from the left ventricle of the heart to all the body except
the lungs
Aqueous humour The watery material lling the space in front of the lens of the eye
Arachnoid The middle of the three membranes covering the brain and spinal cord
Artery A blood vessel which carries blood away from the heart towards capillaries
Assimilation (in nutrition) Becoming a part of the body
Asthma Disease causing breathing diculty; it is often caused by an allergen
Atherosclerosis A deposit of fatty substance leading to raised blood pressure and heart disease
Atom The smallest part of an element which has the properties of that element
ATP Adenosine Triphosphate, a chemical that holds energy for the cell (see ADP)
Atrium The chamber of the heart which collects blood from veins and passes it to the ventricle
Autonomic Self- governing and so acting without conscious thought ie. involuntary
Axon A long extension of a nerve cell carrying an impulse away from the cell body
Bacteria A very small single celled organism of the kingdom Prokaryota
Bar graph Graph using separate columns that is used when the independent variable is not numerical
eg types of food
Benedict's reagent A blue solution with Copper salts that gives a red precipitate when boiled with
a reducing sugar.
Beriberi The deciency disease caused by lack of vitamin B1 (thiamine). Symptoms include paralysis
of the digestive system and poor co-ordination
Bicarbonate indicator A substance that contains bicarbonate and changes colour with dierent
acidity (red <pH7, yellow >pH7)

Available for free at Connexions <http://cnx.org/content/col11410/1.3>


287
Bicuspid The heart valve which lies between the left atrium and the left ventricle
Bile An alkaline juice made by the liver that emulsies fats and neutralises acidity when it enters the
duodenum through the bile duct
Binary ssion A form of asexual reproduction in single-celled organisms when they split into two
equal cells
Biochemistry The study of the working of the chemicals of the body
Biodiversity The whole variety of living things found in that dened environment
Biotic Describes the living elements of the ecosystem compare with `abiotic'
birth control Any method of having a sexual relationship without causing a pregnancy
Biuret test The test for Protein. Add dilute NaOH then a few drops of dilute (1%)Copper
Sulphate. A purple colour indicates protein
Bivalent (in life sciences) A structure made from two homologous chromosomes joined together during
meiosis
Bladder Muscular bag that stores the urine which is made continuously in the kidney
Blastocyst The hollow ball of up to 64 cells formed from a zygote in mammal reproduction
Blind spot The part of the retina with no receptors since neurons leave the eye at that point
Blood plasma The matrix of blood tissue, the liquid part of the blood
Blubber A thick layer of fat under the skin to separate the warm body of some mammals from the
cold outside
B-lymphocyte A type of white blood cell, made in the bone marrow, found in a lymph node
Bolus A small ball of food which is swallowed and moved down the gullet by peristalsis
Bone marrow A soft tissue in the centre of some bones that makes blood cells
Bowmans capsule Cup shaped structure in the kidney into which the capillaries of the glomerulus t
Brain The swollen front end of the spinal cord, with a large mass of interconnected neurons from where
the body is controlled
Breathing The movement of air from the atmosphere into the lung air spaces and then out again;
otherwise called ventilation
Breathing centre Region of the medulla oblongata which controls the depth and rate of breathing
Bronchiole Thin branch of a bronchus that carries air to the alveoli; has no cartilage rings
Bronchitis Disease caused by an inammation of the bronchi of the lung
Bronchus One of the main air pathways, strengthened with cartilage rings, that carries air into, and
through a lung
Budding A type of articial asexual reproduction when a bud is grafted onto a scion
Bulimia A disease in which a person purposely brings up the food eaten, so losing weight
Caecum Pouched piece of intestine where herbivores digest cellulose, in humans the
appendix.
Calyx The collective term for all the sepals of one ower
Canine tooth The large, pointed "fang" tooth
Capillarity Movement of a liquid up narrow tubes due to surface tension
Capillary The smallest blood vessel with a thin, leaky wall so that liquid can pass out
Capsule The outer protective "skin" around a kidney to hold the tissues in place
Carbohydrate A compound of Carbon, Hydrogen and Oxygen with twice as many Hydrogen
atoms as Oxygen atoms eg C6H12 O6
Carbon xation Holding the carbon of carbon dioxide gas in the leaf in a new form so it cannot
escape; it becomes attached to ribulose biphosphate
Carbon monoxide A poisonous, colourless gas CO: it stops haemoglobin from being able to carry
oxygen
Carboxyl The arrangement of atoms -COOH which is found in all organic acids
Carcinogen Any substance that causes a cancer to develop
Carpel One unit of the female reproductive parts of a ower
Cardiac Of the heart

Available for free at Connexions <http://cnx.org/content/col11410/1.3>


288 CHAPTER 5. GLOSSARY

Cartilage Supporting tissue which is softer and more exible than bone; cells called
chondrocytes
Casparian strip A layer of waterproof suberin on the walls of endodermal cells in a plant root that
stops water moving through cell walls, directing it into the stele
Catalyst Substance that speeds up a chemical reaction but does not get used up itself
Cerebellum Swollen part of the hind brain that controls muscle movement and balance
Cerebro-spinal Of the brain and spinal cord; cerebro-spinal uid lls the spaces in these organs
Cerebrum Swollen part of the front brain that controls voluntary actions, memory, reasoning etc
Chemical bond The link of one atom to another in a molecule; the most common bonds are ionic
bonds and covalent bonds
Chemical energy The ability to do work which is held inside a molecule; for example. in a chemical
substance like ATP
Chemotherapy A medical treatment that uses poisonous chemicals to kill cancerous cells
Chlorophyll A green substance that has the ability to convert radiant energy into chemical energy
Chloroplast The green organelle containing chlorophyll found in some plant cells
Chlorosis A disease of plants when the leaves are yellow instead of green. It is caused by lack of light,
or iron or magnesium deciency
Cholesterol A waxy substance found in all cells, it can cause disease if there is too much in the blood
Chorion The outer extra-embryonic membrane; it protects the foetus and helps form the placenta
Choroid The middle of the three layers of the eye; it is black to reduce reection and carries blood
vessels to feed the tissues
Chromatin The substance that chromosomes are made of; in interphase it is very thin threads of DNA
together with ve special proteins called histones
Chromosome One of a xed number of thread-like structures in a cell nucleus; it carries genes and is
seen during nuclear division
Chyme The liquid mixture of food and juices that moves down the gut from the stomach
Cilia Tiny hair-like structures on the surface of some cells; they move to push things along
Ciliary body The swollen region of the choroid of the eye near the lens that holds the ciliary muscles
Ciliary muscle Muscles that control the shape of the lens of the eye, they contract for
accommodation
Ciliated epithelium A thin internal skin whose cells are covered in cilia
Cochlea A coiled tube that is a part of the inner ear and contains the receptors for the sense of hearing
Codominance When there is the full expression of both of two alleles because neither of them is
dominant to the other
Coenzyme A substance that helps an enzyme to work properly
Cohesion When a substance sticks to itself.
Collecting duct A tube that collects urine from the nephrons and feeds it into the kidney pelvis
Colon The main part of the large intestine where water and some mineral salts are
absorbed and the faeces form; rich in bacteria producing vitamins B and K
Comet A lump of frozen dusty material that orbits around the sun
Compound A substance that it made from two or more dierent elements
Concentration The strength of a substance in a mixture. It is a measure of the number of
molecules present.
Condensation The joining of two molecules by taking a water molecule from them (see Hydrolysis)
Cone A photoreceptor cell which is sensitive to bright light and colour
Conjunctiva The transparent membrane covering the eyeball and attached to the eyelids
Connective tissue Any animal tissue that provides support, packing and insulation between the
organs of the body; usually the cells are found in a large amount of matrix
Constipation Disease in which it is dicult to release faeces
Constrict To become narrower

Available for free at Connexions <http://cnx.org/content/col11410/1.3>


289
Continental drift A theory explaining how the major land masses of earth have moved over time due
to plate tectonics
Control The part of an experiment where the manipulated variable is controlled or is missing so the
experimenter can check that the variable is causing the eect
Convex With a surface that curves outwards towards the edge
Convoluted Very highly folded and tangled up
Co-ordination Causing separate parts to work together as one unit
Cornea The transparent part of the sclera at the front of the eye
Coronary artery One of the arteries that surround the heart and feed its muscles
Corpus callosum Bundle of nerve bres making the connection between the right and left cerebral
hemispheres
Corolla The collective term for all the petals of one ower
Cortex The outer region of an organ such as the kidney
Cotyledon The part of the embryo of a plant used to provide food; owering plants have either one
(monocots) or two (dicots) in each seed
Counter current A system where two liquids ow close to each other in opposite directions
Cranial nerves Peripheral nerves that come from the brain part of the central nervous system
Cretinism A condition of having a poorly developed brain and body due to lack of thyroxin
Cristae Folds on the inner membrane of a mitochondrion
Cross pollination The transfer of pollen from the male part of one ower to the female part of another
ower of the same species
Cupula Jelly-like structure in the ampulla which moves when the head is moved
Cusp The place where two curved surfaces meet
Cuticle The waxy, waterproof outer layer of a plant organ such as the shiny coating on a leaf
Cytoplasm All the living substance of a cell except for the nucleus; it includes the other
organelles.
Cytosol The liquid part of the cell which is the cytoplasm without its membraneous organelles
Dark phase (stage) The part of the process of photosynthesis that is independent of light; when
carbon dioxide is xed and then reduced to make carbohydrate
Data Known facts which can be used for analysis and interpretation
Deamination Removing the amino group (-NH2) from an amino acid to leave C, H and O only
Deciency When there is not enough of something to do the job properly
Denature When a protein loses its shape due to changes in acidity or to high temperatures; then it
can no longer carry out its job
Dendrite A shorter extension of a nerve cell; bringing the impulse to the cell body
Dental formula A statement of the number of each type of teeth in one half of the upper and lower
jaw of a mammal species
Deoxyribose A ve carbon sugar with one atom of oxygen less in its molecule than a normal sugar;
one of the building blocks of each DNA nucleotide
Destarch To take starch out; leave a plant in the dark for 24 hours to move starch out of its leaves
Diabetes A disease when insulin does not function properly causing blood sugar to rise when carbo-
hydrates are eaten. In type 1 diabetes no insulin is made, in type 2 diabetes the insulin loses its eect on
the cells.
Diaphragm A sheet of tendon, with muscles on it, that separates the thorax from the abdomen
Diastole The relaxed part of the heart beat
Dicotyledon (Dicot) One of the large group of plants that has two cotyledons in each seed, ower
parts in ves and net veined leaves
Diusion The spread of a substance caused by the natural random movement of its molecules
Digestion Breaking down complex foods into simple, soluble nutrients
Dilate To get wider

Available for free at Connexions <http://cnx.org/content/col11410/1.3>


290 CHAPTER 5. GLOSSARY

Disaccharide A sugar with molecules of two sugar units joined; it can split into two parts which are
still both sugars (compare with monosaccharide)
Dissolve To make something into a solution (become mixed with a liquid)
Distal Far (or distant) from the main part of the body
Dominant In genetics a characteristic that appears in the phenotype whenever it is present on at least
one homologous chromosome compare with `recessive'
Dorsal On the upper side of the organism
Double fertilisation The special fertilisation process in which two male nuclei are needed; it is unique
to owering plants
Duodenum The rst part of the small intestine, it is about 30 cm long. The bile and pancreatic juice
run into it.
Dura mater The strong outer membrane of the three that cover the brain and spinal cord
Dynamic Of movement; dynamic balance is the force of acceleration on the body
Ectothermic Getting body heat from the surroundings (compare with endothermic)
Eector A cell or organ (usually muscle or gland) that an animal responds with
Eerent Carries something away from a structure; for example eerent arterioles carry blood away
from Bowmans Capsule
Egestion Removal from the body of the waste products that could not be digested
Electron A tiny, negatively charged particle that is a part of all atoms
Element A substance made from only one of the 92 dierent sorts of atom that exist naturally; it
cannot be split into simpler substances
Emulsion A colloidal mixture of two liquids that do not dissolve, one of which is spread through the
other
Enamel The hard resistant substance on the outer surface of a tooth
Endocrine Of the system of ductless glands that release hormones directly into the blood
Endolymph The uid that lls the canals made of membrane found in the inner ear
Endoplasmic reticulum A complex system of membranes throughout the cytoplasm; they act as
channels for transporting materials
Endosmosis Osmosis of water into a cell; compare with Exosmosis.
Endosperm The food supply of a seed that is not inside a cotyledon
Endothermic Getting body heat from the working of the body; that is, the body heat is from inside
the body
Energy The ability to do work
Enterokinase An enzyme of the duodenum that changes inactive trypsinogen into the active enzyme
trypsin
Environment The surroundings of an organism or cell
Enzyme A protein that activates and speeds up chemical reactions in an organism
Enzyme activator Substance that must be added to an enzyme before it is able to work
Epiglottis Flap of tissue which closes the trachea during swallowing
Equator An imaginary line round the middle of a structure; the line of the middle of the cell during
cell division
Equilibrium To be in balance and therefore apparently not changing
Erosion The loss of soill by wind or water action
Erythrocyte Red blood cell
Ester link The special bond that joins the fatty acid parts of a fat molecule to the glycerol part
Ethanol Ethyl alcohol (C2H5OH), the alcohol found in drinks such as beer
Eustachian tube A tube leading from the middle ear to the throat; it allows the air pressure to be
equalised on each side of the eardrum
Eutrophication When too much nitrate and phosphate enters a body of water, leading to a shortage
of oxygen and the death of many species
Evaporation Being lost from the surface as a vapour or a gas.

Available for free at Connexions <http://cnx.org/content/col11410/1.3>


291
Excretion The removal from the body of the waste products of metabolism
Exhale To breathe out
Exocrine Of the system of glands with ducts that produce enzymes, etc but not hormones; compare
with endocrine
Exosmosis Osmosis of water out of a cell
Fat An ester of glycerol with three fatty acids; it makes molecules that are important in cell membranes,
energy foods etc
Fatty acid An organic acid (R-COOH) with many units of Carbon and Hydrogen making up the
radical "R"
Fermentation Anaerobic respiration in plants, fungi etc to produce alcohol
Fertilisation In life sciences i) when the male gamete and the female gametes fuse to make a zygote;
ii) providing mineral salts to improve soil fertility
Fibrinogen A soluble blood protein that can change into insoluble brin to form a clot when a blood
vessel is cut and the blood exposed to the air
Filament In owers, the stalk part of a stamen, it holds the anther up for pollination
Filtrate The substances which have passed through a lter and not been held back
Flaccid Soft and oppy.
Focus To adjust so that a sharp image is made
Food The materials taken in to the gut so that nutrients can be extracted
Food chain The pathway of food in an ecosystem from the producers to primary consumers, then
secondary consumers, etc
Formalin A poisonous chemical that it is very good at killing bacteria
Fovea The part of the retina with most cones so the image made here is the clearest
Fructose A hexose sugar that is common in fruit. It is an isomer of glucose C6H12O6
Fruit The ripened ovary of a owering plant, it may be eshy or dry and assists with seed dispersal
Fuel A substance that contains energy that can be released
Gall Bladder A bag-like container on the underside of the liver to store bile until the duodenum needs
it
Gamete A sex cell with a haploid set of chromosomes; large less mobile ones are called female, smaller
more mobile ones are called male
Gaseous exchange Swopping oxygen from the air with the carbon dioxide made by the body; compare
this with breathing and respiration
Gastric Of the stomach
Gene A short length of a chromosome that contain a unit of genetic information; often each gene
carries the code for making one protein
Genetic code The way the information about an organism is stored in DNA molecules; each amino
acid is represented by a triplet of N bases
Genetics The study of inheritance
Genome The full set of genes that an individual has; also the range of genes in a species
Genotype The set of alleles for a characteristic that an organism has inherited
Genus A group of dierent species with many features in common
Germinal epithelium A layer of tissue in the sex organs of animals that makes new cells by meiosis
Gland A part of the body which makes a useful substance for the body
Global warming The recent rise in average temperature all over the world caused by extra
greenhouse gases in the air
Globulin A blood protein with a folded shape so that it forms a ball shaped molecule
Glomerulus A knot of very narrow capillaries that sits in Bowman's capsule of the kidney
Glucagon A hormone from the alpha cells of the Islets of Langerhans in the pancreas; it causes an
increase in the amount of glucose in the blood
Glucose A common, simple hexose sugar that is made by photosynthesis and is the main respiratory
substrate. Its formula is C H O
6 12 6

Available for free at Connexions <http://cnx.org/content/col11410/1.3>


292 CHAPTER 5. GLOSSARY

Glycerol An alcohol with three hydroxyl units (CH OH CHOH CH OH) that is part of the
2 2
molecule of all fats
Glycogen A polysaccharide used as the store of carbohydrates in animal cells
Glycolysis The rst stage of respiration when 1 molecule of glucose changes into 2 pyruvic acid
molecules
Gonad A sex organ that produces gametes
Graaan follicle A structure in the ovary that is lled with uid; the female gamete develops inside
it until ovulation
Gracile Delicately formed; compare with `robust'
Granulocyte A white blood cell with a large irregular nucleus and granules in the cytoplasm
Granum The structure holding chlorophyll in a chloroplast, made of a pile of thylakoids.
Grey matter Nervous tissue that is mostly cell bodies with few myelinated bres (see white matter)
Growth hormone A hormone (from the anterior pituitary gland in animals) which promotes an
increase in size
Guard cell One of the two curved cells that surround the pore (or hole) of a stoma, allowing it to
open and close.
Guttation When water drops are squeezed out of hydathodes on a leaf onto its surface
Haemoglobin The red coloured conjugated protein that can carry oxygen; it is found in erythrocytes
Haemophilia The genetic bleeding disease, caused by a person not being able to make clotting factor
so wounds do not heal
Health The state of being well, with body systems working eciently
Helper cell The kind of T-lymphocyte that helps B-lymphocytes to produce antibodies
Heat exchanger A system where heat is moved from one place to another
Hepatic Of the liver
Herbicide A chemical substance that can kill green plants
Heredity The process in which characteristics are passed on from parents to their ospring
Hermaphrodite When one organism has both male and female sex organs eg earthworms and most
owering plants
Heterotrophic Uses complex organic foods to get its nutrients (compare with autotrophic)
Hexose A sugar which has 6 carbon atoms in its molecule
Histamine A chemical given out by mast cells during the allergic response; it causes dilation of blood
vessels
Histone A special protein that attaches to DNA molecules and is used to package the DNA; DNA plus
histone makes chromatin
Histogram Graph using columns that is used when the independent variable data is grouped according
to classes
Homeostasis Maintaining a constant internal environment
Homeothermic Having a constant body temperature which is maintained by homeostasis
Hominid The group of primates that use two legs for walking; only Homo sapiens still lives, the other
species are extinct
Homologous Structures that look similar to each other or have the same origin
Hormone A chemical messenger substance made in ductless glands and carried in the blood to act
somewhere else
Humid The air contains a lot of water vapour.
Hydathode A gland on the surface of a leaf that forms water drops for guttation
Hydrogen bond A very weak chemical bond between the negatively charged part of one molecule and
the positively charged part of another molecule
Hydrolysis When a large molecule is split into two smaller parts by adding a water molecule to it
(compare with Condensation)
Hydrostatic The pressure caused by a liquid such as water when at rest
Hypertonic Having a lower solute potential than the reference solution

Available for free at Connexions <http://cnx.org/content/col11410/1.3>


293
Hypophysis Another name for the pituitary gland in the middle of the head, below the brain
Hypothalamus The oor of the thalamus (front brain) which links the nerve and hormone systems
for better body co-ordination
Hypothesis Statement that is a possible explanation which can be tested by experiment to nd out if
it is true or not
Hypotonic Having a higher solute potential than the reference solution (compare with
Hypertonic)
Ileum The longest part of the small intestine. It is about 4 m long in humans. Most
absorption happens in it
Impermeable Substances are not able to pass through it
Implantatio n The process in which a new embryo settles onto the wall of the uterus
Impotent In males, unable to have sexual relations because the penis will not erect
Incomplete dominance The blending of the expression of two alleles because neither of the pair is
dominant to the other
Impulse In life sciences, the message that passes along a nerve
Incisor The front chisel shaped teeth
Indicator A substance which changes colour to show that another substance is present eg. litmus goes
red if acid present
Indigenous Native to the place and not introduced there by people
Infectious A disease caused by a living organism that can pass from one person to another
Inammation A redness or swelling of tissue in response to an injury
Ingestion Taking in food at the mouth
Inhale To breathe in
Inhibit To slow down or reduce the eect
Inorganic Substance that is not organic, it usually does not have large molecules and usually does not
have carbon.
Insoluble Does not dissolve
Insulation A substance to prevent the movement of something (for example of heat)
Insulin A hormone from the beta cells of the Islets of Langerhans in the pancreas; it causes a reduction
in the amount of glucose in the blood
Intercostal muscle Muscle between the ribs; the external intercostals raise the ribs to breathe in
Intra-specic A process involving individuals of the same species; intraspedic competition is when
individuals of one species are trying to get the same resources
Invasive Of an organism that easily spreads to other places
Involuntary Not controlled by the will; it is automatic
Iodine solution A mixture of Iodine in Potassium Iodide solution. It causes starch to go blue black
Ionised An atom or molecule that has become electrically charged because it has lost or gained electrons
so is no longer electrically neutral
Iris The circular coloured membrane in front of the lens of the eye
Isomers Substances which are dierent, but have the same chemical formula eg glucose, fructose and
galactose are isomers with the formula C6H12 O6
Isotonic Having the same solute potential as a reference solution
Karyotype A description of the number, size and shape of the chromosomes of a species
Kidney The organ for excretion and osmoregulation containing nephrons and blood vessels
Killer cell A type of T-lymphocyte that destroys cancerous or virus infected cells
Kilojoule (kJ) A measure of the amount of energy, for example heat energy. It is one thousand joules.
1kJ will raise the temperature of 1 litre of water by 1 ◦ C
Krebs' cycle The second stage of aerobic respiration when pyruvic acid changes to CO2 and hydrogen
Kwashiorkor A deciency disease caused by lack of protein in the diet of young children;
symptoms include slow growth, enlarged belly and reddened hair
Labyrinth A complicated network of passages eg. the passages of the inner ear

Available for free at Connexions <http://cnx.org/content/col11410/1.3>


294 CHAPTER 5. GLOSSARY

Lacteal A tiny lymph vessel in the centre of a villus on the lining of the small intestine
Lactic acid An organic acid (CH3 CHOH COOH) which is made in muscles during anaerobic respi-
ration, causing an oxygen debt
Larva The sexually immature form of an animal that hatches from an egg and does not resemble the
parent
Larynx The voice box; the part of the air passage above the trachea and below the pharynx
Latent heat Energy that is taken up by a substance when it changes from a liquid to a gas
Lens The curved transparent structure which can bend light to focus it at a point
Leucocyte Any white blood cell
Light phase (stage) The part of the process of photosynthesis that needs light; during the light stage
water is split and ATP made, with oxygen released as a waste
Lignin The chemical substance of wood. It lines cell walls to makes them strong and
waterproof.
Lime water A chemical that turns milky in the presence of CO2 so is used to nd out if it is present
Line graph Graph using points joined by lines that is used when both variables are numerical and
continuously variable
Lipase An enzyme of the small intestine which digests lipids into glycerol and separate fatty acids.
Liver The largest organ in the body, found just under the diaphragm. It acts as the
chemical factory of the body
Loop of Henle A hairpin shaped part of the nephron that is mostly in the kidney medulla; involved
in controlling water loss in the urine
Lung The organ whose job is to exchange waste carbon dioxide from the body for oxygen from the air
Lymph A colourless liquid that comes from tissue uid; it does not go directly back into the blood
but collects in lymph ducts and returns to the blood later
Lymph node A small swelling in a lymph vessel which helps protect the body from disease
Lymph vessel A vessel that carries lymph (formed from the tissue uid) instead of blood
Lymphocyte A white blood cell with a large round nucleus; it makes antibodies
Macroevolution Slow changes in a species with time, making it suciently dierent to cause
extinction, or the production of a dierent species
Macronutrient An element that is needed in quite large amounts by a living organism eg. Calcium
and phosphorus
Macrophage A type of very large white blood cell that ingests foreign particles to destroy them; they
are found all over the body, not just in the blood
Macroscopic Can be seen with the naked eye
Macula Sensory structure in the sacculus and utriculus of the inner ear; it contains otoliths on hairs
which are sensitive to gravity
Male gamete The sex cell produced by a male organism; it is very small and motile
Malnutrition A disease caused by the diet not being balanced
Mammal The class of vertebrate animals with fur which feed the newly born from milk glands
Marasmus A deciency disease caused by lack of food in young children; it causes thin limbs and
wasted body, lack of energy and poor growth
Mast cell A cell found in connective tissue; it is similar to a white blood cell and involved in the
immune response to parasites
Mastication Chewing, using the teeth to physically digest food
Matrix The substance that lls the space; for example the liquid part in the middle of a mitochondrion
Median canal The central canal of the cochlea which is lled with endolymph
Medulla The inner region of an organ such as the kidney
Medulla oblongata The region at the bottom of the hindbrain where it joins onto the spinal cord; it
contains reex centres
Meiosis The double nuclear division that reduces the chromosomes number to the haploid condition
Meninges The three membranes that cover the brain and spinal cord

Available for free at Connexions <http://cnx.org/content/col11410/1.3>


295
Menstrual cycle A monthly cycle of change to the womb and ovaries of a woman; it controlled by the
female hormones called oestrogens
Mesophyll The tissue that makes up the middle of a leaf. The lower part is usually spongy cells and
the upper part is palisade cells
Metabolic rate The speed at which the body is working
Metabolism The chemical reactions of life
Meteorite Any natural lump of material that falls to earth from space
Methane A colourless gas (CH 4 ) released by decaying organic matter that is a powerful greenhouse
gas
Microevolution Slow changes in the characteristics of a species with time that are not enough to form
a new species
Micronutrient An element needed in very small amounts by a living organism eg. iodine
Micropyle A small hole at the end of the ovule of owering plants that lets the male gametes in; it
remains as a hole in the testa of the seed to allow water in
Microscopic Cannot be seen with the naked eye but needs a system of lenses to see it
Microvillus A very tiny nger-like projection on a cell membrane that can be imaged with an electron
microscope
Milligram (mg) One thousandth of a gram. One gram is the standard unit of mass.
Mitochondrion An organelle with two layers of membrane that holds the enzymes for aerobic respi-
ration
Mitosis The nuclear division that is a part of somatic cell division; it produces two identical daughter
nuclei
Monocotyledon The group of plants with one cotyledon in each seed; the ower parts are in threes
and the leaves have parallel veins (compare with `dicotyledon')
Molar The back, broad crushing teeth; they are not found in the milk dentition
Molecule The smallest part of a chemical substance still having the properties of the substance;
compare with atom
Monocyte A very large white blood cell which is able to absorb and digest disease organisms
Monomer The building block of a larger molecule (a polymer); the polymer is made by joining many
of them together
Monosaccharide A sugar that has molecules made of one unit, it cannot be split into a smaller
molecule that is still a sugar; compare with `disaccharide'
Motor nerve Bundle of neurons oncerned with causing a movement such as a muscle movement
Mucin A slippery protein substance that is called mucus when mixed with water
Mutation Any spontaneous change in the DNA of a cell; it may happen to a gene or to one ormore
chromosomes
Myelin sheath A fatty protective layer round an axon; it stops impulses moving to other axons nearby
and speeds up the impulse
Myxoedema Swelling or puness of the body due to water being held in the tissues
NAD Nicotinamide Adenine Dinulceotide; a coenzyme that carries hydrogen atoms during respiration
NADP NAD phosphate; a coenzyme that carries hydrogen atoms during photosynthesis
Negative feedback When the product of a process aects the production in the opposite direction;
so a rise in the product causes a fall in the product, keeping it constant
Nephron A kidney tubule, the working unit of a kidney
Neuron A nerve cell
Nissl granule Small pieces of RNA that can be seen in nerve cell bodies using a good microscope
Node of Ranvier A gap in the myelin sheath, allowing the nerve impulse to travel more quickly
Non-disjunction This is when the two chromosomes in a bivalent do not separate during the rst
division of meiosis; as a result some nuclei have extra chromosomes
Nutrient A chemical substance that the body needs to keep healthy; usually carbohydrate, protein,
lipid, vitamin or mineral

Available for free at Connexions <http://cnx.org/content/col11410/1.3>


296 CHAPTER 5. GLOSSARY

Nutrition When food is taken in from the environment and then used in the body (it means the same
as feeding)
Obesity A disease caused by eating too much energy food that becomes stored as fatty tissue
Oesophagus The gullet. A tubular part of the alimentary canal which passes from the throat through
the thorax to the stomach
Optic nerve The nerve that carries sensory neurons from the eye to the brain
Optimum The most favourable conditions
Organic A compound with large molecules that contains carbon and usually hydrogen and oxygen
Osmoregulation Controlling the water content of an organism
Osmosis The diusion of water through a dierentially permeable membrane to dilute a more concen-
trated solution.
Ossicle One of the three small bones of the middle ear. The Malleus (hammer), Incus (anvil) or Stapes
(stirrup) bones
Otolith Chalky grains which are a part of the macula, allowing for a sense of gravity
Ovary Any organ where the female gametes of an organism are made
Ovule The structure in owering plants that contains the female gamete; it becomes the seed after
fertilisation
Oxidation In chemistry, the adding of oxygen or removal of hydrogen OR the taking away of one or
more electrons from a molecule
Oxidative Carries out oxidation, or the removal of hydrogens
Oxygen debt The need to take in extra oxygen to remove lactic acid formed anaerobically, which
otherwise cannot leave the body
Ozone A colourless gas (O3); it is poisonous but protects the earth from dangerous
ultraviolet radiation
Pacemaker A region in the right atrium of the heart with nerves controlling the heart beat
Pancreas A gland of the digestive system, with a duct that opens into the duodenum; it is both
endocrine and exocrine
Pandemic A disease that has become very common and very widespread eg. the AIDS pandemic
Pant To breathe quickly across a wet surface (eg the tongue) so as to lose heat
Pathogen An organism that causes disease
Pelvis The inner part of the kidney where the urine collects before running o down the ureter
Pepsin An enzyme of the stomach that digests proteins into smaller units such as dipeptides; it is
made as inactive pepsinogen, then activated in acid conditions
Peptide bond The special bond that joins two amino acids together.
Perilymph The uid that is round the outside of the canals made of membrane in the inner ear
Peripheral Near the edge; the nerves that go from the Central Nervous System towards the edge of
the body are called the peripheral nervous system
Peristalsis A squeezing motion caused by waves of alternate contraction of circular and longitudinal
muscles
Petal One of the second ring of modied leaves in a ower; they are often brightly coloured to attract
pollinators
pH How acid or alkali something is: pH 7 is neutral, below pH7 is acid, above pH7 is alkaline
Phagocytosis To surround, absorb and digest a small object such as a bacterium
Pharynx The tube from the back of the mouth to the top of the oesophagus it carries both air and
food into the body. The eustachian tubes enter it
Phenotype The actual characteristic that an organism shows; compare with `genotype'
Phosphate A salt containing the phosphate ion (PO4)
Phospholipid A molecule similar to a lipid molecule but with a phosphate group in place of one fatty
acid; used as an important part of cell membranes
Phosphorylation A process of adding phosphate
Photolysis The splitting of water using light energy that happens at the start of photosynthesis

Available for free at Connexions <http://cnx.org/content/col11410/1.3>


297
Photoreceptor A cell which responds to the presence of light by causing the start of a nerve impulse
Photosynthesis Making food from carbon dioxide and water in the presence of chlorophyll and light.
Pia mater The easily damaged inner membrane of the three meninges that cover the brain and spinal
cord
Pigment A coloured substance
Pitch A quality of sound depending on its wavelength, it is heard as high or low
Pituitary gland The master gland in the middle of the head, attached to and just below the brain
Plasma The liquid part of the blood; the matrix of the connective tissue called blood
Plasmolysed Describes a plant cell when the cytoplasm has shrunk away from the cell wall due to
exosmosis; the cell is then accid
Pleural membrane Sheet of tissue surrounding the lungs; there are two of them, with lubricating
uid between them to reduce friction during breathing movements
Plumule The part of the embryo in a seed that develops into the shoot of the plant
Pneumonia The lung disease when they ll with liquid, making breathing dicult
Podocyte A special star shaped epithelial cell that stands o the membrane on small legs; podocytes
allow liquids to ow past them very easily
Poikilothermic Having a body temperature which changes with the environment
Pollen grain A small structure made in the anther of seed plants that carries the male gamete to the
female parts of a ower
Pollution Damaging the environment by adding man-made things
Polyploidy Having more than two sets of chromosomes in each nucleus
Polymer A large molecule that has been made by adding a many similar blocks (monomers) together
Polysaccharide A polymer of many monosaccharide units joined to make an insoluble molecule which
is not sweet eg starch, glycogen
Polyunsaturated A fatty acid with more than one C=C (see unsaturated below)
Pons A band of nerve bres that joins the medulla oblongata to the mid brain
Pore A small hole through something
Portal vein A vein with capillaries at both ends (most veins join to a larger vein at one end on their
way to the heart)
Potometer An instrument for measuring the rate at which water is going into a cut stem
Precursor Something that comes before the real thing eg. trypsinogen is a substance that is made
into the useful trypsin
Pregnant With an embryo or foetus developing in the uterus
Premolar The broad crushing teeth which are also found in the milk dentition
Pressure The force applied per unit area, measured in kiloPascals. Air pressure is about 100kPa
Pressure Potential Hydrostatic pressure in a closed system that increases the free energy of water
molecules
Primate An order of mammals with a large brain, colour vision, nails instead of claws and forward
looking eyes
Product What you get as the result of a reaction happening
Proprioceptor (or Proprioreceptor) Stretch receptor that informs the body of the muscle tone
Protein A polymer of at least 70 or more amino acid molecules joined. Used by the body in enzymes,
cell membranes etc.
Proximal Close to
Provirus The form of a retrovirus when it contains DNA instead of the normal RNA
Pulmonary Of the lungs eg. the pulmonary artery is an artery to the lungs
Pupil The hole in the centre of the iris which lets light in to the back of the eye
Pure breeding When two parents will produce identical ospring to themselves for the characteristic
being considered; they are homozygous for the character
Radiant energy The ability to do work when it is in the form of waves that can pass through a
vacuum eg light

Available for free at Connexions <http://cnx.org/content/col11410/1.3>


298 CHAPTER 5. GLOSSARY

Radicle The part of the embryo in a seed that develops into the root of the plant
Radioactive Of a substance that gives o harmful rays made of highly energetic particles
Radiotherapy A medical treatment using high energy radiation to kill active cancerous cells
Random It happens without purpose or conscious choice
Reacts To be chemically added or taken from another molecule so that it changes
Receptacle The end of the ower stalk where the ower parts are attached
Receptor A cell or organ that collects a stimulus and turns it into a nerve impulse
Recessive Any characteristic that appears in the phenotype only when it is present on both homologous
chromosomes
Reducing sugar A sugar acts as a reducing agent by taking oxygen or adding hydrogen to another
substance; monsaccharides and disaccharides except sucrose
Reduction In chemistry, the removal of oxygen or addition of hydrogen OR the adding of one or more
electrons to a molecule
Reex action A rapid, automatic behaviour that is protective in function and does not need conscious
thought
Reex arc Pathway of neurons that causes a reex action to happen
Refraction When a ray of light is bent at an angle as it passes out of one substance and intoanother
Renal Of the kidney
Resin Preservative liquid from the bark and wood of some trees; it hardens in the air
Respiration The complex chemical reactions which release energy from glucose and other molecules
and store it as ATP
Response (in life science) The activity that happens when a sense is exposed to a stimulus, causing a
change in behaviour
Retina The light sensitive layer at the back of the eye; the inner of the three layers making up the wall
of the eye
Retrovirus An RNA virus that can make DNA complementary to its RNA using the enzyme called
reverse transcriptase ; HI Virus is of this type
Robust Large, strong and tough; compare with ' gracile'
Rod A photoreceptor cell sensitive to movement and low light intensity, but not to colour
Root hair An extension to an epidermal cell near the root tip to give a larger surface area for absorbing
water from the soil
Root pressure A pressure due to activity in the root tissues that pushes water up the xylem of a stem
Saliva An enzyme rich juice from the salivary glands in the mouth. It starts starch digestion, lubricates
the food, etc.
Saturated Describes a fatty acid that cannot hold any more hydrogen atoms
Schwann sheath The layer of cells that makes the myelin sheath round the outside of an axon
Sclera The tough outer layer of the eye; muscles are attached to it to move the eye
Scrotum The bag of skin that holds the testes, keeping them cooler than the body
Scurvy The deciency disease caused by lack of vitamin C. Symptoms are bleeding, loose teeth and
weak connective tissue
Secrete To release a useful substance that has been made by a cell or gland (compare with `excrete')
Secretin A hormone from the wall of the duodenum. It causes the liver to produce bile and the
pancreas to release alkaline juice
Seed A reproductive structure holding the embryo and food stores; a feature of the higher plants
Selection To choose some things but not others; the environment chooses some suitable characteristics
to pass on to the next generation in natural selection
Self pollination The transfer of pollen from the male part of one plant to the female part of the same
ower
Semicircular canal One of three tubes in the ear that is sensitive to dynamic balance, the direction
of acceleration
Semilunar Half moon shaped; used to describe the valves where arteries leave the heart

Available for free at Connexions <http://cnx.org/content/col11410/1.3>


299
Seminiferous tubule Thin tubes in the testes that are lined with germinal epithelium to make sperm;
androgen hormones are made in the gaps between them
Semipermeable It is permeable to substances with small molecules but impermeable to others with
large molecules; same as dierentially permeable
Sense Ability to feel or have the body aware of something
Sensory nerve Carries nerve impulses from the receptors to the Central Nervous System
Sepal One unit of the calyx, the outer layer whorl of a ower that protects the ower while in bud
Set point The level which has been xed; eg. for body temperature it is xed at 36,9 ◦ C
Skeleton The system of bones that support the body, make red blood cells, etc
Small intestine The long narrow region of the gut where digestion is completed and absorption
happens
Soda Lime A mixture of sodium hydroxide and calcium hydroxide that is able to absorb carbon dioxide
from the air
Soluble Able to dissolve
Solute The substance which is dissolved.
Solute Potential A measure of the ability of water molecules to move by osmosis due to their free
energy compared to pure water; it is reduced by dissolved substances
Solution A mixture of a liquid such as water (the solvent) with another substance (the solute) which
becomes liquid
Speciation The formation of a new species
Sperm duct The vas deferens, a tube that carries sperm from the testes to the urethra
Sphincter A ring of muscle that can close o a tube in the body when its muscles contract
Spinal cord Trunk of nervous tissue that runs up through the spine to the brain
Spinal nerve Peripheral nerves that come from the spinal cord region of the central nervous system
Spore A single cell from a parent organism that is able to grow into a new organism if given suitable
conditions
Squamous With at cells like paving stones; a squamous epithelium is a very thin skin
Stamen The male organ of a owering plant; made of a lament and an anther
Starch A polysaccharide for the storage of carbohydrate, common in plant cells; it is made of many
glucose units joined together
Static Not moving or changing; static balance is the force of gravity on the body
Sterilisation i) having a medical operation to make it impossible to have children ii) killing all germs
on an object
Sternum The breast bone that the upper ribs are xed to at the front
Stigma The receptive part of the female organ of a ower at the top of the style; it is specialised to
receive pollen
Stimulate To speed up or increase the action of something; compare with `inhibit'
Stimulus Something which causes an eect to start (eg. in a receptor)
Stoma A tiny hole (or pore) in the surface of a plant, especially common underneath the leaves
Stroma The enzyme-rich liquid that lls most of the space around the grana in a chloroplast; it is the
matrix of the chloroplast
Style The stalk that holds the stigma of a ower in a suitable place to receive pollen
Substrate Substance that is going to react with the help of an enzyme
Sucrose Common sugar (cane sugar); a disaccharide made by the condensation of glucose and fructose
(C H O )
12 22 11
Suction Having reduced pressure, which makes the external atmospheric pressure cause a uid to move
Sugar Any simple carbohydrate that tastes sweet and can dissolve in water
Surface area The size of the surface (measured, for example, in square centimetres)
Suspensory ligament A ligament that hold the lens of the eye and is attached to the ciliary body
Sweat The watery liquid poured on to the skin from sweat glands to help cool it
Synapse The tiny gap between two neurons because they never quite touch

Available for free at Connexions <http://cnx.org/content/col11410/1.3>


300 CHAPTER 5. GLOSSARY

Synthesis Making something by building it from simpler parts


Systemic Of the body's organ systems (except the lungs) see pulmonary
Systole The contraction part of the heart beat
Taste bud A group of chemoreceptor cells on the tongue which are specialised for taste
Tendon A strong, brous tissue joining a voluntary muscle to a bone; when the muscle contracts it
causes the bone to move
Tension A pulling or stretching force
Testa The outer coat of a seed that protects the embryo inside
Thalamus The part of the front brain that collects and interprets information from the senses
Thermoregulation The control of body temperature; therefore a part of homeostasis
Thoracic Of the chest cavity
Thrombocyte A blood platelet; a small piece of cytoplasm which can cause blood to clot
Through gut A gut that is like a tube with a front entrance (mouth) and a back exit (anus)
Thylakoid A small disc in the chloroplast made of two membranes; a pile of them makes up one
granum
Thymus An organ in the upper chest that matures T-lymphocytes in young people
Thyroxin The hormone from the thyroid gland that speeds up metabolic rate
T-lymphocyte A white blood cell from a lymph node; it is made in the bone marrow then passes
through the thymus gland
Tissue uid The liquid surrounding cells, it leaks out of capillaries from the blood plasma
Tone The state of readiness of muscle cells for work
Topsoil The soil nearest the surface which contains most organic matter and nutrients
Trachea The windpipe; an air passage supported by `C' shaped cartilage that leads from the throat,
through the nec,k to the lungs
Translucent Light can pass through, although you cannot see objects through it
Transpiration The evaporation of water from the leaves of a plant
Tricuspid The heart valve which lies between the right atrium and the right ventricle
Triplet code When information is stored as sets of three units eg three nitrogenous bases in RNA are
the code for one amino acid
Triploid (3n) Having three sets of chromosomes in each nucleus; found in the endosperm of owering
plants
TSH Thyroid Stimulating Hormone from the pituitary gland that stimulates the thyroid gland
Tumour A swelling caused by overactive cell division in a specic area of the body; it is caused by
cells becoming cancerous and dividing uncontrollably
Turgid Swollen and full
Tympanic canal The tube that leads through the cochlea from the oval window to the vestibular
canal
Tympanic membrane The eardrum; it separates the outer ear from the middle ear
Ultrasound Sound waves with a very high frequency which means they cannot be heard; they reect
o tissue so can be used to make a picture of internal structures
Unsaturated A fatty acid with a C=C (two double bonded carbons) which means it is not completely
full of hydrogen atoms
Urea A compound that is the break down product of amino acids; it can be removed from the body
in the urine
Ureter The tube running from the kidney to the bladder
Urethra The tube running from the bladder to the outside
Urine The liquid which is mostly water with urea, pigments etc which is made in the kidney
Vacuole A uid lled cavity inside a cell; in plant cells they may be very large
Valve A structure that allows uid to only pass one way
Variable Something that can change during an investigation.
Variegated Of a leaf having parts that are white and parts that are green.

Available for free at Connexions <http://cnx.org/content/col11410/1.3>


301
Vasoconstriction Narrowing of the small blood vessels of the skin
Vasodilation Widening of the small blood vessels of the skin
Vein A blood vessel that carries blood from the capillaries towards the heart
Vena cava The largest veins in the body; they lead blood into the right atrium of the heart
Ventilate To move air across or through something
Ventral Of the lower side of the body
Ventricle Liquid lled space in an organ eg. the liquid lled spaces in the brain
Vesicles Small `bubbles' of liquid in a cell
Vestibular canal The tube that leads through the cochlea from its end to the round window
Villus A small nger of tissue with blood vessels and lymph vessels, used to absorb food from the
intestine
Vision The sense of sight - being sensitive to light so that a picture if formed
Vitamin An organic compound required in the diet in small amounts for good health, they mostly act
as co-enzymes
Vitreous humour The glassy material lling the back of the eye behind the lens
Volume The total space taken up by something
Voluntary Controlled by the will (or consciousness)
Water Potential The tendency to take in water by osmosis; it is the sum of the solute potential and
pressure potential
White matter Nervous tissue that is rich in myelinated bres giving it a shiny white appearance
Xerophyte A plant which is well suited to living in very dry conditions
Xylem A plant tissue that contains large cells with lignin, used to carry water from the roots
Zygote The diploid cell made when a male and female gamete join; a fertilised egg cell

Available for free at Connexions <http://cnx.org/content/col11410/1.3>


302 GLOSSARY

Glossary

D Denition: one place to another ( walking from


= Muscle is a contractile 2
tissue 3 your house to a friend's)
type of animals
L Locomotion
H Human locomotion = Movement or the ability to move from
= the ability you have to move from one place to another.

2 http://en.wikipedia.org/wiki/Muscle_contraction
3 http://en.wikipedia.org/wiki/Tissue_%28biology%29

Available for free at Connexions <http://cnx.org/content/col11410/1.3>


INDEX 303

Index of Keywords and Terms


Keywords are listed by the section with that keyword (page numbers are in parentheses). Keywords
do not necessarily appear in the text of the page. They are merely associated with that section. Ex.
apples, Ÿ 1.1 (1) Terms are referenced by the page they appear on. Ex. apples, 1

B biology, Ÿ 3.1.5(209), Ÿ 4.2.4(253), Ÿ 4.2.8(274) Ÿ 3.1.4(199), Ÿ 3.1.5(209), Ÿ 3.1.8(221),


Ÿ 4.1.3(231), Ÿ 4.2.3(252), Ÿ 4.2.4(253),
C cell, Ÿ 1.3.1(1) Ÿ 4.2.5(254), Ÿ 4.2.7(274), Ÿ 4.2.8(274)
cycle, Ÿ 1.3.1(1) life sciences, biosphere, grade 10, biology,
D Denition:, 98 Ÿ 3.1.1(155)
dicotyledon, Ÿ 2.1.1(53) Life sciences, Cancer, Gr 10, Ÿ 1.3.2(20)
Life Sciences, Gr 10, South africa, Ÿ 1.4.4(45),
E Ecosystems, Ÿ 3.1.4(199) Ÿ 1.4.6(51), Ÿ 2.1.4(75)
Environmental studies, Ÿ 3.1.3(182) Life's History, Ÿ 4.2.3(252)
Exercises, Ÿ 2.2.9(107) Locomotion, 94
F Fossils, Ÿ 4.2.5(254) M mass extinctions, Ÿ 4.2.4(253)
mitosis, Ÿ 1.3.1(1)
G Gr 10, Ÿ 3.1.3(182), Ÿ 3.1.4(199), Ÿ 4.2.3(252)
grade 10, Ÿ 3.1.5(209), Ÿ 4.2.4(253), P plants, Ÿ 2.1.1(53)
Ÿ 4.2.8(274) S science, Ÿ 3.1.5(209)
H Human locomotion, 94 South Africa, Ÿ 3.1.3(182), Ÿ 3.1.4(199),
Ÿ 4.2.3(252)
L life sciences, Ÿ 2.1.1(53), Ÿ 2.2.8(107),
Ÿ 2.2.9(107), Ÿ 2.3.4(153), Ÿ 3.1.3(182), U Unit summary, Ÿ 2.3.4(153), Ÿ 3.1.8(221),
Ÿ 4.1.3(231), Ÿ 4.2.7(274)

Available for free at Connexions <http://cnx.org/content/col11410/1.3>


304 ATTRIBUTIONS

Attributions
Collection: Siyavula: Life Sciences Grade 10
Edited by: Siyavula, Megan Beckett
URL: http://cnx.org/content/col11410/1.3/
License: http://creativecommons.org/licenses/by/3.0/
Module: "1.3.1 The Cell Cycle and Mitosis"
By: Daniel Williamson
URL: http://cnx.org/content/m43135/1.1/
Pages: 1-20
Copyright: Daniel Williamson
License: http://creativecommons.org/licenses/by/3.0/
Module: "1.3.2 Cancer"
By: Megan Beckett
URL: http://cnx.org/content/m43059/1.1/
Pages: 20-24
Copyright: Megan Beckett
License: http://creativecommons.org/licenses/by/3.0/
Module: "1.3.3 Summary"
By: Jesuit Virtual Learning Academy
URL: http://cnx.org/content/m43130/1.1/
Page: 25
Copyright: Jesuit Virtual Learning Academy
License: http://creativecommons.org/licenses/by/3.0/
Module: "1.4.1 What is a tissue?"
By: Megan Beckett
URL: http://cnx.org/content/m43172/1.1/
Page: 25
Copyright: Megan Beckett
License: http://creativecommons.org/licenses/by/3.0/
Module: "1.4.2 Plant Tissues"
By: Jesuit Virtual Learning Academy
URL: http://cnx.org/content/m43140/1.1/
Pages: 25-31
Copyright: Jesuit Virtual Learning Academy
License: http://creativecommons.org/licenses/by/3.0/
Module: "1.4.3 Animal Tissues"
By: Denver Greene
URL: http://cnx.org/content/m43137/1.1/
Pages: 31-45
Copyright: Denver Greene
License: http://creativecommons.org/licenses/by/3.0/

Available for free at Connexions <http://cnx.org/content/col11410/1.3>


ATTRIBUTIONS 305
Module: "1.4.4 Applications of indigenous knowledge and biotechnology"
By: Megan Beckett
URL: http://cnx.org/content/m43060/1.1/
Pages: 45-48
Copyright: Megan Beckett
License: http://creativecommons.org/licenses/by/3.0/
Module: "1.4.5 Organs"
By: Dr Zdzislaw (Gustav) Meglicki, Jr
URL: http://cnx.org/content/m43162/1.1/
Pages: 48-50
Copyright: Dr Zdzislaw (Gustav) Meglicki, Jr
License: http://creativecommons.org/licenses/by/3.0/
Module: "1.4.6 Summary"
By: Megan Beckett
URL: http://cnx.org/content/m43064/1.1/
Page: 51
Copyright: Megan Beckett
License: http://creativecommons.org/licenses/by/3.0/
Module: "2.1.1 - Anatomy of dicotyledenous plants"
By: Siyavula
URL: http://cnx.org/content/m43142/1.1/
Pages: 53-69
Copyright: Siyavula
License: http://creativecommons.org/licenses/by/3.0/
Module: "2.1.2 Transpiration"
By: Zamekile Sondzaba
URL: http://cnx.org/content/m43163/1.1/
Pages: 69-73
Copyright: Zamekile Sondzaba
License: http://creativecommons.org/licenses/by/3.0/
Module: "Movement of Manufactured Food"
Used here as: "2.1.3 - Movement of manufactured food"
By: Zamekile Sondzaba
URL: http://cnx.org/content/m43091/1.1/
Pages: 73-74
Copyright: Zamekile Sondzaba
License: http://creativecommons.org/licenses/by/3.0/
Module: "2.1.4 Summary"
By: Megan Beckett
URL: http://cnx.org/content/m43074/1.1/
Page: 75
Copyright: Megan Beckett
License: http://creativecommons.org/licenses/by/3.0/

Available for free at Connexions <http://cnx.org/content/col11410/1.3>


306 ATTRIBUTIONS

Module: "2.2.1 Skeletons"


By: Denver Greene
URL: http://cnx.org/content/m43098/1.1/
Pages: 75-83
Copyright: Denver Greene
License: http://creativecommons.org/licenses/by/3.0/
Module: "2.2.2 Human Skeleton"
By: Daniel Williamson
URL: http://cnx.org/content/m43146/1.1/
Pages: 83-90
Copyright: Daniel Williamson
License: http://creativecommons.org/licenses/by/3.0/
Module: "2.2.3 Associated Tissues"
By: Tom Caswell
URL: http://cnx.org/content/m43144/1.1/
Pages: 90-93
Copyright: Tom Caswell
License: http://creativecommons.org/licenses/by/3.0/
Module: "2.2.4 Joints"
By: Daniel Williamson
URL: http://cnx.org/content/m43161/1.1/
Pages: 93-94
Copyright: Daniel Williamson
License: http://creativecommons.org/licenses/by/3.0/
Module: "2.2.5 Human Locomotion"
By: Daniel Williamson
URL: http://cnx.org/content/m43153/1.1/
Pages: 94-98
Copyright: Daniel Williamson
License: http://creativecommons.org/licenses/by/3.0/
Module: "2.2.6 Muscles"
By: Denver Greene
URL: http://cnx.org/content/m43159/1.1/
Pages: 98-105
Copyright: Denver Greene
License: http://creativecommons.org/licenses/by/3.0/
Module: "2.2.7 Diseases"
By: Daniel Williamson
URL: http://cnx.org/content/m43104/1.1/
Pages: 105-106
Copyright: Daniel Williamson
License: http://creativecommons.org/licenses/by/3.0/
Module: "2.2.8 Summary"
By: Megan Beckett
URL: http://cnx.org/content/m43078/1.1/
Page: 107
Copyright: Megan Beckett
License: http://creativecommons.org/licenses/by/3.0/
Available for free at Connexions <http://cnx.org/content/col11410/1.3>
ATTRIBUTIONS 307
Module: "2.2.9 Exercises"
By: Megan Beckett
URL: http://cnx.org/content/m43169/1.1/
Pages: 107-113
Copyright: Megan Beckett
License: http://creativecommons.org/licenses/by/3.0/
Module: "2.3.1 Blood circulatory system"
By: Daniel Williamson
URL: http://cnx.org/content/m43150/1.1/
Pages: 114-146
Copyright: Daniel Williamson
License: http://creativecommons.org/licenses/by/3.0/
Module: "2.3.2 Lymph circulatory system"
By: Daniel Williamson
URL: http://cnx.org/content/m43110/1.1/
Pages: 146-148
Copyright: Daniel Williamson
License: http://creativecommons.org/licenses/by/3.0/
Module: "2.3.3 Diseases of the heart and circulatory system"
By: Daniel Williamson
URL: http://cnx.org/content/m43156/1.1/
Pages: 148-152
Copyright: Daniel Williamson
License: http://creativecommons.org/licenses/by/3.0/
Module: "2.3.4 Summary"
By: Megan Beckett
URL: http://cnx.org/content/m43080/1.1/
Page: 153
Copyright: Megan Beckett
License: http://creativecommons.org/licenses/by/3.0/
Module: "3.1.1 - Biosphere"
By: Colleen Henning
URL: http://cnx.org/content/m43182/1.1/
Pages: 155-161
Copyright: Colleen Henning
License: http://creativecommons.org/licenses/by/3.0/
Module: "3.1.2 Biomes"
By: Daniel Williamson
URL: http://cnx.org/content/m43240/1.1/
Pages: 161-182
Copyright: Daniel Williamson
License: http://creativecommons.org/licenses/by/3.0/
Module: "3.1.3 Environment"
By: Megan Beckett
URL: http://cnx.org/content/m43180/1.1/
Pages: 182-199
Copyright: Megan Beckett
License: http://creativecommons.org/licenses/by/3.0/
Available for free at Connexions <http://cnx.org/content/col11410/1.3>
308 ATTRIBUTIONS

Module: "3.1.4 Ecosystems"


By: Megan Beckett
URL: http://cnx.org/content/m43207/1.1/
Pages: 199-209
Copyright: Megan Beckett
License: http://creativecommons.org/licenses/by/3.0/
Module: "3.1.5 - Energy ow"
By: Siyavula
URL: http://cnx.org/content/m43176/1.1/
Pages: 209-212
Copyright: Siyavula
License: http://creativecommons.org/licenses/by/3.0/
Module: "3.1.6 Nutrient Cycles"
By: Denver Greene
URL: http://cnx.org/content/m43195/1.1/
Pages: 212-219
Copyright: Denver Greene
License: http://creativecommons.org/licenses/by/3.0/
Module: "3.1.7 Ecotourism"
By: Daniel Williamson
URL: http://cnx.org/content/m43191/1.1/
Pages: 219-220
Copyright: Daniel Williamson
License: http://creativecommons.org/licenses/by/3.0/
Module: "3.1.8 Summary"
By: Megan Beckett
URL: http://cnx.org/content/m43081/1.1/
Page: 221
Copyright: Megan Beckett
License: http://creativecommons.org/licenses/by/3.0/
Module: "4.1.1 Classication Schemes"
By: Daniel Williamson
URL: http://cnx.org/content/m43187/1.1/
Pages: 223-226
Copyright: Daniel Williamson
License: http://creativecommons.org/licenses/by/3.0/
Module: "4.1.2 Five kingdom system"
By: Daniel Williamson
URL: http://cnx.org/content/m43221/1.1/
Pages: 226-230
Copyright: Daniel Williamson
License: http://creativecommons.org/licenses/by/3.0/
Module: "4.1.3 Summary"
By: Megan Beckett
URL: http://cnx.org/content/m43088/1.1/
Page: 231
Copyright: Megan Beckett
License: http://creativecommons.org/licenses/by/3.0/
Available for free at Connexions <http://cnx.org/content/col11410/1.3>
ATTRIBUTIONS 309
Module: "4.2.1 Life's History"
By: Denver Greene
URL: http://cnx.org/content/m43241/1.1/
Pages: 231-235
Copyright: Denver Greene
License: http://creativecommons.org/licenses/by/3.0/
Module: "4.2.2 Geological timescale"
By: Daniel Williamson
URL: http://cnx.org/content/m43214/1.2/
Pages: 235-252
Copyright: Daniel Williamson
License: http://creativecommons.org/licenses/by/3.0/
Module: "4.2.3 Cambrian Explosion"
By: Megan Beckett
URL: http://cnx.org/content/m43205/1.1/
Pages: 252-253
Copyright: Megan Beckett
License: http://creativecommons.org/licenses/by/3.0/
Module: "4.2.4 Mass extinctions"
By: Siyavula
URL: http://cnx.org/content/m43235/1.1/
Pages: 253-254
Copyright: Siyavula
License: http://creativecommons.org/licenses/by/3.0/
Module: "4.2.5 Fossils"
By: Megan Beckett
URL: http://cnx.org/content/m43253/1.1/
Pages: 254-261
Copyright: Megan Beckett
License: http://creativecommons.org/licenses/by/3.0/
Module: "4.2.6 Key Events in Life's History"
By: Denver Greene
URL: http://cnx.org/content/m43229/1.1/
Pages: 261-273
Copyright: Denver Greene
License: http://creativecommons.org/licenses/by/3.0/
Module: "4.2.7 Summary"
By: Megan Beckett
URL: http://cnx.org/content/m43085/1.1/
Page: 274
Copyright: Megan Beckett
License: http://creativecommons.org/licenses/by/3.0/
Module: "4.2.8 - exercises"
By: Siyavula
URL: http://cnx.org/content/m43184/1.1/
Pages: 274-280
Copyright: Siyavula
License: http://creativecommons.org/licenses/by/3.0/
Available for free at Connexions <http://cnx.org/content/col11410/1.3>
310 ATTRIBUTIONS

Module: "Glossary"
By: Daniel Williamson
URL: http://cnx.org/content/m43254/1.1/
Pages: 285-301
Copyright: Daniel Williamson
License: http://creativecommons.org/licenses/by/3.0/

Available for free at Connexions <http://cnx.org/content/col11410/1.3>


Siyavula: Life Sciences Grade 10
The Gr10 life science textbook

About Connexions
Since 1999, Connexions has been pioneering a global system where anyone can create course materials and
make them fully accessible and easily reusable free of charge. We are a Web-based authoring, teaching and
learning environment open to anyone interested in education, including students, teachers, professors and
lifelong learners. We connect ideas and facilitate educational communities.
Connexions's modular, interactive courses are in use worldwide by universities, community colleges, K-12
schools, distance learners, and lifelong learners. Connexions materials are in many languages, including
English, Spanish, Chinese, Japanese, Italian, Vietnamese, French, Portuguese, and Thai. Connexions is part
of an exciting new information distribution system that allows for Print on Demand Books. Connexions
has partnered with innovative on-demand publisher QOOP to accelerate the delivery of printed course
materials and textbooks into classrooms worldwide at lower prices than traditional academic publishers.

You might also like